From 1e484eebac7082f62b546ba2d8eb48381980d377 Mon Sep 17 00:00:00 2001 From: wieerwill Date: Mon, 7 Feb 2022 12:44:17 +0100 Subject: [PATCH] weitere 100 antworten --- ...die Neurowissenschaften - Fragenkatalog.pdf | Bin 144452 -> 151203 bytes ...die Neurowissenschaften - Fragenkatalog.tex | 213 ++++++++++-------- 2 files changed, 116 insertions(+), 97 deletions(-) diff --git a/Einführung in die Neurowissenschaften - Fragenkatalog.pdf b/Einführung in die Neurowissenschaften - Fragenkatalog.pdf index fbbc4cc26005cb39b0175dd3e5b898e032e9250b..b41b61f172d2b807d30fddb3f08972db69c665a2 100644 GIT binary patch delta 59681 zcmZtuQ*>t0)&+{jwr$%+#kOr#Z0Cz@8x^x+8((bOs@O?IxAxxux##6Pt@X6poO6uU zTOYmmxrWbRit}LWX%#?uQdk1%0Sb_aFN*6ixK>DE%v|Hk1t=4)k(rM6kXlC zBVZ^qor5T*otR)cYvz_e{Mzjw=NQvt8{V!=WloNj(>~{V?!*crdi5_n0C!jSiH@IYx=4dkMhx%A&9)ewe(~IB&UPIZY)PsnCHntG6+L#FW_-?H47nR`hNmCeD zALUGJ;*?bCJ+~xmK^6lkez7J5tpFn6BNH~9nuLm>^cvnVw+^j1H1hP zl`um^?!}3RNX#1?moLEwpq^&U&-%u<$>vb|rEOl}CjYIQZ38*KE^iy)uEAAGJYTjP zupqnXa%QJ<(F(WNXO}Hm(Edas=}d*A%NC4D`=Q&1*N4znva{4B`E_U65W0J~u=|@k zAjt6pT46H`b{k5_<`8eQ^WyVxPBVZGV(c`e$K6P`!}0lDBV7&*keQ;M?uwMGB||57 z#~SXu_T`nP5?wqdoqnaUu2o*lX3hgMZvXfv+xbgW`cU4!E-TgKXtwbp;8&WfeIGxd zvxlDFUhjq(#PT5_j+Q#nl9wq}R)h{tSt6adpOi(#4$USS85>A{nIK6QItZDPM7uuu zsNc4k?PT)E9-kHmSbvLXxEk|0dvZ*{*+t~Sh<9JN#b&ICr8K8ya50IiwYf9eHj70$2Rl%ph5dzQ;I^ipDK zqu?AT}X-PV~R3fqQ%Ik)3$4$L0bBUx1%2I(3)oiwB$y^pia@uPA?F)*@} zNBkvZ-nz19!ba&j7JRXZ-CEB?n81^sTYf&4P6vh6l}3Zj~?we&=T1V?XL=^TK}#Qr%mE+r{7=)w=iCx%yE$Kk)z2A!x}yJyzAmE= z&)k#T3oRiF!w=#qYLltLp%v1{`CTuZjXzGx2PEg_^`CRh*LN@Kb^P6N*J~tT4<+#G zfQUPbjIZ#y9CxGNy0tqW)|IzDCwPXxsNN~;-AVNeri@oaZeU*%HNIiC*uHxM{c9Kk z;!gPKkaR5tl?gpXLj?nZm5rS@g*A`{(4wv2y2*vobD>qp{j)6mZsfNbFpnyK*gktD z`_cILM+8e8t3)S>jr+$%7YT@FvVIhyrFda=f02*3b z{o?NQh;P>iov*q@*10GXEOKcVWQ67hWXICVZOp{wIGt@NRK(I3<54Z0>-tO|qtNsY zv*q^`4aE->iPT}56V&(v2!Eq#hhEgwp*W>T<9sUXZDu`Eo>7~@{ciEqCgC8B4Jq)!F_bOT6#59+is>@QLHjU2?hkG z2FIYBi`ZiTwf8lCb(Mz2_=05Q^J)+QkB${tZb7oXI|1ncsTwx z{H@zF`#_`Cl204ufBx5b&3T^TonN) zRea;QMgQL>aJ`aX7;qlO7M+jSnP;l*uf#g;U1@0A6~UQF0Jbfx5EZ|0v-?Y-uG%lE zA~cuN2u_erd##f0nkGw5wl%F!iM^VW;z3U4FhtrjeaV<{Dfzkd^o(bwzMf!E_gkO#u9MPqn=Kh#|$N_S_Q7z;bnG?Nhyi zkCk4&`omU=sFf>T>hlt)J@p7c>Zj(95biLRB1=d+XkM1;Fw`=?ZgcAPvlJfd0GEM# z3Lva?3B7h}pK;pbJF}ct$!bE?#Kh0cN1~=Fxt~Q2fV@n6!v$mg*be-azDM$3&h^*= zMCi&@PHS?P@fZAtE%3j-?sqv4zD6MzHOiYcdh&gKXzTH;ai%C0iA+BsNQ67auTF4>s;FL(E zc*QrLt7*i%-1?XB z?5-9YeLcu`Zp)hD%(8(=5LELrMqbv^&J1TFeVt3=)g{so0sA2o||}-86ImxC?A4T0)-!^NQu!Pcm4UFT7I$ zm8YxB<8l40J3y#welE9JI%Us7%eO)040k}^NmhFc^G$8gPpr}ow6aX>Yuzj{D2khc%M+lAE12EL7 z7=l{;HppNuCKseDJbyy}b&o(?*$0BA+@7Og5-L{4S9auN>D*(}L}dJbyu@>eXF@w0 zF836*;V~@|bsuGt(ew~j;pg`hH8dV8ueW^s6$C8a;_$qrc?lxl&79>Y&qg6C(ZxiP z2wA|!H(;$FnLy~rE`v>%;rhU(a%ZjF+1C=~EsrzJ0)iEV6uPW4X{k{VqH1No{+PY0 z@EJ<;9mW^7U75bA%ozz6?PG*8R0D5+1pHz9=eXU3zzC!adG5L)k8ovKROcOMm3kt* zbkrT)U<&f8exwC4|796n^3QeWbFcuu_rqzCZGcZ3hX^z?qotJ!iCmvbHda03tY5;c zPt!#Jnz^pA^TzWIr3hR&@6^LFgJovp0ST;_Upq_uW`QCYWJvopM!+Tc{!Q9-uq3At z_7_fN8?n$LRU7XqXu=OwlziJk3AM@{J&z6Fdjlcb2f;{uJ2!6ib@z!&-m}btS0sp; zQ$QoE)in|~Gzuw(oOTvyWF*4~%PmO*&cRUl9o%~gYLEsH(h4={03cbi9wEJVm|b4E zm`oz;cwr%|=D9+MkXlJ-3K5_E@+G20!q!5Y5u==iDnsOtqLa(4B*aylBOhyO$mIJ? zwMUt&9mKjlo8}au%oR!hF0)83{{vKQ4ltyTc)LHy;{agO5;)~4CvzGZ+lQ<4)07gs z_xm+?cl2QPV0gaXnDsU)%pDAcN@BL!wcS4ejF*2`QGHi0{If9Z!AbYavRuGu*!xlf1}7qnNEGF?{@}c@$uAC zq>*Fze|@6QnsgvP939zks1G==+m%g}7RqjHaKGN7lz7<|eVaDo>U|S(i?$) z>+pRU3TRG`e*3HZ#LcW!+UMi`(!yq6j-(}V%I##q4ypuSIyQ(=1o}SW|L%%_Hzhch zGTG{g-@;*!ZvQw45dO?#ah%>TI>moidS?H2`NPnI_fD-~i&fgkqzHhZuzqiaNeUCD z>#S$y*pis)u5sj|=tLZ3_-u-Cari!6o8_lxDG!M^(PfceJh0H@l`b4Zz84yfKs>~J zrhZUdB(So<@;;^Bf?U<=*I<~r+*S9%s%j8kp*G{_X-W(gytk;7FYF4c4$yLpy2`oI znF}6#6PXtu)X9Xp`#`yLfO6)@cE|z&`v$|PtU*p`a771YNpTj#0B2|WA1PeUl=}}U z9KHFAF%_51iW=yZRIa$ttf*F=qUqoo=^7{&c@T*VNpT?~}p$6Mqe4ngvg94);1G)Dp??GoEIEuF-<4qFQq$$W!a*1*_sFy zgK6|w=Z|-R>4UoUDRhi#UO;py?xX*5%jehNrgylRdjziy9&YZG@NRF;rwji(pBNts zP7#aPdnfe$h&g@7SlnyrKDV@27kKTaqLXWa0tM0EF_y78e7qi?XTO zY_k{jW#rW<0xe*Yh<@FAH?Rhmkkak~g>SMXv$Xs7=V(!Yoizlb@e0s{68ejY-zvt9d@IEa`eK$?b2HN@#uiR{p; zY;OP2xET!)qylh#gr2PuQ^Ex0s$%;y5>@}H_DQUIVJ@oJaJglEh+B$Kp)&~<9;nyE z1FmC)N6qj~&p&+F%m?ENj^APsTjC2F16YSb&*^ef`dNf0ZXK`U)yFo<*R@I$(%}9s z;KNAf={fg_>FXhetP)U0IAKLAf8puqytc9n!}TzS9RghPornsAz_=S3&2#+p2_iyJ z448SdAf&RvvP-Roqrgfa&nO7c{nqSr+xl>j%=#M{3D#|b_~+VgG|_^AS<&k;z*&3s zwg$|5y;}T35`~>1-=gJ&Hq&JYUIq(qD1-nQU`+XE#%hYz;k{w(2HUA(!GgdUI-p9) zGHtn>1R!SVPO~L|aji;++pL`7zs^6@#wPyiqJp?<;d>mgN}BqUBQESsbE16X6n-M! zp$jTRfEG)8^hf12)TUE!=YALdml<9bQ|?s5mItZs>gT4u9de>@hExLd49T9|!d*WE z=-BK9k?)DuKknb=yz_N`806MB&^AggyshHE33w=MqDWvnW7ohWZCMRZwkU6<9#A3D zg*~0aa@L`Cq+h1G-k}9bA{=zY7dmyL{e_ndjM8lN%&G^b?OYuYU;ysI zy}xVz#I>4c6Wbwfd3rNy;lXaR2<}M7Iw5vOxCrLzm_z8Uo(iIuOJ&<3A!j$lCA;@5 zC(YdJhIS?RHtvshyzxyG0a8bscoFA1qC&|MhRaW@lyn?|faG`QQ0E{N^v_ zKPatKKFASpKjLuCmS?xGX=RUd4J4Ny#DOqKY##l58o{<$PqL%El%WVTU}{F*HK-q3 z(MifvihlHM&HQ-9ny&%F6l2hZYSOkY3!kl%T--#jnVxVWIDbFJ(o zE@M}}|3EWyIc9bRR2Ik^_&vSk&>pysSbb~|ciWTMns=<@uI zvrFT+b2tHLe+_JMaML?ZJt#0<+5NS+6UsqVPFqJ1RaU_p`dZEEtQ^l7IVym4!)Id4 z90;}|VDIX=)+iFQ0Sh83(O1Ys;`;ZYI%s*Zkb^MQ1uCWfG7j$kZAm7#P+7?5oa#I5X2tX=$=qzTzM$Q@W8r$6ikTc#*-x>H50t9U%jL_!?fgMEq?Go?1}KE z5JZw?NF`P+w-PNBXkDncN#|;uE?kr5ble%&&rE@g zqCM}*ZbUls&)VwLqz;Igt<}l*Rz+|{^J3-aC(@sATAgx2;drW|KWuoTb6JHxxc<#x zStO#o6(mG3zo^mO%89bG6>AqKKV(|ymXa(1lyzOe0Agtw`5Cue`|LuaJDtfQRZ5Z_ z8gx3_NBl7xs5Dm?yu%Pzh!%XEu4n816EnC5HN8gD(G@$qo&pL{xIh?u3AAAlQLgwB zRG~pG!&c!eipv04DR7xfujDy)-K88|_q-*o!Lmmojq)O=4pGw=O|+(R8(DK+rxG1N z#Mcj{4C2=Ka9RuNR~_c3-iEZv!v*Up)} zj2~InJMJ(+@WPcbL`bIbD(xZmE3JXyz6jLlE6k?X`j;D&$??XMi*e$O#VvDzX3~Ad z29se)O$wn1eNSPF!NIjX%Y*5&{dX_GEFaICab>V3bj5%V*ZxRooJ5eAe*i_gHLkI-e8O00AsF~lZJ%I^;Z8TA%Fc2KLe20^A%JNK1bsz<~ z+F@Wj-`d4Fi%>vFWOc8<^JYTry$rYcM)&&1nWa7$gf(-T2E3-ibgJ)dG;+VA-p~Bt z$1AcXRAt(KfD?@UA7oae|6`%-|C1&&Fi%d@QB8V2cg z5ulOhDeBf-G+hcajrz8eb4Tq&lUZD{|{k{2p?)XzwM50qm z=dZ-XaVho3Xv|<`AYvS>=1*UH7Si&oTUBO9(ep9y+zsscnJ*SRHm~3BLIJ?4kcI_1 zLMS;wHOaSUDK$<{{m9<$GB%&7`*uBUdtr}^v7_w-%NE_{l)H@0wt4YHvIMHsWKmf_ zmE-_jbW`zaTQNWiLtd5)Z=NECEkBuKviSG(*+T|J_Q3Ea4Jm<}pXX#Y%tm3dSXX>m zvG5Bf3RbCvEJV4mT9P;!DL~%*dyxcPrcobZA;>b9^m{4xux=WD5U!ryj3Q%jmo#j8 z8}Zq6{xnKH`L)1##!Les(Z?`Hojc0CXLYeb_7#p^`;7jMv=jj8qnu$FWCR8AT50=4 z7_r>LsBr3cc0#}TCb|CV4qsyKpW(#XmbcKyi^4-5b%e@5LOa=w2_PCBq9{>CgE~PX z4H9E0q3#}|D1sDL90VbpiCw(UNJ#`ScngQ{Neo4nwZ$Y5tPOrIz41H)5?4=LJCPmG z4l|yB_nqv=TMb?YCqpXdU%jg(tCxH+5HI-%pe6&4JK5p%h1GfiAq!W3NbS~W9wM!V zXh(ISYDgoWA?;YS02@LtDV|Ps?{^SI)fw?URjz6b0ro(S-qCKcL*sBHRkfD-Z9*lr zrtnMXVkLAHp6fW;oMo{};>6WXp!k?ev}R_j!|AmTo2MX;Un#N1@+hRa>{WyTPz*8# z>eN`OTAo?woaJODe6pM#9U=cUs3rdm7zeLlWoK)drc^Kv2>7#Pq1Cc+t#_*b$lZMP z%=96W;m&i5<{8>GREP*hyzPz`ll{2p{+v@F(RB+Mn1>`)Ipvys8uCjWOoYKqIDrLL zCxj?rfs`*7xBQqnUKKW8E=D1>k^R2qsjdN&Er^u2T9hN4q{7l)`X-zu87rK#v5T&2 z1ZuCl0OL*$@KoI*)pb>?H{u< zgj@93ligG|SOvJPX_WrC4FY{y`nos(q)Z?figAy>66fCoC#e}H6o^7-4aB|2E*4=|Lpl3y{6+Ro zStwHW#^SY5|8G%#s;Lre*G|pZUt+1+;!ygu+@n%MJumB zPOjJ{7DloXa-l`lvk6;^p%e1K1t<1`J#@wT6YoJkRTv7_RrgW?DJ6lBtF3|$FT-qk zMqca{bJ5iPtFQF{{WDD#`%#Ojr?^iR7l42yhMeHsEDwit1w>K*VT%!s?OupAfOd^9 zvGm#sH3xwR8x}7a%HwRr2<*?1h-i02acc@AlP|bKL?Z-EDi$LKAvA(2laEI#7GpRO zu(^ts42(?IgSQY9(`pEA|0_x;W12AKY(g68EuogGVE5%4cjm4Zt)e`uLa11LlbOfV=UOol^K^P@Y744Q9$Xf3fx7APT-x}tu z0re-EqG(~i*FegDMi_wzCkG$mepm8j^1AIuV_pI*{Ozf)kygeKW4Fh(WaUKuq`*`%t4)4x-FAcAL zGYcE^>>B;v-0(Rs|8xZ80zAFdcDVHGZPFUxB2)<<2qpRZg2l56Ag>h+E_aqj`JQ$v zpW|c+i`pDms~W5e)JnaNf)6F>M-SYHV)waw((!{>hu(wH!2R z7;O*U`QbKgTy9y<%mzL~hP-X;&Anzb;WO0PoMcL9F*Hs#f|rlv{A;{Ex;&&g-_+u{ zDtQqzzr`2MLJZHZ46x?)WBl>^1k`=k+32EiM^Qgq?Br12qYVa)iC^?apNca+5O)Rg zo_rLK3p$BFwzjHFnSi_6!JZetrXPW-gTeU9xb}F^Lc(T4xlX_z;K z_&k@^=9HkQZDw9G!ZrG7z7+tpZ}573PLCN7w_VMx<3l=07PXkM&#-S0U9K(H_5d=E zW8lrI)f7>;_&f9SR*rllC$O1(a3*u?v--68qq6a-)fB7ZBX=<_DbwWfYC?-<=9gJ6 z$RB5PiVxDvoO!S&8h|}`yJp{D(`M}&a25{cu5KHTR=>Mu5?2M#H0ZW@`COmhrO+IZLO!WcOJD4`eEd7B2ir+GI? zm?)u1zUHR0*qjj#izH=$zGovAP|2GG5LZef)gn=vxNN7Dyuk%cLxF&%=zpuOXq~Ucv;TGA{)aaGf6Cr|+7JoW^z$5sTV%Mq~N0imPm~ z8@=UGOvep!pMi|@bCKG1f>V5}`A?Vu44F+Vtq6Pa2rOc?0D79fJZ+p8wVqSzKhalr zaesg4@@coAELI#a`qkk+r=E`f{@Q79SE}Cakj9bDx#>+h5~u81nRb4(;85bo^)Y$t zKZxk~6Qs7mcMVQ2ADcDk2DSbeH$LST4&){du>=156>jLkk{0BVDrdXyGp}7~7~@y7&e_n6EPwsyV)!7Q}?N+04p+ z751vj`^=Zqv1PCK#^ATWO4YS4wr47aa}aJerHuZo9_sQyt{ zf$SYIX}QMM;_Q1J=T$OyQCCas--0K>wr7d{>?e)3JOMbX4uZ5}8Vl#_Qr{w4m`&K7 ze~ZTcw`c%V!p4V)G)Th1)!uq4X}0bA#3^Z;KgLF%#}iGq<(Rc6QRz=#yz1~^x9;W# zB7)>R2dxK_FKFrDY;MbWnCn3!#TS{6s&sfnb1JUFsG5pGT9SF}G66C6u*!>!W%!6WNA*P1Lo91qV=or^$?XIMJx+ZVwA)dm z5MDODq9sj1yrLj+nLAIPbKuMmTzSsnbaSY{qOfkP;*6Ta4mk4FCfH7R*(AhsT~e|~ zf(tmv{|c{}2(DW0snXP~lIqaCDEo;rBN?cvE6NvmfW36PcMFQKw$Qys#ID-!8&Q}5 zgBK)VQ05CEIcO@HoM(c6IAm|u_qksb^3b0wQlfm1UsT+OMbMykm_N8JZ38-*7VX;$ zw@QREt67)K=EH8G1CdelhuZba{R!ddQUkEtoIpZQ_YqsRzj2>}hJ}TOpm-ya|66~0 zp)iqN$n_$}LQ})5NbK`IJE*UvXYpp>a{UYmtjPL^fmqBK4&~HKX^`3?7I#}_4~N(T zzt0%J5=8qWBkkf>(v>E?7$l3Th6ooD3sQDTBbwLVb$ef>cRz_+t=hQjQF~8S);6H@ zK)be`m86?{$p$|+|7kyamqT;<$CXM@5^?QXS(<@eJqLYV?uUWW$)9yz{+G0cfY$|^ zr=QjA0`bYsne@8BC0m$ z0mXoaeT4IJKOJ*Brs9h*PaFuuRC{l!T0v;Pr;#9BnXy%8c5pX#B$tM&FSl2 zGUc@(+dz_O_S+?e>cCH=huDo9%J#fZR(TLuX%#3Npi=3NK=Z_-SA}p0N|x!h5`t~6 znNeZ~GyMdHwhNSgb|mL?FJ7lXlNSgxoOXa770mRvzPX#BFY7h7Ty?NM`Ggy(yRvkT z=5J+@_C9kRq0S)A<1IV?k~_1o@4*uVgQ!+4F48n0RkQ0x<_mWRdrSIXYAwt{h+m<> zrJoc<2S)BwW&W+3wF|$tV0vgZY5}*!28gO7yu?^uOoZ%~3Tn^B9zKVsJ(nNl_51+# zhVX&{14yApReSy4cy_GYj;!u_0@0+W+lK|gq*f4?sktIFVVOt8IEv%*_Qf&f-^}4F zm^_rILyt7!DwotR-4?i)Brpvwv9cPH{&>IF;!@gk&PnIeSQ4gbChP*);%L|})a~sb zdE(bH0xX7^`oDEJ;s`)NS;yumgk=G$?|Zkr%rYc1=QJ>CF75W+sAD1VMPWrqKYry%9rCjW`w4yAmij_9Z+~^=bQz)5 zzAhH3xsm2Z#?zA4R}~G{s^I3X1hNAqajan56hD2cqnv;uvD}qOVk7Ni!?3&mc!{g6 zY`g>h=^=CZY+VK`*4d#opxYm9e@;cAE8;wJ_H{l%R#@6SIE#KgvdT7nt9#B%A{q*0 zlZsB)sJ5$@8iD4DMRnovbLFelN&BthP}&%QH_KeQH6<+8JO#l<7O5@HbJ!>r;urxf zS(4&5l60g^Y5X1os&g&_Y z&doL}1T4W-R0j$*+njKzHGJLXh1Zr)&Bbk#hZw+0Jq&f5DycYDnUmrF3=T@PlFHg+ zgM6*01>6>`xhcPoQ%>{EVKiBRWH7(ZFfl?$pmw8pDdw(YHjwafih}2^>D=h-F7?x_ zrJrMRx0W}+uO&(V42iOVF+ZG&X-X&eN3|8h(sl~wBK8qr$X>Z5q5Gt!7PT^U6EsyPz}96Kj&xNdlS^pgt%Zd>tw#Wle;1?iw@;C+S@nFz`1`=- z_+9~ML=*0QXj3Q!TbXlm^6%rlYj|>3Z7(I9G#%ON?bhV}Ov}p9yX-BbuMMpqkTTkm z8jn?QlH#HrWBE49^79mNKIcvjx#_}ghdfX=k`h$Xxj>F{lD&ps*HHYdF5H^2h~FM_ zHGT2od{clf-|ymDE%K!P?(bga%dG^;+zSl{FNV8l0dm)5K za>c_l2rpWFI$_yBBD!33>JjFMf^v>Hb7+7cK>)x2y7>>)umvjFon@-zP^&KINZB7# zcDr5?8QTf(i1rk))>Q8si+KdNbFk5oJ!m4g&>vHCIzWhGL_3Hg)R zlS2S~x0EF}-f?f8-rCqkte{6!#@qvOyKO?rLtETi^Fi;5ZPRPWA~D1t@%UwI@mG<^ zAvG;fV14V_=+az0^Qa%6= zH~GY=U2qUkfe0IH(A704N|ks8W&fub`@GjGE{#vPO;0JK?XlOW4pa!x*+4m5g{HL} ze#~6LvCED~@^0a_w-6|Hw~7;=x;BL%RMzi#{KlpmqVGRjnpDj55_bEB4Yrra>t-(! z9+1geb2_1DPR5%93ST$RSpuPES84!bt4hCth1n4VQ&c*F>@0!otm`R1tehm-%7b4@ zb`!Dl(z!HzKkU+ zdYlG<`YhiDhRdV)rj!~6ECBOlDhJuI${nL1GT+uqW4=i zR&hAKCPPPyB9JYfizm(N;vqnXv+kFRX}`ObjMgoBz?Yu4)*Z5{JRB`hNP%@+BznE4 zRn}hXVw&FmipbYcGJQ0)+VD77tZGJmQ94rS}>Ja1hVM=PXk3q~7 z)l>;$?+aP0D(1&OoTUDjEa&C^Kk6r^wk7#5MD$Ke#jSFjHzkL6enh5d>SQ;;s$NRB z7xDv~Ka40R34*miNRW9y%wQ$eMHJ7yt!%mP(-RqDn5FK3AlWtFPjy*)wX_FS##6`hN`W^O+Gth?z!1Q%_t8{{ zl136ulO%nQV8te-HGS|wFM=uX8VGYD5r3Vh6S9Yg%^Punm&f$Ag%F6FT+(0Z#R?%J zZ#ofeg{qotniZqNs*m^nSy0ySesKb62`Q$NZ;~wg?Ob`Z=09*2K9fO9I&K8btHk%c z?HGqgtbXB0SL+8F@8;VNxHnfWBbHy^(NC}D!b>}RLch?n-SkFQTcK=EYq^&!mqm7x zNon(-j$~gWlDdUaed!W#=sr&3vKLNOXzDrBNe_w1RC|S@9E_;X4B~q{e!;@fVN$JHdsW=oA2R< zX%W19zZS^|x!(AyVh{P-#sNq7z4)LSG41bB@(2=nJ>_D6R~z0KKhHp=MBaOc47idf zMqE7!hIh6Ha}LmKhm5a zNn{6-n=$aZJ%;7LCnxi7CsfsG9A;4c!f9gDJm2r)893Q4Mg+&naXDl7erC**8x#3E zWp&=#g>l<6pRB51Qn`CYK8RzshQ5tt0f)=RC$}v=P?UcUuqcEWKv>&_JkG`8iX*-S zaHfi)F&sfyv|>)cgPfvOfto^n!0H5Mg5r@;kLPKo3w3$?l2k^Ni}ZK;t#G{ReyFw5 zTC*#^el`(ej;+)eWXQQwA1#uwc`?HgZ}we+u(AV&B^1|l5mV(rh_7xcLq?D_dlEcp zvpUD?sQ&H&;3pDQmLIE$l45}pTg+crAaGUmJY*#gmow}8uD;uo2(rnS5IEX7@x;q9 z$~5vK8wwd%W}i^T#JpDTh_|*Xq|Rd;X-GC&D%@Q7T9ElksB3Zk5w}ble&uAgS_1}A z?MZL&tZ?qw`BeBDXbESm0fcUF8lL~-0Rx4d8;N=bNSbIj7+fH()hry=Iq`A9HYV6o^98j^=-11?s~4@DQJS!NcPBICqK zNEMQqyAG>*&bRTjgQQHix>|MZNraieXAIJhK<8%d3H$ra!A8`RpTJ2OjV%Wpn(^|S zG)H3eUl$?Xe1Es9z>QuAc(7-u^nCNcNc55D2VNrI8RJrzHVptf1)1CLM*{p%8EGtW zoze};aIsXFwi-}@ut%9}1B+}tUkI- zth4n`nLitvMSurTA$fDTO+VY6&@yHwbB)s0#v+qHOLplIjy45<)HiWg@xZ!@c3R-| zm{M9ZvbCyNPm`G#OBX~#|iWNzO1*b(GasU8tG>e7Rl66YXm4RfSPx2?tvtzxI*bBOh^y6=- zQrV3+uR-2~{H8yZun9XhFSoRlO(k@>arU1_HBE1NH`rF0((zRK#tjFiZ!)Ge@Th?mNJWPjTl)Wzsjy4OtZ0?y|DwEQ`n3ruJt3R6>kzt4| zXBL3zMOS8BGxuA)B%u>LA8Y>0Mr|zhai1W9LJb3_C>tt63 z_pKD*H}-;ENBy?ueN*5%Az3W7s4tlxw^K%13}f>!moC{KMMlqS(70u;1gXSdXz)c5 zBmm0Tfe&{&B{EL&A%(-(kC-GhSz}50gjAmR)afBx&+6=vIvmtGob%qY9Z0Rk=IJVZ zOd?e4zLw>T`n)Bh3m}%e_4Ndy=UCWIVy>$tZAnhs}Fe+AtuxV6qxm+8T*e%HP>MHZ#m z;<1}Mr;QwB&E$d^1jP%$D}P`PPaG<;LAW4IjUb)6G_;`j)di&qc-i4jQ`>+J-n`!P z@sIiIPro4nWr9O~#a)aTC56O*Bqu#d+UtU3HF9B{`{*7SUNCb0G3j6GWT>+74yc-W zeqwH)iR93;qYDc`DsTq*Jik%yMRKS*svOrm-tk`D44Bi!5*w*LB;8?yY^fkHzXE|3 zj!HSl#-D`fHk*I1ntfhQLp4yr5vW|1!F8YwNO=J+`DPHJGw~~v97$#Th3=c1^to^m zJd`~~F7(Zf&vEie9Z!L_*TA3$0GLXnz>gLBKsflZ$zpcW)CG05RAl=EYg3G_>UWAHVl>+=N4;@-SGw@%SgWgf0Ic5G0&&`Nc=^JBx|SOhJmFz+mkI1ePcNMU|k zuP{pNvpC-!8##Em!6gC$7Y?7|R7|!s>!GEYycc%Ru;gB(0eTx?62dIg<%<~M0)AfU z#D8a{t8A9Rw(id?(Lh)$6=AX<7-^=pYXX~*V+gHarl6RcHn-xWQLdmsJSnx1x9U=k zKwD!K0Rl%hX0Ok&!w>RnL(go53G@C0)_AV?S)xw;6^CgfRZNZNn00Vv1!Kja{0I&d zi!ve_GbUWWD$PDXc5MH=L96#q;Q*u65ykTA0bzPipVQ}6_Xeh>PIe935C*}ilTw*4 z90_in78-aRcTLOc%MK>G^EM7xJNY~o!bPUi7J1?2_8%x0?MQ`q)EH#I_{GpaWYzHJ zq*aLXCfhuW0M=#g?7_^kAluh}E{2RJecO}nvd(f1Tqyw{M|-JT1JpH`4)c|ZlhUqs zrDbb8&in1nZqrX-tS85%n&S(a4n_{f90xfl&G~bKHNMLKnq zMHiEezgDl~ujvua8}~)?Mn43_M_nZ3q3!qisdAP&$_C3 z29h}Z8ia5q?q6D}I0lA-vHh|Js?+z#aiewF*b2W9WW9;boDj~Ns54?cfo_xI%8ew* zzK1{Uon^_u^s?dPy{l?&7B2a$DG!OSE~*Yt2}>4ot?_?#oq`j^HMb$K9(cDw?d;DG zO()s}8dpb`4W-Qnp*I|d7CgP=LyPOAL31@<^(?pg+H;0sEpfR{%C0bf3BXhUY?z^Z zGRo!*Y+b92jiYlfiAVl|NYux7;kbFTOe^BWh=iH_-+t8~F3d!l5l~IAG2?4AecM9x zDm^Ccs-KZjLR$4Zkag6RvP7CKVnzg>GX|Ve+dco~9lk*g;88s;V6I#a~+= zt5s4MyYoUl#W^^?Am^Yd{r^AU=6}&?7GBnWel6Yq`=bMai67I)jI5!+qP}nw#^%JV%v6dZ*1GPZ9BQ~P4e=6|9kcHSM~H%b?-UnOm&|%r}wP2Hsc(` z`HgWWvyxFVK}uA8e+yOOkO3(j8(h3uamci*Sgu9os{| z(+5XgRFBr{DvYF>!Qyu$j1s9%4DkkKtk(j-9eJD8vzlTP^&+m{?WpquQ)ssaPBF>%2}pu`;Tf`z$aV1$$?Gm%MAg_&Yg*RqjVb$H0_I9n|?82#fr!``#HDgLn`A zopSlxa4_i<#-$A*fE*J{wG^VCWk?e0u;KHmZVoTG>&%#X2Yzyi+;2BDc)Q6NaQC+_ zRqCUw!D6<;HGXF?`kNZFhB>;F(0!V(X2(5SNJMr0%}a*_?l*Hn9VXjsxDgS0KlnJL zG1LDvx{M_Se6ChW0Q%_wS0P~i0bgN|9DWy&aTr6~wRV7od>=K|qPH}&_7}%P--4@{ zkG5{=nCKXKRIeVkLRyYp5K#LMh>$pD@>!FX5lznpqnC>G$Zx_{3TXRexFf$#FgS>* zk*Yf=%N#(~4IDw#?ObPpiy6uSWJ|)ulN!hpH+e(N-+WGQ*xAq`e9ED4b(MEr7R@cF zwZ)-G%L!ej6pBtg7ot*C8k0yrxl)rghJs#*%WZT%!UdbyCeZ1I2nd;k%sb6yNa=YC ze>K`a#prYcP{{fHsn)LPhe1)xwE%?e+56zg5VMs$3M;`tY>L3iN{af1FfnWz!jmgz z@(4@sLX-0cCH+-esAAAPEEtdRXq_Gq(3us}!>BLeH*iIiT`-mQp&Qqwd$}^G=L za*H1|W#Rfu8#*UC2G|2iFTuo7)iIhm*JcTO{m*UmlP{Kv4?^R|zGA}`E)xO4z<7s0 zW~cG(-EVD(0b@h_#*#7;GBKnJ3<3N;R#9 zbY|@nhdL;FTV7%scd8E#%MY>0%1Xt|pY*UM&sD$i=JMV2b_$^0Wos)hzm6wgt`S1p zM?20>ne5hSSnakeFy^78TcRO{!1us*eGjTIo-msUnk|0!2|7U7+5FpMT zS`%Fa?P{1+BJ!fi^*)tMnMRlagz^ol8Tu9Rzu?S&iN{8}iFZs(5!|N5t- zDK2>4-WGQc<;wMzKlZsWcOC7>TuwHlicDH1$4ZACkHSF_ zraR*Hy%~B8a;hgkF!8Or<{+rn&3VV{+0bIhGQ7w{OAgNN8IwdEW9dauY910OQtth6K(I7-#?@rF!zz4f7)+mTT$;pY8=>p*MTiqbrvB@KG}a3+ z6bF!Q;p+Qrp$nB1NA7uWxYSKl1AiqGV;tij`tPv6t5c~YdQ5a?1fkfVDqa}j+G236 zF_&MhrV>yX)cH)syR%Xbr@4P1o;NY04FUjVP%yF8jPJ3x-&P%d^}0M1)(gUemTdf_ z=#y9)B(aH6(t*VFV$$fSgY<_T-_t70!gVI9)xztXrQVk22_#0`yY;%EU4rhOABxR)TP&(-fX+g_ruB*K(kdC9nj;Yl_0Sj=#E~D$bKJ5|;sX zAV5xKNq?C($DjUw11di*jz$(7-MY_fcSY>s?cX-fU9w3-Rn^A=#6wJ&a)IuA_j$C+YZjC%iR;_hvV zI+|tyJ()Um;_DcyM`$SPZGXIXiiH=V#8mX3Z!RsM?(}|rzwm8g`<)>jrBOcgi{{#o zy&Em&$3x7k@XJcoZD*Vu1{_!fienHXr{y;ijj=qIC8!i zgUC>R%YY3#S<`(O{$}=z_wdLzbj%RLomlp6o0<{RfNhU%hQ(|)xRQXBL|wF1~|OmVm^*dD4TW*)B}dwljpVgf54 z<}t+fV=wMsf*E8ENzs!BgeC zlD&4SKPNExHMBIQ3ki*1Ph9nDin^HQM(p8TYrHWgM8$X1(x=rpIF~cKdb4jWJSRd- zk^p7Ie5FQQ@MuQ~|84Av&%Qh#oICe4(?|Jc;zy2D`x~Q0o+r}zvoHB*FY6)GyO&cE zawf-2S4C8DofrEKL3o{i{+NhRS7PZAj%4ZL)j* zxb3g|YV1{3!|m_R4WBRVJ`3JpDX2hzbb>bwg>ev!>g>+|qRqdbLhSDImRWtC9K1HO z_ty{&KDf3#evC=wP3c-vSDaJ?#!v<8yc)4fYhUXZ@@SXU*lZ`7Zp%xuSqklpJV+ zkT|Lg|A8ZBvY%x0I)I%ALY-dmVeG7dFArimna6)BobfaHlmotDTU~SdoJ1B4L%3s% zEKpbiybwBazu~9H_z{h26;zxS9ZXyMI5z|RTNO*PuRf!P#ie_{JJQi>EauvZZtSh6 z`$}8&#U}?f?tvh$t}58&XWfP zt)t1?!%yY4+ZsPWT8mfl8*{g2I`vap7+@=8wrR~6qB7PU(xXHB<*#9RHJ__$Y zMEFNEZ;;rFe6^tA@S*h_kdbHN$a`bwFeIi<3x0^x>&NA)!A?&mX1FWbQ++zL-Ox2^ zJXjnIenG6%qHvn@rU9zdGl2dlmSV(55qXdhQ2VnmUOplb&0M8ELJx)^c!`dJ0D^W} z7*vane*2gZAIW}Dg3y#=NxlUFCc4dl+Pmy`AU-3fp|h|dE96k%;GF|x9d3wzD3G@h ze71tfySsqO#d+(PVrN$v#!V~SxH#$otaz`121zS$4+mtMJ_2R40Du5V&Yw7l6{9Iq zq===$QhC1kIX7}Ghk5utuQROW{p8W@D`i~3JW=1dI8N~?x3UmfNi#_V13rh_;o-YE_dEA9dC zAmPy%S&fOEAYkg*UbdBOF&F1#u&%DJ*h0o)a}B0k?E{D=EqO4eonDQ! z4|7NB)Be4kz4ZT_J_biW;~AehKlBSlhb?^kJUSUQHO+luGfN@iPFiMfZg;xYt|w{N zJiYZ>7(EDIjsb+V248Oum?MB)`?>RLe~(T;e5%>JZ=+w258!40Yx?HXIG$VWv+3hH zKKu4YAWf4QBU{5%y;J4s4X1m1YY^-%ST{c&KYQnJ?UU)58R}jTei8W{scatUi&@nU zv!>cBiru!^cyyxfqB?GbJ8h4#We<+_&X{@-JUX`31h9Ux7FO@pL1(?rm6*1kxzLc8 zwu1alRULR|zQ)kKAXmuIwjrZU+v1hGqFPms)dX3YC0m_1%B>_!V z!~7Y|2cVR8;tOT|_DxoG&EAlq==Y_W$ekZCwV6rcA`^WD<+pX=YZY~E!jAF*XX~f7 zn4|w>KzgrGucTnhjZsb=l>P`zS$irZ~F${f8oV#zIu{dp=cd$z8ji>Z;+>Tw(Z+DXDt?9#A@#VszWY%9l)2 za}dwsFz=+?$cIdpC0BR%7=a0= znv!NC-Xznpy~IygRq$rx{|g%7{vQcl_T>5^)W*43C@`Wb?;?=I55f-2+rs}V^8XRY z$?~7fZzEh1G8hrJmjy`9H#nKXk=g$e$;JMkHBh5cS{EpcLcCtUJEA=iP+59E3J7ZQ zd_Q`lK-MHEOcE$&@Lx>yzBjP{o&fm|J+u<-zmAsFjTU)yU@$|CNJg9Ad8ZU-|BV1< z`QeeGHVzeig2Eh3fk8bZ`V+qYPpcqo|Dj@`HU2Fl0E2lL1K)c?d}C2I_|M1CfLR-v zE7iaVy-GmtAK*#f^r8Qk5wmmr&+o&nF+4mw45OU6gQcq#5i<+ZfB%b{{(tMH;*WqN z5bQ7VV8EOa>|^oZG^t>)H1?G|B%o=llO|yx%02-5k-Fq?*@)zl1r_NGN=vF& zQ>sY3Nr`U&y(EoEVV*--MG)XE2c^7bSCEKH9zg(?kQ$ASm695H3g7gGX{Z`edS8shKys|J;Y8j^wc~vAf|%DwD2Ej z0i*?>YS6Jvk2K1hHA>LLaMI!?KvFb&RA$_2>mNl*@NZ6`Mr3#mb>dJZ5%UB+)L$)F z)@YU)#_C{IAza9nVfM_&br3@0ePu0FaFz5aqiWXiXwEKwrGN*^303wO5s&x%Y3ppM zOqNIr$q%Ihfw2g0V$$HE7%eR)PwYF;0BFh!A5Cvht-Go(cnVDES9532lhf0q$J5!h z4JO=n9D_EM?b);SON~M;*cwwV*;T?Y$8*ME=EqwUwgokrPP_c(izlPo(>FH_?YrQ1^R2FO+T$C-#On(LVkgNJ&@Z#AmHe1)!jT=EV~&J#D8Gbq(}m>rX7y0MLM-lqZj*Sn>Zkk3pYH$rid$p9@tfm`lmrb_f=0ZW?3!R~ud!U;=?fLDNv+GpkeA3xcdxc-ayC;h)c z359Pmf%XkEB@eg%mp~Tg{|5FBmxB_?%%P%>6&lJWy$by=k*w^E5#ukwu;d0>v}5mJ zvb+3Y=}KzARE-Uj61eG^y};;=@TYKuAnYuyxDdehcp&U-tx+Vv>&zfOU~aAkObo`! z#m$u7_Y(-l#m2#&oQOgRuxm3-V9;-&*K4`hk&TI20TxuNl;x#5K! zg`a&2Zh^54h!V3K5ECs000Sm!4XE0){>?GN|_Fb$mCw%1UVW~ z3mrl~QU^8xkib=t0EzOV22>ph9=kKlY<(BV$L897FXPkq9Ruia2fyoMzDD2ktCB|n zblzAL6akT?^{{H$8XFR0I~{0*!LNcndMjf zub6LWZJuad?~E)eosNH`&6$PMf+(N7HOBAxeu=E?^{&mIirwde-v1hlDW2Uq^1eOB z0UNI`L2wgU7c?ppY~QRv-wM&|*qhl}8=0VLdSCMkbK^$b7Nav(q*8s>>eZO`ocsCKA8EUb*TLbX$l!_8mi!V}?N258jjVfySMTcSlj{ zK%>&u*>x)GwYEvbpc9F%i3Kx2v^j~isao$l2(Vty-T>pC){K#WdFHE?KAR(;B z=+;QAX{t^+BoVG?dqU3t6aQR6oatB{3y|~xQ^t=F{1U6zez^g^RyF-tOjZQ0wvkCd zhtM>umwi{qUtyZ3IKIjeLh0SK+DHJPj(JtZs`tKLP7y;9SHXs3Zh;mPkCe5ito~a# zzIM(!C-URXxeTq?JYfJ{Dfe^rE#I+gtM;fVP|`xa=#QfsoPujkb`sJ=)=_Q9#HrY< zSXcK%NTB1AxBAsLr(_lEmbugG$v0u1pYEgAP9ax9Zlf811^T0TY+`{NN zV@&KcQp(e@IUglt$006eq*=aX^3+TuGh0`b)a*+M%46Tnhh z?;38Lia(g1Jqb}7%h`tFrHuRVzvLHo^!Pnr9i+j^uh|zMnfyB@_o$Na?=4pl zQ{e6(_cR8Qu`)=v^h~`k&iuDXXR1bXzNoS>ayc_jQJO|~Nc7TXE5vXS>NEE*%M_>` zi*?2RZrxf5S}DIBKB%JQ)BQd_CL=}M-EDUl#_cEp!!@PA{zO#3dcol~r5PbS&Bcw_ z;B|V0rzVj7*`td?g%TfZc2ohOpZDsZIO7Uwvj71c4s6J_xbQtW+MloL(KO>l5dGx$ z!hIBByo{_S;M^dhI;7f9e0U}{^+a*PW zVrAnkof&Z*PoXB@dpapA^eWz(wB%n?_g33>&DY-gDiFaTP_C|2x641i<|u~~ehhB; zP1(A-v;A?mxad}^`mnaDNM7#yOO7p_`R4}jM*Vc!>l`T%OvxXAeMs$GrH0Et&vL!` z1}YUB7G&^ewE?>E3^@I;S{AP%+OvTs&0^R|qzd`NysW8!H4P`smHT8>BLtdc-)ln7 z$4;CGp+{~#U<5m>{AxN16jSL*IhM)j4*1AOv)JP^xF^j*JtszD;B+o8VV8wS&TK4! zbTQscLxXaVmyY>7(9|ci(tP;51F;beQRD{OO#t-fWf!`ikyORzF1SDU`QJtVW8m?= ztJfxo$FhFFAz01p-BT0hhLOaxU=C7Mp*uR7CdxqW1~PpXEp0oeU{}c{ZlqLBB>73U zOO9u+2{+6GF$F)%ec`xL+X-G3mnEmisqy*2t);brP*s~P%kQztR}|PC2F=cB>Ih>h zvnMp;JgOh@b|$8@kG#!RMCMHoWdo+s)Sz_f^`BV4yUynClr&j=prR%Rj5nCQWt*WBKIy+y=h_w*4( zIn8tNS~`S~w3bIu?uYEaCyaS#3sYo=vTzBgvF_{OZ#GfQm~GIT+pECW48>^YvgKD6 zAS5n%Vis)#nZTzv;Ehg*^LF17zKj$kOTTb{u#Pb!-gntqv4MtuD>uMVU$CxPJh*pw z${=)?i7!$n8c0L@H)uUNqe`d&-Gfun{TrprMb^2bwSGq!ws!50+t6osGA3S>7e-#CDjc~@pS|r*w@B2v0T^y5M<7-_DtH*cFt2uo8pmu{8qx|Cu zO%v6%CjV+{A!v<&9H5yRyqbivf(s9=%Rnba*`NcaPH=<*((v(NIZIzzkqklHWeIw`eI74XWL@j-< zLHqDj@13q60ZV(ST$w)|5|LhfNM!z%pBf?lw6s9Zbw-9;JRH}$;<}$z1qepi7#)m{p(!o ze9i4D8e7+}X{=QeYuQu;KM@iDfx`=pWmaeJkT&<_Za@?%i{(4Mwd8P2L{)IH79Z2$ zKD_V``ca%RC33+&ovY>h1`5O{bbJ^2@Z@A3#lTC7kwg$`mK)))HX-G6Qh-&SN+lZ<);Fw7p@_caD+M7#tZFd(CVY;zDjM|2i zp(etCrmU^?&$UzAXVvp|&^FXuH?_({(pBaV+kT-xzXVKghjZ)I)BftPa5d*EICpkRL-l_J#{kbLeFJI|PO8hZU^K<_%AH;tZZyKl)1| z1*u^*q+6yNMdAiCt|0@Q)*)b2^}oFev-DE8;4ualdRCF%Pq^|B{>u)}&3gW?=-qL* zT3O2r>uM7INDMYH(zm&o?PYt!k}K#R;#>L9`@h=A@>}{xK}~A`ctuE%djYMXPiAe4 zHSy_rf{xoMEE;vvM=&wC_2L=_vaRrmdP9)KA|4bPO`x7^uREHH(+>_*du|{;u(a_g z*{V`r;?WyYS+`vIl)PNWMy;~oog`b_h~i_SfvupKq`Tnzn`}YW0{-J7CK=5hK>G54 zDGyaBJ?L{juX`MTN6HW1Fl0cLaFAVE_Q?I^GD6e=2AA~4I55iLkO_sJQ#!{mo~E%9 zX7>C3@bKUyaJE>((gYRGbtO_pQr#tW>*+k20&LX3iFLP& zaw20PM{WXF8DoR%n|ES4|HG_!o?L38KQhxpjJ!2!uxvqWuOJsvR!n@~l><5A$MaAQ zEHdt#ti;qx22AMUzx-mlCNfau^nfdDbwCI`Kq}G!GSp8<_LDa*2_<2`gdP`>HeRS$ycg|BUSl${wx&OCr-{&qsp74$heAPE&P z;~V$xXsbE?GS8#+}k0HFE}jbqffFnc>#VjzWdfmuOc0MFH?fq#1O zeYQZ12~q+-4jK*n3wzYM+D!^@F=fllzxA>rpaPa z@GI>bO%7?QJp>$MLOHmIrGjS7PU8A^ao0&~u{oHwviTfV2y{ak|0hSp%efZX$_U@> zNl4sppkuiR0y_mxO_#u=Fqh4OS7zpEa>k{5ZD>4g6#e0LxSrE4A0MyJKT3GtHaYE5 zfEU~AK64f@$P)C#f!xb~2pd`Lp)%v)!KI?l7ea#3@zlX|hFJR~eV9SZ4o1!_UaRS^ z0nL$w>m%G*tHFpUFR{|_q-c_bs7f7T9+L?wUg?=0U(mzwQ6SfvCFx=pVs-TILiMUz5f2iQST6DWS{^-xZq;Qnyce z7SNJLx;y~b$TXzzaS5CF@Qd?&oE)>JdL{Ab(j?flCMDLl{9IEk`_ty12dIqasUZ3H z>NkW?B_+SQJm%|>;R6djlR*J8Xa=Gk5#zQgWiO3ifR2`C`Ad5UyqnhRs*wzKI#MV9poG z44%I~ymUhj|Gkj19jc^h{r;0+;EZkVBk1`^-m%dewDW1fQoY`_`N#&z0if)Ct*5>kWDg{>zTf-CNUJW>=kZb>wX?Lc2Yiis!W zOTgWYkBq9)YHE6~fInnBk*v3oMhGKo_U?N#73?Ml%217;zr~T3qJIFzJkmixM&jaH zZ{6t~mi!E-0`!^=+$2MMN7ENc zCQTZtq#e0EOkImD*rGv+-Qj;JQ^ENJ`Z*_@-=Kc0@&Ytc{E8JE#h{JO8h=qw=(PzT zgZVbZcJ0#HPir<@IRzillo9u!VU z@g~>j8|J4MEdv}IHL)XD*yeeP4Ge6S)D(a++xyd#|CF#h1D;myFKBDd*jK*$`zB#h zIbW}wt-3X4$-dB@UU|)T!03vam6s$1ntB7FxuN9Lq3k#NgS1RW+c<7xvjsR-4#7`O zc@Qn+2+pP%{j7B)FBWc=+d$MoT3pMS<2tpdYED7JS))_+M%27TS4(g9&u{C~vCOOs z(RuE8YTcoO0S81f;WjDpYn(H36?9yq^8^-AcG{hbv5)mrd0dpR{Noi7pswOL0-e9biLcTNq);BMZzm= zuFh@>=(AbBQPN+fa3_noM=>j%&Cv}(fwZ+Zcb4%n-JJKYWt7OrsH!kVuHs<#4Hy8p z-`*pas@RgZ8iF(-V+ZmXCJ!Tlt|kf3{gE^tc+@IT5=I_x7FZR+aOfBY0%}T;`InRb zt{VA9034x>v6jtotk4gjX;v9&ZvB$sWv!y!>t~bfG7{D|K9wx7HR9gbiZAPoJt$9p8rHo_Cp z#QaMWZV6ru(Q>QU4HkbilMnBjg}IJWsJh;t!tF|7q;ztZ1qs*eX?^O@K>Jfor$>7L z9DqPera|G9gHGY8$yUBwMWc^wZ&Ccf8_EGsIiIHSbzVrAZlS8A*0PD;pCNB(SU_v}1`a;_@&o>0-!w# z4s+Y4YBn>5lM^VOlu7P&P}F4q{=%7K#fB1rJTcT1w;z{G%&(h_8Gr}hne{70TNv5U zaE@UfP@_TIP^rBGD!R^^+}$YZ8+J*zUUzjwm3DQcVa|kk4}!%tN9C> zazdYyih96L*&h}Q5>UOa0BT$-!7=W%NxV{kFMJ;K=KS6NR@Ra#W5$kq>Qx_ylZ`(Z zZ@Q-Z#r#>__vbDyan}l)_1~t@s{3S4r7E}O%L(4SBe8cBnlTEN`u0hyD0Vp`D@j>t zEFabol)4)fvG3mu`~yxbtxC8L6LyPp8(rqL%@arsoouP^QNRc}#N~98S%bQv+-e2) zG)^U!D7)&<^`X?iXhF0bUj@ym(hj;WD*8&+E%!^F*OFN|D60{Y=2RGS zG;mXyCy3tEDNa66LQJR1%AFIte=BUAJh?}n0G2*%Xou_Y2ai`GM&~7J-C1Ox+eBSF zDS?38sAbV+eL#JzQ-dTf(4u4I-}k%PPj53_*eY%7tO}e|UP^MF` z%GYMuGQgbaFC&7Y07Ybr+m+9F$*h5bDFnS#5gT&xyjyPuG2~_^>%CwbNCNa${Ip8v zolb>(a2~9{iRf;eJj6lV1c`1FVU(eqRv+@uax3%F4LRWd|S?yE`*6)FG^hQ6}4;a z`!AyHd|Nz8^8+%RZ*$L>^f2=B<+DOx_Rxd7Tlkvoyn@5TuM%9P6x3}N!-~Cu_d$N^ z4gkr-Q$3Y+9jjEY8E|Dba27#dF_9HLD<5HH!U}lU`*_4hBw~rgq$s@Hhm>XxPdkp# zzsT6i0-eCFyPgJjgA80wBM{#NA17;KO2z5KSsNA@cN;KVnWeXGsURl5WIC}3=+zuWv33f^EoW#_l-!4?PExb@iH8lH$o z{-C}WyB619(4pZl+8`5S!hAy1mURis75`sY%6??6xVokt>5IBLFJw z7#%0-*&%FGc7u1CVpusUlVfulSut0>&z(XZQ7!~2h|r?C_V-9WdDd; zEd&Vgm5gZ8U$8!OKBU-U$~o9M^@wQ?LU6!vq<;@J-5^XSR5#YSa`~chI2urI;{40O z=uH#7Ka6jdFEV+Z0=FE<_Ai6m4*;pdJFFU%561K-(wnKy!q38PI(mPy!?7*bexL6i zkX{zt&*QA+L-u$@y8V9Kz43ZJphtN?b89CzKADFQWg)QJ78HR&|C1N9Q-#D9uDiPR$-~{McYrp~mPEbQ{ZJa3B2(K)#Olo3gmy@nCGR9;A)cTk zQ%k^G&m;P~fv#Mk#!zqhVZ8Yv;z47^D{G~$I#y*?h7K`rPf5a&q0=Vl6$O4$U_3mm zX){eSerR6GOP|{sx)*2nJ`#Z9chs1uzCJvzS?~1~!fuux$z}v8C09rwd#`V>K(a_! za(*5s@%35t>+@H}*gj+l32BT;KJR`>?+6XKc-YWIxKiZZYf9{km$S6P(vKTx8MVXa5 zf3km)J!0a6NbT%KbIP93-sN)V&l>qnh}Rq|w=cqec;@XjUY%`kL3AF_C7v=?InltV z3m5qB=MgTuCbzpk{_z|DG2=f$>@p?n=pEw5{L3pZL(6X8x1s^-(_;uf1+|D*3c*!%xBdn7|T&Qe@CDZL#O~Awpx!!=vKO z&R4EVA*?CcP1y&ujH#)@!7SeDMKVt*??dW@mm+Fvz{)z_An~b?}6b!6QX^pB3*|WQ>5aPEv$xjE^@AkU;6;>mxBsL^wH2gJe>h` z{vqxLR^z#&Td=Ogm4GGe8v^&3BAPhtC_r!D2qtk6=JGlXTqu4O%nUL9hk z=_J;F{6p8^@8<;E2nptDs}-Ied)#z{jp?XhC-0sks~VJt!&KqmQJy3TB?I*QF* zQpVQT@{BmZXX)7b54-1e`3h5yao9ywX&SUS)?(})up0qC$#1MOTMZo;^|Zb8$%E{3 z=1FG{XM~nWnX*j-6nw)kXG?s?)?ZZK7QDul1juC|=6;*FzLmF~<|nk-HoNtOOjft9 z!ieDSMq3|k-1<{kJuhy-`=o?Mm>@Gxg??i~5jkFtqRTiU@n|GG`P=b2>I;cf5aYCi z`i&<-mu-N5`u2UiZ0nQ3%J#FkHzrH?ZL*I(E69-^YM3a$1V76+S^kx@x^8g3sd6QY zRSl{N{i0!*&M5DT`&72`R(+(h7lAGgqG);r!?_#^XW{cqZe{u-jVrlJR@OWs&VASh zNpUwhNYO(nmEK*mCIA2;0Im{+sLQKA1>vB^Fn0$OB{)U=%wzQi7hJ`0?H8>r*00-M zgx&X8R&rC3wUBrplL)JO)Fpl-P8Cvgtxx^G0xT|lZQ>?H)LZ3m`g<~KGqriLAaQBH zSKdmrLv$6I%U^GL*!uOZ-Ga3)&E!>@gS<=huH)$T*P6}Jr2Zj|Z{Yj@l#q}wzN>%m zUm1XKhy{o^eADDhE$J6iE&2C(+|G4BX`PTR8Q|nIgm7zMealX5+ zvP}ON+-aZM!gQaA)XOWXt(Oo~VB9OC7MDgLRIJ_U*Pn&TS)*v`3nQzwMI5=0qo)CW zyEH_m^lH%%RqnZV=9CmbMzhS(mD*^BbHf(Bjo200^VAZoyoVjN6`!wyLra%aOglYv zvSG|7+JV`7hP26yFF{(&3DEA|X!As(xp-L>#-~EOugII~IQ)ecW`rXxapwPO^1DOm zk7nai36}nGdsGiqe`#)}``Z#J`t%RbHzDZpMXw_79b*yp0d)S=3~8L$IIM1N|M6mR zUGDB!wbI`1`MUnWNPKMQ*cC>J!6Q&+Q+^+5m9_3~M=^Fz5mU9p#pC(3`jE=nl}=kw zLTJET9TeP+ukyVombQV;Bd^C4!g;_=s>DWwn$KG(bxESAgz?+4`OG`s^m7pqk)mqm zx75NzT?wXUplfV{=ugVu9%Okb@-nBJfC(Zh?IeL3iqcxM7K(nd`$KH$A9wDL{b=fQ zFT|c~jD&hm&mL9j>lL zK#TCQG7Uo)F@z{Ov9$TGe^Z8(%01%F_>#*?lw1@1r82-P+MGsIs=)@p*TvLhRI^}H z9NFb)^I%JF?NV%FS8*IQ@!1_2>&RSC{z2(N<+Wa8ma?^f<45GisNM9tm3jhTV0Q)bA9hcA(QipGxl)3!WMW_bou5$UQ`xSU*gJ`}8 zSQK%+Igok(g)7ltrDo>~DP7erA{ejlkn?x->LPE0AvnN+Xg5eB$v75fnA0ZL0Ee3J z_7>SX6P#Ess55i&&R&JXKdUuJJ-JwMSK%HMh^SB|zq>W$VDx$SeWW!R#=)!I`n>H+ z-d{;pQ$X}(=`(l$21m~ECM+vI#f&IV40{Hn*Bca$nM1>MxsTPu?w)xhM8g%2q-b=7 zK&$1OH5jN=`XJ$kR?j8z)}tOrvv$@i661n5Ar|4{0@}C6D5cXPBu{a-nUQr5tq5mAgiL{h~a&1M>6$AIUF}21HQ}V+{qEM|ea- zxn#c&&t49eoXA5#s5aIwpYq9{@c-!oMO&$3=fPLt*%)xA`8XwbQX` z&|T&JMN%(cT#G}E-grZ;?x6`6>L@}{%-dZYVxXjQM`b@KD+|)BBMqy$E8{6>ja*H2 z$Zt~?n6^+6jS{4M^#rFcuF{3Q;5i-h~95>>v?il`E@3HkW#e1TT>8OCX#C0AD}i-`(NhOx)Pyzl=1a`as0C*vI5?W`ptO7nMq017~nGQW& z(?1F(#YKPJe6KKm?|N0KAtX=X(U=#RR(<8|Jud1nT}kn#O*z%v^>a^38W#rwNU{+ArG zjk=#TWHn=@GIf=vv(QMYTWEPgC{yMx>tSE`%1uG9>99it8-v%6p8J~{c~DT`1c~!P z`tE-Zg7cttibmYH<(B+z*04t$DecmBG+j)klb9OoYZG<0Z^GAI#1}{5Q6fr6E-dEy>ueFRSD4`l#+jA z!VnjVF5!7SfxNp98;X9*8^2rDoZpzG$>84}+2Yx~g>qV`jr_Q>KCDy6Nv&KoP}pn4 zx3}AT122Gg9PPu)k&S_oItL1dZvmXm-wD?Q$R-uCus3kxJkhX_LwPOCAG4nC*pnJv zi&ql@nL^obcTjVD#VXx_FT|^1Pv(E}l%8nKp^oat&i!Q&2OU*WY#$O`8)08xO@Psj zMuZ!ED1*JB37i`waR1Q;nK2gklb}m(gh!6jz%xE%6#zHUh5XZ9gZ;GMZPs2Xp(*y^ znRBmSQBio-EEZgemko=r_)MFO`hWStgI$;i0WDe{_0w3H1=CI;Zp$PeqZBm$g-j&yNkPKIMO3->Fsg zevDlFtlaa=EwhtOu8xyPSQ&ZmG&@Hjfd;dDL?zCy`vnuVpDrb~wF7{}EJx9ncs+pf zwn$7++L$>kgJSeOiBr5kX-3pF+$cLuP@h zU%d}zt)dVOWqW*|IVfaJ|h=A&D~cN-}9C|cxUW?x>e5K+3kQoP?%HWGW7`5FhZxg=Cxp3 zO($;ni!R`zp8e>EiZc@m9@%5kZn~7@7h&;NSrAVYuyzdWYJmLnsZ4xgy6e~c$fZ}TXt~zNKwfx!>Z1?YK#OAdM2HncKIE?f* z;a7v0aM|UZFzVveB1FlWD2S=(d;VsCb5?Ud$m(EAvemo_KZoVVc4YaTPCL}T=!E;( z**pSNu@t!=&AYSW=yRwUUvfMqmN=hHU&v(I_fGW#6sOQLdhLG}&H#HVx5h|I^96w= z?wFGri;x0A?HztIQx~xwg);LeyXG$~WXW9w7UaVqP>N>F!=)#5kWHsqCuQ9m*0P4( zcsdF>R%mSvIXm{B^bH?)*3iy(;hsC&jwa13XSXQak7f9J@2y@h{2+GajORmIWFl|o z1RWnE$WE+wPiB8y`K^fpI1NiI;+mgtSBW@R(A(spQR66vxWm8N=38^aG~nNqP5T!o zUdI5cp3Q68jg-rbwf5h@Ifntu%zE(BalxmL+*sNrSwBbb#Uyj`W%#f+o+FS_nE(WO zT{?+IR0=4uKH9onAce%bY+`PKEGkA3>CF|)RcAH|j+cM5s`y(h?)COYu|EQ30uaM` zXHa&poG7N~*8@Q0CfDNR7zx$^BRj4)Aiq7=qnzx2;OYCxugP-Uj6*+@%bJ=*4@FaO8B-_Ob|8@pXN- zRo|MWsttctKQBAat3JaQH$L~k8p&{#;$W4NA90+UoD!rppQy6!=OouB3{Eg$if7wW zmUR%GttCq?k_m$a7$MndG;Im-56%zJOA{sjIi=AW7#67uIuQxIB+th$L3h$qW-lT` zh3Rz!6Zzktd5adAsJn3BxE?ft5cClA*#zg`s9ApoG&XFlMuIwYnD`5|I7i1F2DaVM z0bmd{yD$p^hs(+D+LvVBbGk%?x=CTze|>7Iz)OGGrk4Sc_ofT~j5nY#I1vltI5K}6 zCE|%i>c$G8kyg}}>@{N~8|dSc^P-o~hh^xjE*+1PEPHW%@5hpEgIv8A%Bb}9Ye(K| zuzY{Hb@W1DFjZ*4&Aqe}*rR62j<4#oynp$HrYYJ5tnb{aVFYX z5mi%o1UcfdF)^+%ITgM_7VPxXrUx@q0M&D!ULT=w4YbSa#b`#(c9hpGJAgjts$K)RB!EitmyR1j>LTx5IzD znh=j=WYOd}JWJ89!3zq*No8NZ7BBMLp`<6J@Rdb*)g#dDy05!L7X2hT5SJGYeKs77 zcqipzW8(LsfYh7Ygt$&?rl-`RablWQ-hC@z&!Z5j2Cn3lhP29mCip`m&ezl@eD(C2 zWiCim`iO=oH6gtD(V|ssB1@pKlGWczZwPV)5%((H;tFKDYm>6@03~G~7 z;OPFzBewzN^N5Uw);|Rx=2UMq!TzFAQ=Z;<5nrt~t8ICQT3CSGtsk?MlC~bnTQ^#X z2qca=O}{7wTQK+-4xPiNuAkiy(oE$f;f@N(WVXf|Zt-Jw#qazwL^}mcY*2sLqnZ7s zMOppq6J~f`@vh#tt^Y;kWp}lmGYN5_MxUYmB9eH^3#(TftFYjqurNaG@Em6ZwRxl~ z$9g1VjvXEZuN#r`n`wB{(-_ z@so`Zk@x@#3W#4`eYxf*=5@s_jDB@?X=3Up&>;(PhXq=h=D8TiPO1pxpz4Bs<#_N< zVg(rpP6_#ZnADlrCshuY zzdm|DY@aaKcIaFf?q*;dW9{m38G7EgJWVRc-6UUjjY;syelGhq z)}70?)P8JR6*x)pknG3Dk&yjFv=!)=&sWRIp{}ww!C65TfVyCYP^hCl>Ee&3V9SdN zd7xX*Fo(W|#;ILC@fVbi*@n);Xc3EimF6{zAxrxVr-F>+i;>op#k3X)Jet?+dlT5W zkZ~60=n^{ZQtp5E2&n<)UKY962c?vwTk|YvGGui$=ejJjqgG0P81axw#vW3#^RvrE zis??a9DV~?NH2NrNP}FltlH0xjC>6UF3KrGxF-oGW@jJ+-I}D^;QuZJuO11`r>N6r z$AGroe~2N6ipm5YLZ}GjNRKwwI2qO`Q5lVWKqm413SxhTkp)Be;$y(aKbLJ8K~EEQ ze6cEIL`s&AL)|OiUW!l4A(ppfE8|VF^)zrd4oEBO9DqX!D(4s?U3ze6)O8QNR8!9j z#)3{Q%z45_JV)hdDU{|8>`Hz32YR&ucQ3 z8I&97;JJTDMsc?5EkDIy{^ytUYUH!U7$&vX1xm1r9XS%E3-)D0cFr~&SE4LvG&zHT zXG6XPyBt#ry8wyVk`xSY7wB#n4pBr$r)=uPiQX}Ik+>`BmrKZ5ox#B^k{BgUETKt2 zLcrLnm?Fjm`K3?4)icN%MadVXTmXa)Rqa*%@#e1J+0CWvpk5 zG-(XsO;QFaVv!|@^ubtHD}*?u`0?;9f`qUsdD$2t0BfmD_4)~2E7+C-X^QAYdJhnT z5$Sz-VprQff`70emvkc+>56()=>*w<;u}qg(?`Tl2a_avR1-|C-kp;f>WTo{LZ=SR ziq(IJ^!lQ%VPGL_A?T@D*@Z!qD9A|QIsa$e@NCuAOCWuU*|QU!9j76dXRJU3e9r=Y zLKHey>{=uNJkrxbGNUyKW;G~YgR_NOcz&Yy%l}Rp-3cNQ(a=(JnQsrFR*-^eL|@L?qu1e z^lF&}4skfX2c_6&lw>U+nj74Wf%vgGXoCgWSo^@ z^No5oS&NnXNcX)xb1G2U3qn7w3Sh2-L=0ag(=cC{+qcw@TbUwd&o-whqOxbN^~e)m zNpATbwYKz8pdE$<4OIA39_blRtvSNy*;)yqzojd@Zn*I$af}69+H-%dDQ_XhcdeN8 zVm_h@2UHa`+^5zie!a0@cA~~qInX@hC)t*IZ1P>LUpbEH_5CTW@`_B#?)^+YP!;T* zqa9fZ>%|(8<6p*5jvDlvG>S}vlZ^kn(;YZy8lQ0)!4mR=#=zXP5Y48hU2=cNBkPoo zWDxlj<*O^WM3}{%Yu10#w-Jj7mna!EljU;fick3??~nt_EA56Vs-!dv_Li@DDv|H)b?Eul#AivdgHkD zHI9Nb^66-VXl8Habv~Gsbf-{Z_m7P`OQaPmjV0bLcg9y%rh9*J^D^m%b6=JortRsz zaK+RWhBd)+Y*mIflgE)?n=FR-``Dx0jFgup+*m4{h^#G@L@ zsB1{2Hl~t4d62ye1}5*UOyHybyo=fX4AK%aw!Hx=B$BXb5BzVh@I4oAEF}(y4L_Gk zxR2(}tx0j4;m?1LkEKk9zbJsvg>x{XMgpjF@y@*!xI!Ha5twqM$g3E)`1XqAXdc?n z9M4864SwY9UGybp`iNcwq(Z|=mp;FKFZ>ufD6raxAEtT^-^?7mS9^S^IYXbb@U?+l zP%01e9X+^!OWy-|Ya*mJIYq4iiDwe)#KzSExhK%fpPql1KpJ9kjX}@CM@vgv*B!rl zR>*l07B}(q2lZGR1pU33*$JSbjLrdy!W#N){8a7l(w#|xa*I>AqwDmOcJ}lSPJ#NZ z&he^66((#e7W&1XZ)b~F0NKUZ=`MeTSm36m}5FN z;QK8MGO>TRc+QdZYa;Ma3ZBeS6*+5%-GeOhMuOo&z(rCp{M3fitF zmv!t5tj62jxz&SkWx>F|ve-G47&4Ip)1pL$ehuqy!BFjYbUBRXE(v`G>@WKuR*V=S z(|iT+1aYN^lUC-l)KN7QL61aw6R>rU)BPPa^ErPY$DnQW!+;8gK8BoZN)qQAbWi!X zd2cyKYKy$}3TL^E^lL5C6UW}i^y>I6ywC@obWM@#2hlk|dRwzb_gSC^?lHaE+3U3D zI)B6uP7otWOu(_sP&}0|v^n_F0!arFi}tTq;z4X=5})HnP=Z9#`xWeW^b21wj2Zav zAZma7Z4I^WP#$>eJ2rpXl!3S~x04|-LCtqRUUD}i z{DEP^!rOHXI_hmjgAa2w*XGKl{5pI#NfNr?fan|nq80pnADP&3$dVscg4%s z5$x{EqHkp2je`r)c;3XFH!XT=b6c_f65U+&zO=GgRW=W)O;_jUpt6!kQ*mX4_kMr3 zPf@r&L+jSL;V@*E-{8GUYcR5fOIuTu>1pf`Qgbp-v^nttD_oCZ8&dY4)hYtJDx^|X z_hh>IvQz%3a#iwH_v}iXs2}YSV~HXFKa}5$F2eV=IV@_k(2Dy*Gw(d z|0jwrbY0TcBF{BR;3kQ)1JWorAzw4Et2*+g7 z#XgD}XQ`#YFqQ9cFe_kSLiP0n0IQsd^D8_!OB-wlnaCx|rkJJl2$Vk_w$*>oVnTo| zoa@oLDn+M&Fasz&cc(C3Kq!73UDZ-UTdIqvVXOxiXrcC>U8wr56@5;3j+T@Z%+<9k zDW;xZyAQ|Vp{RoeR8-rWoHqWi+_Y!MPoFE~s0R*|ARn{(ENuu=Uuvz}qHJ6lz_6)$ zA1yKb7)Nj6JcNXR`oO&zS!3R3L#KK%dY)4@PO{{}!!ZAz&ilDBUA7c0Ea;JV;bzx7U5 zaLp82530{K&r-`CzOL6>MYd%M(C2iM^?|p@8z?JzFS;XQ8WWomc_V)|>F5%C0J$sG zTYI7q>(G0&_4?W|heJ4{S>S+pqu#4Q6tkU|JM+lssDqDZvyDh&Ri`?k!)-O4&8ECg zx!}eMEtOuWSpF%P+5=nG>9=hz0DFmkrOt0&L*=O?mu^0|n2iu=|D>OY;%U-xAp{~F zjo--vMon-6Rtma8H~D`8SKf443u$Tefh7pdeAK;BLY%rqv0``oR6h56P~F)QlPqrg z+#i@Vrk;Z!bZWm*j9^>$264c=?vFCF&=E2PEurrg5l z$uV6`i<45iQUWp!;l#O98hievEXft1PUw+|o?7@+=Ed}m9A|&!sK>d6VP;k3v2Z(R zwLwFP1@j-i{bTDrRb_$ZJPn+p0k~Lt&BNzQ6k^srzS5xV_gguI-k4n6l7yy=WNqEE z?4Fpoj@TL-5p9+5|7p5erp8&fyfYyI7y+3m!p-LVZ%W-l=;l{`+z#qeqK|21@O$Do zlR2k^zx4ctb!C71BgV~*wG6{Y7$%lSV!-&tP2SL<6w1hnQl@MKs$0;BG?S8^=ZRh& zM|VV)g*jkjhM_pSN1(Y6y&6+eNMxd3!3NkHY2OR+@WTL}v$;_}V#YM4%pnLAJmi)t z%2q|TxZf6yo4#?kWZ@R*x=pKgI`W=N54+4a`Q>O%Qw)bzA9gvckD&>SUn_iGqz~HBL=f8c zS*EuVxnfzL9pw}<&20mM+pzKv)1|};IbbgIim$`isQ+mUPp!hWRzg?;#WAq9eLD)) zV>b7Wm+^o2$DxBhddAM%+S}Cg&J`d;oYo937i_LsvbnZ6AG&*roQQC^7(vK+`Sg|Fmio}Oel8NUO)s1VF$FI7v_{gM;U8!4QZpqF0FFPaN+TbA>SHvc&Fxc z*2;hGBuZqgG!8Wk7@z5wqe(JoRj``fyI!1=X%ByJ?Ej#Gf>lS>XwPX%HYVI@hXb9N z$4~wv{Z1)OL*{#qIGmrYTTNwuKTK+!%_wn?wyb~FHtp$v}Op1u!%@k3WZK8KM=qMXnVYnjC^vi~Y>7Xy_^Knw)&cf5&- zjl1)tHdT4CF<}VDTtU1*B2ig?gq`-biM6VZW~*xyb@N+_(;5kw zrcVX)eO3Gu5!%&-wKXoQD31Vq8IiYoBj?L3%aXd4Kk4biN?nNjitmhM)E-CCJ!pQf zhV>tLYJcV(9_rbuF5M-gMms;9U|Zf#mqvfK*h>lY6zrn<$Ht=l$I;&~Gm6}9201`o zKNidcUI)@Gl8bmHI9WWEoI8ztfZ%WdkG>04@wzr*pi?R#Z!@=+l{3N37 zaR>Dn2mV8tM3CZ_f3_3>-MCc_S~JU>RbzYEnj6FG|Cw|~{*zcs4RgvZ0tOc-_V<5o z!IU?tSd~#IQ8>Y|)U4=G>q|ZcjK~-@F8i17X$nCw66rmXXvm6sFrO!g!UO9t@Lq3Sm>2SI-X3wo}qRVN3au|^4YQ6$Z3F^ z`QnDlCj)<0!w#*Z%tAvfCEvBAMOx{oFzNQT3qZuZg8nzgkmORS3GFn{f1_r&B<{Y>{ zSIXYyS`%FNGH)?SAQo|ho9%z4{#cxP=bMo!axoMEjU$ZWf7e9Rc)*RgUT3ugJi{Z7=34} zjZm1Y6O6D}mIGrc#=br;nQ7g?M+b|?u)xQPH`ce#bp?5_*Ai2!;j7@Ev&9Yhw51p= z{}GR|25~`6_+)Yb7I=UCMmK=skTQY{Ap!9XBdIEoy%oToYk$r-=cT!At?5_J4u71H z2T1u?6~V-f+bK3jnG~EN6U?68;t@LKijvg);vOm9JTdMzz9Y%j435#&B82E;ud;pJk z@l?te1#3~<6;53LYKKB4j{pn&hFL(yhbXhM(VNyX_k~zt;6K&D*QwiF6QZrdLrsr} ze|p-IH0XY2<#z6!+k!*01$HUV)dL6&^Nf@nMy6N_SuQfy8X}3U!Uiw+_8~bAZ&Moz zJ))(FU(YV#!o`2m$^UqI99$7v%g>6x#cjWDcT-Oyew=jq?5(9o9Ov10mF@VXGJ)#r zfCLt0I^q-#Vd0nYxPHf_WfPyW%o}z{t>6IZN%6prnDj!%X1eD}2-!<;rF)^FJhRVz zxY`^IA6H<(%q*Kl8vh)1%g3E;ZgjP!Oi@*j`+u*q?bv_Hxk%zFSh4w67Nk+uT;8KJ zi@YAb1f_e`q2}_G%{GU_KFEpT=xRs%CG<-jwWJ38j<#+sW5=Kuo2SwnA;|1~VGp`!E5s*gZirY4i2a2l=~lKzm6+k2*^>1B=`ZWw|a2@)w&)(GQ;k1d~jg!*j4G2=^6bCGuRexxF~ z{S@v;L3kMNP`knpk(Vcsg<_Wjq6AR5>0ZG5}q@Hpw`08@nv6?r@<1lLqM{gJOG2C3`SAk5W6tP!tV775U~P`CR+Q0}&oquOSfv zOw-|xf;L?_U#`;!4^_h{!v76eltrIe-VH4-&x@LE!1nds4P`w}eQ?_PPfl{Lp_z+l+QpgImY+d#Fig!8T6oa|X4vp~96b zo2^%6@}*=03|8JPt%O}MGgJaN!fsI_G=x~UX)X>)&0jbSOskS;%1gIvI1TOtmQJi2 z8t$`qB=@q6nm3v$O@wkbL1pLA@N`d@4S3W>#bOmfefHWPHK4KB7*rkL`lq5w`=x*9 zusk3vb)c_!Q^C<*gq!}F zOxI@2cOB>8DaUf(bPi~D73=imMsk00MGMVX`C>EhXlT-uCkLGHFno;J9Bjf6+-lkN z>{5}0J#i~g#rpah7pn4(kg|@8iAgq##CzIzf;W?d zr0cNceYt@=VB$pX$eI%3T@KMkLiD~E(7w6 z7eow#W0km!Vb?l^^X#&&R)P2gt|vl&mIIC6811>~)1LHUe?NJrx2#%L_f_Z9dyQ>r zbTEEmb@1PqqjB5FTzNl*k8Z~5?m5uZ&PKTV4OR`L2hMt5%_B3y%DhQy)0ycikSwU^ zx{*&BNS91|6BS|k6TAH#0|gPH)-*Mc~!N*-_!$dzUpdWI=KUfm15a^MNNcEqX zuoSbh1ep5ls(0BQuz6I@k*P5$OAq#@}p_ zdroupRIwdz$s8UownJ509>hACOnGP@37yvB8L9#yI;8_o3#u+g+=sgSix3M;P4S;F z)RU9!wi3@VBVTOJFx*+x8FvzSE}6Nflc7{nlZnX=!<4%=TZRSW-|R$XIZgVS|K$^8 zHO|{qIaYwY$ir*Ii2i@dBF=n7{3&r8yQhqdAke?}Dobi9I&<&R$dspKJb^vlw&kzV z_~BLy-`zpB3H9&k#DZWBQB)<2n&<K`ZM{;qU`F5($ z3)REqHf@#+T>(<#dAr}l@*Fi#DH4_qlM%7^{Y&bfPSYWF-cZwIK$dcB2Hu^iuw`fb8NBXy@ejVNgjCwAQu1s7!=-FTbrG(pbDxs*MJzg zm*1}wjyr#?Pk1u$9uwqy)pd1*j@yHR!MaB>^T0*uzA-1ep&CCT3}&OGxJ1lcNhEND zO+Vm)74_SZ{Mkf+5P(qtZjcId4MIBQgvjC!rp)O)AjQdUdDxo?VXp76M0=O_qMki% z!_iWC++D3RG8_K?Vi6%XxqdVWrzSR8# zfx020Mzb+|+W_gN>U)lknJuGCNLK{-OWk{Aw4oLq0rgAdm3N~An`+;H?$GLOH3#IG zg7Cqc7z^1H#*g(^nGfg#(F^VQ*k`QU!~UEziJSm@K(sqLC>b|~C=+Ix5Vg_TsPHI8 zSZ9AZ(BwrX*jH8G(8s_?abUL4Rpmt~&mq`gxNt$B{j~Ayl$cZ?nSIfv&xYP_T4$W;jCJ$S zfkVvk7=VEqQg%-;G{)wh#~+9A?pKW&21UWKK$@#`_-(nG7DtHETm&VLBNdPB?V*1$ z4d2jdxBWHU4ciae;X+aPEAV~CdcFXd<>X6W@=8le$Hs4!Z@#=5 ztIsZSlm~deYB@qPzr1%E2%8IMXRv>2rW*)Q;rq)?vb^~%Xy6Mf!i30HNpPYZbIj5R z0P68*lw;uo5h1=#?Mf?flU0&O7BiAdDUH-Gls5DII* zN>Zox((ArM86o$pK<*79#O)yA`vAawxjx~e?U<)1Po9AzaE{25B{-CajxCY68cWw%KxQr|jBzn3cai(cW+VW?ax=0}z&fe6$v)mlYvWR+zIZLAYbl1C@=J`*d58mhBqUf;==6mfo=O0&oV?2 zp-2cQKJMKRNX7Ra-ziCoY6a^XuW-4f6}SxMT5tEQ%Bn|d9pe3ZIYE8*Wm5O<8aFRN zY5ai{^DH4cAaR5Ffw6yLhW5~~6;AR^C@)Ww#hKpMS-Wwe=}F>wL<0v|>qT03LbExj zX)6L+axnC%t3b2gZ_A1dA>HuLp#UtsAG<}X1DKolPjK(+VR7&Gyp^Egfoq8jwf?lM zoPwpS=Yy={{`UdR8&3Z)9*i&|1m}SwMWY*-(P>OU!|ITWGIM zD@4;1XKsIE#H7COW(nubVA3149@(x|}L? z@vD_aK^4n&SJFo>RbC^6RN>ly;V-R&dmoIgg}C*-S0w2PS_1_(RCi}j*9Yc0gs%;h zD!VRqS-!Lz%q=lI36X0H!fEcWX$X7PzlvG5$oqeHTm1K77c*!XNLsDjbJ#0V@wM>f z4?whBJ1;9Qf#P43^r+a1i+0agj)&d7DRt>IT>CIV1LLi8(Q)y8=&$B45A(eP*VToZ zo8-YWaMUz&3<*ldNawHnW(%G$nxesnW)pw9j$VWiOD^_-&?ExN?92&jtZx69EH0zX zh|GU)_wsKH!4+HdEeG)DK7p;HxEsp3M!mfh3fw$K_<|`4LC{~Vx?}V!04L{U(tf3M z>fVnrVUF`2>?ldExK#{Jk5OGAaQ{pOIdOpRf_}f*MA?8H4|k3QEerFsah#;t>KxD8 z*9^IKMV-#zEU~H4ejkvgreTzxg_Y-D&|iO}(|$VqK+giix80n=n5(cQq7jB)wi%RB z&C{=r{hOq`Ka4a}rQ&U`~5AGoO)@85?5_Y7@*F-8-;+>jQIR4p< zb8>ev_Pl2d4EhT^8qSALC}A3$Wtve$2Vm`DDbXlwF)K56LZuyx6CLsK?CJM<#T#g} zj$#Y76SR4V*%b=Z)&Iw`RhcVzT#$dn285(4_hW6f-w4p&@fU+~OkDlcwcharSuc&v zF(gnWz3|fRRJ+>9Jw)rnTaw6VyY_kDH|xdvFxRfZ{||ep21gHNtTcaIldI-1 z$f<(xBJerG2Pk2=k_8$?j2M!%6>Q!Vtb@Dj*8zIH1hc(jTOxa*+<4;1ZC z2CrTw+rLWmv{8IbHH`kr6CHmT@r?e*HEcyU-@%GaGzZ^75=ILYXI{CJ;j9e*_HYoZ z|DWYKQbpKv3XM#!X{k4Udq%y9iz`ek;XzX2Va8gd+-0qpRNv9L%S}H6mh6k;`_a0T z3m(K_qdIEK_4fSG2t>F&K53A!irSzR9cDf4M7U+M*uxHkGaOWrD zwwpV1wn74Qu*{kDZLh;tFKc-a;LD!z$Si^1Qka6x7$?|6n$68E`MzoB4l8Pe6Gf61 zBx7@sq5FoGN9Vok2s&qMK*dc*ZF@`v3dRJ4$%$eV5FSMWinxcn0+yqPQSEt9?7tH_ z8hyu4a)gT)YRrq*Ok#g?4oGp{X#Z)$uW##?(g4wLk<1Q(@w~s5q>U5`6YD_y1f`kT zPZRkcpbEQ2i8$5IgkQ^Vp1zCozi^x$jsRRiS=Vk{LNkc|K>OEeQjn9pfS8~363l}0 zu)-jEqOI~M=Yl@E*lKsPTWGf?=JuW)Pb>D02V1SOE?u(V6R&^foQrr+UPo>yFZ5Z{ zh@LtV8ucVja%;qP{Ni}>gi&~O*A!8tgIHh~5(Ym_(Y3vxR{UU7mT7rWaxWmszM68PAa@|1g z>Kg&To56}Mpf7*D&R3ve-ea$dwL4}VE<-bfl1I`qhX&AI5!ZDu3db>Nh9o#E%*U%X z4M%5X-j{D=QI>Dr5Y?GTkq(C3{QevHuN>R7^e0HeZ7JQWgzm0zE1cqFV877-prh$T6mYkR zO5Hr#=#KC{dGS<=cA{MMjo=zbxpiNUVLZXz1)pSk4!iemhcA)V6%C$qjqA) zFpg}Q16Y4WqQLb$l5_P@GH%?YbIq|~K>+dDS!kQ#bDbn&GjUXt;9mkzy)(R&)D~}s zg#=QxR>amGD8bs<%z-tP>m_zok^fgEUHU1IX^0>%hY#h6=43sRx6pA;Cm15N;T+#C zekdq*9|Jetz-112jN<1p`I(z=`Qw|Z{7=7vL-Bux5EATAn7m-OEKwgZWX3q%F-$2J z6r(B8+Qn{QS!xh&0vF0&)g~P#+O!!#=;vdSt|9it6p4C6TZhuEyR9+hR>`=R6@j!Z zck^vcW%3A!?5dM^0RC+jXau$q8qc8Uy`w5c-{LsVR!Z^Sbi?N-n`~gYck|>M59ZmN z@xOok;G14%Z10YZcgae8{iOz*l&8GDis}gVWdtu%|FzkBQ_#XlnYzfas*rhUt}UC1 zc3~UKt`X0jf@eEXD zRd1_kb*M(B{U6x2(0Pwgvi7lh0Er91IW>QpE4)^eF(xTavq|&a6?sehlHr)b`v83_ z$MZ}~%6PLidkWxH`(Cy4Wwh&atV~L4867p$5VXk6jQ!em6Uz zibqT!R8V4N)fgCNW|UAv&UsipD#RX(T1w`Zh}InKyhtJ_UC6bgzoTD7k7r}AAvk}R zin>pe^4+d>WCkVX;^Q=9Wtg~-;c2W8Rn+*m%x1;JG^khturpy^QS|uRjn*OORc^X< z?^~$PHv}XbA%EgrmNvgU-UO%!I?N?q8> zWz9IpU!>Z|I?JH{(3bvG2vO4S7@B`71Ip;Ro7cJP#3j3LbjRV);m=s^VDT!P+r3Lw zN6YBkOFpQf#@Y;tfg64FXD1ZaCWB7KOMOIutNpqkp>>_9d?K{8E5Yd=IeEm2zju$TFx8{a+)W8Qy_l^55lg;H3cC1 z1NO#RmJt8^6%E1Q2r_T|Nj#7sn4^he3zGHj=l3gbmNg!Vrl<5MOLMJJ%}*R`MVcE; zt)ZX7q8?~0iI3j{-+4PAN^1v8{h4lsDHe8z{Lt`sNx}11bjdxNnVl-MRE(^o9Bdk0 zr#Fv=8&E-R z6cM7{{MNjiUX?$fr&<}lrmPKZ18V!Ph;le(2M7sOv?Q=N-pkYuL;C!V^& z&q-e1(%LHWysKoWKyYu;+`2f{N)vn6uu%7=-z=pu<^-lfFAiAO50~)H_IBH#bF!NP za+;VqHg^m#NFPYVg8AcpnM^xgk4dFIWl1lcBzmOJU%^ofmKt`@q}LHO?&DYckAc&h zqv&MTZ;RlCl>~pqi}$fw)-(?EDr(8MT%0kEi zCupaIIDo>~P@eiuOBx!ro7>}WTRE1>%>2FdGaZSd7KV}BqMhj?Y0Nh^xKgIO%zjZu zbwP)?$-Jw-T;&qPo2U>bf6iDNFfANmc`;)>15()bHmDVQKZJR~;hN$3B?XlVih|OcSE(;=3;8uQ%fD8( zHdaziC7$IDN9bfe5HR*VKe+~Qoi`3H{NX(0e}8`TU20HCvq@4WOge6fz6)+`e}VIL zCb%gZXLElhmBz*E$x>pv%Lxg|jxed|!?}eE5G-HQ&ba`E;B|TBm(Hk(A~2D+!9Gc7 zyy=GiZn2DCw+o;gbfmz4m`fv@gPzP99vgWb9bs zT}l89J^l2+y#a}4lm`U$UklUqw8MtfE}H{&!g7C7Y9l#jO7^Z_2$Ymu;BB~~P)aJ1 z2%atrbQPezrmd?DHfoGst5h_13acrBw2G^gG4%CaW7{&IU(=t8f58+*C*s(eo@;a6 zmWaga3A5%`la@#^8rMN`1ZX<0=Vb{Arvf?v|!;pO(0$&?|86{=Ca6pT`=JH7_W`YNJ&$K27|`#fx(Z zR&TrpzLhQ6P>&j?JPK<5{l_|NoTo#7;NyQxxkB*5|M}0LIMu%5t!c@uVt5F3>)*oB z05FDR`KiWU@~yPJTIh(2Wok5{rbgpF2Rkhuh09Nh!7*#KJzZDy^2WvJf=)4p^J+u=hC^y1eX3KA-K&ch`zC4D8h%(d!xF9?7A z6Ce%RR}eY5;K1F#>dsDFWwP7`l|J_o3uZeYOzJHffKUg)x#&!{AX`*|0O=;Lsup_ zG-7*qTPczBrxEVYlS!xAZq$&hG#P(9~A1E)0M0EA>T! ztDM+ZOzf#NPg;z#Qf)eE*AI@6u|Uc_+QWXnZysUfr$fS6Fm8M_P@|;IFF+EeYU;$E z2APuL-Z2SJ?3uIQ`J+(g``Re^U%|xae2`!f!aBZt>ZPK=1+&B!Hh+(bMG#myC6 z=vR5UxT-oD!>I9|7M+HOYl45l;;3n>DuG(q!`aYDT1vI3$`3`0#Wzaa(}!g&NmivO zzbPts)8#tHX;~n#&#U7qE$?P)gz9M~RSGl59ens!_xCaP4k&t2QQ#2nN1vCLG9?8p zL``5kb)2)LeeCr;t`Q+Xd@rBS# zmG$oGs;=rY=BFe1!o48zIuzeTCXRXnlQ;anM@X>5_hVn*im#%Sz5(k%qa6 z#V?ydatoY!MufL(+#ZEfkmrD>7p0<~)2^v20f;0O!THIBz>)YEE`gd8_s~J-PIyUv zA7zYPUovqts6hYB<3>S&>>H&mIjFfJC>G~ZO4cR3%!sW?tmGiuvk}!SxOX_sO%DB82kg@uo^P27NRLiksM#E&gesF-d5}e*>(2lL zK*>FRA;{i*P;RGoOdL;wC{F_+085UC5uz9@wkROlU(bmO+{{5GagA*ZhgTgcaof5Hlu2?SAO&k z90HXDBc!+Xb+3=Xr-{Q=2puNU0c{U2fku@$5!(a*pt@bv$k6TiUmRBHu5NzK$8Jw7 z#z%C$+UfpU3T~f=EI1Wd zjH{cv?$(o@M~U3MWXCs&|KldK;24^7t%d%xwWTbZr4|Kc|!G+-c?wC`K+H+-q1mX=cZ`gS3O#ni)=4Al{{p4gcU=%>sKVeMT+2v8zh)Xc!9QQ@*cP*L;ydlF}WI}csbm0(@i z2>t*?E|%asZh%L!{KmR8p=6t9{>*`TQP(VkKLe4QWI5qCX4+xVS7ZRg-jm&ssKEQ? zF6kC^8AH;Sc%mHGw* z?uFxi8BPs|V${_Ntf&Gn$#5}r_CC7bNzV&WAoG4u6=6%y&KSI)-_ckc3^c0^^#P1# zk`MV0qu%b6vE(JQtw4@uh^_V}%#0v9PxShP6)wKIEMqSXAm$|b(ARCKea!aCfgIe* z3$iPLx4p5sRi>lwEE>O`8@>}dGV${9c3~LA?Ah~1Fi`g-i)#iN7`Qve6ZiKEQ3&+~ z4qh7YJv7R)>CA7mZQCkS1{X6hom2H~C~ossSC2zcd46DSq^{30=?-d_N!6zttbh1& zHhN6qe64PTtmdgHML)Cl?Flne1CD=;cU_BoK(znxbgBkh8D`kK%X93yO}Hf8MolNA zH%Kh~I-niVDOrTkEzoP5COmmZC_QUIREFsPVRn#Ts424xURWEpZEa168Ba(+;D5}) z@`AVppxl8hzz?Iv_e8=3{>Rnh8!oCEtt<3smfYs%-Q5dVh_WZ+E~$sh&L1*6A?ek* zP%V$9GOptGzcz-}Pf!?* z71LDkrWkKxxV%l#0XbJ1lbR_HGha@WFd}(%dNhGL(AVDRWwb3mEKEW)>`g+oeHtXM zbS^|vxt9(nisZy2&C6;Liyp4Tlt*s>+th^=p&XjHYEeJnbRy+|#ZSj$PZU}pg{(k9 zMpyYuoNz6@fiZ@YnU*Sl9v?K zcC9HGsI-GOAh0t?!@MxJU?dBQ=;u>5j#aiEF~Gb)VoMuTW~o=mM+hVN+4DBAN`6dP zh6c|FJin&5`6TF z4q%)NF*1||UVEJ$jSc%1sSZ(^Vs@|88~hK>8Xfo7z7O2@uM^Nczu(;B0yTK-#KWd0 znr{c9Cw8@udqOg;dyX+QJfYLIT2k%N6<3V}kRoUWL^tO51si*rdD~w9=dyOEjqaO* zVLKl2mLpt;8hUK+sob%Hy)=G`9KTFSaUlJAhhInzTRcFBd~E*lK8A*E1osQ^{!Qkd zHAHJ{UUApUZ|G+6ZZWH}!sq_6_2m|+2QD;j=h90;mf-U@uceK_OQ+g2oPEO)l`ngx z5|mfCFIVOigHR})<)dGc8HU}U*Ikxq^h9Pbn%>~JRQlr%e>et*`86YJtMK)076GM& z*V)$DACn1-p%s^%1du#v1t^&x_GSj(VWqf|K*biqIIpDlh0UE%_l@p|XNW4gVT(EJ zWe}yIznAuCsFzl>u7>Fjk>n`h_h>J5Es1)NIP4y{C;#qGp{|`XE1%P>Mv0v3X&rW8 ztQ^PY-BK$m+<$q`JBhdW@DjVA6JWspy60YGp>l!Bw?ubPUoF-DrdH^+j@GbCEliDzA+B%GpB~*08SRPyMCpdex)tg-_$$*)>WSJlH!8)G+$TplKFsaqh zbKg6nKdATuQ9ojIWcWMU!7koIU zQF^_@mg;aZHR-BalnQqE#OQQ3TC%Jde_$qMRLnEGcRyoOslpmIL^J3jSP?R*^T(zC zErdQjzT_3hkodygNQpWrr@!gJ9aM315)rh>ru5l+um>lUgrd$1_=qG&lVzTNZb1CH zDQdu1?NCVzRb?Ytnz0>|Y%4*8Y0lJ6C8I9fhJ+YjGMX^!?axFeZSqgw-}s;ejBZSGEQbv556d5Hf;P>HBrhq9HlWqTK}usetYTQt!3fV zw+pEk;Ni=+0$T6%E`~>6tIdc9$%wW)3;_0W4B}qjKWbKvP7r# zb{Q+gOziqmo{bEZR}AjXi%AMLewaz4JPfAm%XwzzCMkx;6yEDLOKZoT?)2^r8?Vu*@E!!c{0#k)>tMj(eU)n<|x zj6GD!KN}#RXXe~cQ$~;Lo(|2%7MhR7i zjL$7$R$kbyHRhoI4&}~p&LN@tMuW!3ahC{wLzj<^K`wwH31(4IzFZ5-Igj)9n_r$8 zSNG2GO`|LX{Xy#FLA8vkBe_)7iGS{uS!=Z3`eq-px&;<}dnfL0v}%1SKmax@4&>>t z*ZcXyZPQw_D(aBu9H_)r5=2r$D z<1H_0Fn7U#L7l1*W6#R*~aIQOGh>GVZ}@`gAUeoxnL}T*CMg@3yCvLimfljt87E|PJATNw;iX-pNM^4 zbP9Vt|LNz7kBiGu35wHvwj5AR5qh0E(7Ms^{j+*4Q7xt$!I9PulRv_3A9>!e|Wl|zl$P{ zw5~CJUqIr8pV!s?e05hA)ra3` z3cwXUl~n>BChE42r-sG-^P2~0xo1=Tb~p=@s`{I(ghUk@pJTjxpj1LgD2=cP}SOU`MR z>-`1LJ_gXi;rZkDC{HDhkI|`m{k7;{v#Biy|3DD?asnmm-C4fvpc%o8nFh-S{xCN8 zbH49l*pej2#hW@sj-kxSwVF%o^a)5hDjrJ0w^kOw>Z1b>SHcz=vDebB*kynUgBYS_ zs0c)8ib=cWv}#Q=PE?-X6%Ng>Sn4xm-R-j?y>fAYEHngU$x;YY)Amvaky5a3- zgES$p@dfJ%2E39=jihpLaM7tZaXc|odW=gpV_=pcGveJ;lLtY!u>luQjP|p>IGx9( z10%iGf!gJcK#OBjJ)55TPy{FfeKCRRNH}v5;<>*QQLp|L>3;xss=gA{gd^4qia~dxUB;R99SkO}8JmJ?iB%wU zMo!iD&)CpL?b~7{;|IP*a3Y&(?K5+dr9FY3gGJ&TpVFW+bT`$6_s)4XxUpoqx}w5S zp^Gesj)t-O6J%1hggRtPmj9@EZw=bUD*b@0C)@W7eMd@&z{j7eM1yd_Cdkdh$IC0o z4&r78ftXn_xYS&&(sATkRh3v>KIwSxDZJ&~X} zJMNpbYi;vX2&lnrd(0>xQ7OVDng8vtlIiu^XR&tps z?5AcCy1f9EiB}mIvTJCum+8oNktD#?zm7ZKj=|CM4%0;pd01(Kb6&4#wBIU$AEhy9 z(1l`8cXcqiRZvfTWpVWB9hg|mc%5LPA}E3jF^9N zth@`g#^o#GbqHU?A}1PgfN(DoeZEIKa(ucWr|PFb@8o|PF);dC^|mjfG$P(y>syow zEJ#GF!6DrQ_hKqmal-5-A?4KR292%27L!Ie$cf`A3#53OWc_h^gG9oTbzQ&DtSWU( zvI}UxoqZ1QSzdnFIJ_RC;d1SKI2pZ9{lWF_88)JYnIty4a=Jd99ksLZIeOJ9q z3Z5M(x*HVD1JgCQUAMFZ<1baE_~=x^Ut1QfXd|dRdj*LHsMR2*vE`I`E&SI zO6O?q?iXotH+IH4?@kVBo^H>sc2Y~`5DK%%ir5|o`_^j@d&^eILG_n{V(qP=pn^45ZfKr=;M+ppR$vDxf zpnnbG0DmLv;MAk>qdC-rlTMj9<4pknp|B=z!2!iI2XYj27=!L==h{yE>W#bEY0Jk) z*kFzrae}TL80JLYAk<@RCOq^$129E3OvVr4!0E!N6?za+Fe8O~M04rHa)Cwf!YLUzh^^lVRnbQG zy+byniepz;AsLLO>8a!#>%yp|aGt@a6{6Bl=$~i`=7{TriC$EaJ+j@+Ff>ntA<+`C zHeZ4`a0Z=+kY$#nNFp{+VA6syom_FLR<{jch@%h_Yy!pk!@|;!_+`e~jQM(yM;e-$ zffIMzA)1TIk&jF~ADsyv7dj*&VAXC(QHQCUx+kRI-QX%Xs2WxXt`K8XeJX#^=5>w5 z9^!}voGO-IGPl{H!6>!;1BbrWqvhQxbYuLKmuN>kqVOMxqIemrwOD@IBP}bugfvCA z|J9WMWV|I-M5GTRa6Q>PQV_*Yr0e6Q*qe>vr)|Bu_h68ZYdKQ7S(d)d{yfB@uHpU!zjDw+S^`Bn2Lz0tK1=dd6 zQny!p9U43PSgvTK3W&8VWVXA`mxI0^ZX|$znu?D$I+&XlS}}sxp*$fh3n_PFk6FG) zSPtMR_>J)5qD2cdHojwo&VA9Mr+`(;5}l zUsz4xut4qNd1IPx5}8sY=pk4WjzrPCNKKj?Kk=4QNKUSXqHWmSk*Pssk(=MlX^{zk zQ0Y{E!WbG~RIs3;Y)wLLMO;4n6Cxt01y8=e^cvZ^)U7~HNcJoP&wR+45CN;jxps~! z07=jcMX=Ou6bmwBgMwD|nhdaxG_|t88zZJb+H8E4c(!(H*A$5A;i8i_qOqd@PQ&tW zQI4eZj(vXu%V7j|-%a2u6d~oK=;0!QdI%mY_Ae2vzY;_$gkjT0PbR};8qr)X(OjI# zm-2r8u)CJY&cwGYb+clEn|7q~&u@yg!n$ajw|s11Xohvb5HUeO4a1LJxX{~6w50zN?xG;wFa3z&zmTVYa|;602WQx+Bd(q)+uDLOMu zRzzxg=X@-8Nh5bd+*!J}1RJtKqyrJ908D=ODZWz)~A(g{BT4n6)M`uq&r6( zU9I6#NoKcph&33N6eQzH3M+Mo$=Sy5az*jCsh5y2F|PkR%Gm_2MxDK1brUeZXnD2z zR~~;KhSl(_J%LAK=p0rG=SU#KWcTnh7G8nb=9bT|J`Y3OK>$gkEKKuMn%e1%mFn#o zIhE=px6txSiGg~qmgxYvE%!i6BAqQHAc;9hTsf+nx=7BvO57CO$i-t^$(^wDlO2l{ zJA3f8-7b=xi0lrHDPE3Z*k<9*VC0pDuHGnyH`bQ=kmVgY{d=+z|AZbiYRGG>(4T_V zjEOJKb*}n#^Z-Zn`RMUQ8AdJ{TgvyO`GvOa%;GB>KRCCfUdzk9fa-dB@3DJ)te4-_ zkei`*FQv1`7nw?^L!-7ILJEDuS4c}A{~`C-wyRo)X_9#DYY4Tnn}%+QZ;h-i%m z%pPVk_9x4s&kl2*TT^{8=t}H3~`A!JA-$AO8s~> zXZp%&t-cQV7g24@IJ51j(CzWX+L4EA1DrtI;QJ#875b)o`yX{%%wp5ArI^`~o%_?X z1)9FIl7eX~gN<{1irX8XPz4Ez7xq4lF|&?HR_y?nXq}kDl(0AwO0{n}a?R;Th(erY z!KW$B6p3?wjDQ{AB%A~CyHcg5LBF1z<$Z1rYTW@|Q4uy*8Xx@zhX!Kb<;BQjBp9qVGrP8a!>aNnY^49lnR)=JuLs_eUjADI9ef= zWC+vETq1aM75v^QJ8x<6$8gBiMacD;)Ap|PQ{WFaK9I$zknR8AVB4~lM2Szr8} zbyq=s^J_qY`0JSVaLL`JJm-f^R@;8h17C#fvl&~jGsBu|Zv(F<;&~6!V4D@*H}?RR zodi>!`qzzX^n2!sf|U=#7H^aew1@6u7$pu)1I$9Z7H{H7hAZ0dIhyW&5lcwAocEJD z&-Q-}6hlnOe4r>)r+M1HwW>cYf7;^8>GHcjyNbYFF{{dzpfOW2!?kJhjy8L~T5H=8 ze15ulxa8uJZ1;^nF6?^RyM4^<^!LWPUVF4@s(Qx#y~-u<&wNWD^~Wm&I)G2m1cOW4 z-Um!8zz3T*>C+l>)AG`C!xFVzUEN`ce6%21EetLt7aLbtjQ?LTIa*^eAvpmBc?D^Z z3@@L6w7ig@0JjW}upF!73^Z`Zb!?_$IJKkVh_`dxN5R3 zQTM4^6&gJn0zF%vUuhow8C@_fOCACS8wOecu=Fd9&t7CcpL$`tFm`Xe$UdXkBx9zK znyOVuh|j*&9VDzqIF-it?s#1ZTC4=UdQwj{TOREgD_A<;@ixQ%1Ymv+K>6A0CvDwP z8q-cxMO3C!C}2inPMzC^4m7O_NV11qYTyvpz_VE}y_$#naj9$4yGs#`=Nn) zISyxR;7eWAgB>t~b~s+B_2}Z#fZf=KV;4?h7nWUvxsZn{qO-p;(vV$4dDU&Y8|*TN zdcOuo6%@jzcD%D-Dum=ej*Ix9ehlG9i?Bc|Ha94`guWF;P_!T#09MIj`jDt12tnw! z7Dh!IrY;bIH$C=gL568Ctuf}~Z3{R}f6pg}i*%JLD+03yjdYEGLpsh=zNm>e)g)Zk zAcP%4T-D<-bh;F(MQNGdO`RO1-vW6I;KEcZUJJsPI}t+RLXpe zvLL^hzX4$0o^i;?1!ap9CSL4BE^7%n=1gN99Q{IAe~5+$Q^P`Bfj946rJGzW`j=oi{FV+6;m?p|01s@ zFEvf$){bR15mvhYqJpiGxCbi16w5ukTL&?-_wj6c()>>U?#<5QrDwa+ zE;Ym%=9GX_1ob%9t8olb3sS03&*bBrZ4~Np^fn9JKVh7^;r=x=I8wN>!)6d7AKe!~ zrZIJ}#4hTQV!yNymMjD*+yW`C*PcxK_DS>TJ=TB)LR=quS}1E;I23}x*&8zg?OZY^ z*$zaFp~ofb_cjQ0gS_DErr%89`XbixiISs-rvm5(8-(#UBj^VyhFD54>OgcwbOXGy zC{IGQ+Oa(BjdP^izYGhv{Bn!dtl7Q*u zwf~0Kj7HOYB8xI1iZCl4{_mrIN&E}y8`k8N=MfXTAGWX(bBh=X{e!IM0%ZM6i;~On zeaIyD+1O*dk)Crvnm&73AFwkGp1$we2BypQU5_`XDP&oHe7JC2D?<>FVsZ*)D-BoE zKy0WX-=|evgfq^=ZfUSZDawTzIE1dFkq2YuNiLYA2D;J>99B7ze{88M zn<5`|EX@!Z<0y?3`wdCz2`m`&)A>3z#{Yx zJt>hSI@L#c{~Wg53~vq#tA@Y{Ldwl4RUq$XLR8}{%c=IYk=5;Bh?ItQhl;6H;XJ=# zDfHO;(tLBf;~nuNb0R)N+Yhx*I)WM-@D?f>sm~(*o`MEH z-iwYdT6i6P%x3hnkpiPB z#2MBE7-lU0*q={ire;*Aq8Nwj%swvADKsh?B}(3&yRqg((j8rFR8n!?wqUg1`rXa| zCe6mA=d)j_)>?KQ^{%Mi-f-rq+)hjtcBu+mHKbs#sBzY%bm@Obo|(P2l4n+!I^vy3N?*dyO-jUBg( zAC~dOvPA4-v6oz}`LWKs7^a>RbV@iT=3Mqx)%7fnpVyuu`N-v13SYb|c(3mVos>iZ zW9g5ZFp`X=x~NtDYrz!kG25h1kX2hAQS;bavp7p)H2im9Hp+83mF-BHU#M zYIiDZD76nKm5kj9eTBf*%aR%_e7JtpFI1mety@+KPbbzeM(^U7)scSa*z?Q%x@ylk zm++Ko4?`AGGrki7am&sLXO6|^$y(540Pjd4Cq0=Pj^Pdx~RmAM5;ybD znXd7iP-S4SGRZr5160z3#-pXgZzEtgkWgoIVbg1O;Qc9O~f0*Vh7)ID#xBNqbR zq>+okeCQD)qFtdQG>iC*2ue;^+Y@_4+@cySA~`YP8zhg)+zzany{U_C*{o9=}Cx%tjO?TaRcLV;^lTQ~b@{S0$4ANts6ho3Z~^%K8% zJwXtMJwo^`rXItazdZ@FI*BJRUKR2fU=&kDCkzSHvH{mkg10) zQhSZIboz;nmBE}rv@&%`DsX|n!=9SO^U><{tvsw&BPn4&zhrB|zkNH0OL6kvYTm;{ zGh#pZ)*x-1-5ok#ajOVV@?`EjxzSJidT7<@!XfuIeDN};^#lzo)xLKSS4l=$E}{bL ziwPOv&GHT4p|-Cpt_-lY5MF202>gk#Wqb%K@5|lM8E~)}L#g7zrE8OYM-(n#XXksL zk^hQgSX#1LvL64Hw6NqE!hiAcnWi|V*Q}1VG(_}mf#lT-Z6;FdHabv7lqIw%GOUKVZZnFug@ZE)k7>FmqeclKA2eS7R6$+&-XTzGJK?A{YR7cb>9In0Yl@`4>IT`}SLP6%3Grl=4Jgq|xNQcdn6}fVye* zjD>r*-Vcwk@Hm%l;AJed&3|EoOJ-0nv^TG)|Lbc#_=1ZWqap@f34j1fVmHitANWTzYzRSIGXR^6wa>=BS)Bq!&Uen)gfILY?c z7Vyh&q&#Zsz0zVF3x69Zh{gyYwr@`(xct3zbJX z$bfgG>>$w2DF#B;8-uH!3 zBi(2kd_$P`&+@|(yN9bz2GciLBWZPMN~dr6Y3ask!_+8Pl9?MMxFAEJ=mm9q^2Vnp zWKdOJye-$(lTrtRkJ9KS#hw0ml=2h4Jf_*8gtZuTtj@ci5ZbpRixOd7Ztpnn9oWNQ zt*_k7%yP`A>Rv4@P0jQINk zFp_th^Uc(}nRKkvbdas+)TZdvS{iNtsd7K$+TZ95hO%-&&fOvl zVQ~OhY>(WO)X8im6#D#o=y}R%{I5k#7B8*&mw1_45@)_Z>>E`B*%Q-t+kXV6_3-c< zdh%inn_5dxUIyE=`D(KVPBrH3WZ*n8l;Q7?=7yfz#t|3hO}agU*}RA`2E|Tt_!5GQ za^zjF@VN#fdX=SD}Y;<8Vm&2R~@5xPsZc}!UL_^ZHs zcOtdu;l4XL>HrtNn$~$bLX0vsX2ofki|C{64-0BOd=Vx%U0W9Vq(kv z>)cxPRKB^F_sCzkQ_|L+<6>2Gw9S(JGGUC^Hp_Tke_sZlS42|X`4gvN|Emi$wbRb# zbR)NFM+91~VYJN$g(HOTN%mcuHzDizJH53E6NI65OG7RbI z*NCHY%A{&0UwYqN7B78i&+I<1%;9&jLKX`+;gT^?V0xKl&>^gHWe&0oOG2A$ycx%> zS+u1@7G+>n+I0MiLz_(x4yNpa1cAx4>ae&qVDk7jI&*)tzDkn_P)tgtv4zt0HfkoL zx;uD@^8o)HVgcTim?*m8rKiqcbgIt0a)7jok4@SzSlzr}@)D)+!wc}6HH5V&e;se8 z9F&D>zLX&MOP8N7RD9#yA*7u~#M|vcuqbZfU(~`0bSYNnIkglwdQR0ya`Am&&NBj7 zc#j*O>d8-%PrHfQ3AQoaAnK!oFYtI#0V_T3$3Cmlx5L@TG|X0!+T(f7fTb zzh@4{*08KBEX2&jf6w^%V4391>@8d^iFtTUW>fH#a&`A^ni zo5c#Cm{yz!357L!H+9*;?((Z46w3v>*hk-(XjC?q@V=>U4VIEmO7&ap?;!_r2>o^s z+I~-maes$43`m4Xf~777i=W>;X9JVH|4k;8un&vhX@%ivl0kPoM3JHR_SpjjCP7np zbU>DI&6F#A8?Yec-d3Vb+bx1KS$boQ@#nY4Oy;_Pf#Mm5L@}%fqYWpg657S5o%~@8 zbIvMGN#kz#MgZLg|4yoSkJGqWuB30p(jjQpVHCYp7lB( zp)O>$zGxgx}uf9c(;3s3=-&-ALY5+j_NS_T<-JR7&9oJty z8s!%Y3#5NqknP9FwXk0-WDO-wMkURnLXo?i5ofU#%p%@_cc%0S+P#IaYSXuEVz`sg z0I*hXl8+@z7sdUe8iT*xr-O8yK`Up8As^JnIN|m}5b0s}-xa~ku8AM)(x4W9qvAFO z{C}wnF6Bx-xQ*u4ajl9TX|3p69GvlE^x3qv_AX;2sF6@#6{khRf>nH#CRX)zHG*G_ zhmv+FCqEZ%m?_cp0y~+_u{siLq1ldo0^0NZ|G-z}HfI~#x%wgC79=<)LQF95IV)2M z%l?@O{V*FBR~f>YN)82=2;t1~$z065Mr%0gWtH80wONzSgLGv%-$F7Z0GcY-3?^Br zyy=``P!emX?$y&hL^LPA1MAY;eK$0S_gAXZy6vOV67+QggbxqHs6HRykmo)O97Co_ zwTnv$N4jw3mo&4`2>7CZt0Rgs?}rId5l)w> z!omXs;hwJT`{(qoQt12ARb)EicjDJ2!QWq{HFLmKx_~K*<#9l#aS$I`b){3=Jf$)J_1)|@2)9>ReYlc`6vsHlOaHNP6H? ziROjV7#>XZfn7b9Oc&sWcc@!HuFxsTfg7D58JQ z#La7G6*;l;I|UK1Ht6qNA?n43i2?|OuVcPOKui$!dg`;=>F$0>Ik|Fkk!{oDthw&N zKcG+jAwyA8gANN_M?Ou%`I&3zmPvAgn^SZio z;ffi`Ya19HcrGAMUDe;>k?sAO(OTu!u5Yh=ul2I9?(ot_U{mQZdO@e5Rx6O0E)oDx zTH&MS7*os*6?{w@-p8c5P-3b!%ggzvT>A(Sw@nzg(s}6i?ehi*HG0di5n=MsTc@Wp z^4*ZKkoUntBh7|?bxWTU26itP%a4^DK|Q)I$F}5o4@D9PZi4&G2~x6tLl)xNWC)n2 z7}J@#vfOWS*`0k&x=uFB!3#>2r`^$I8OKnBP5;luvBKfP1IH{sheUH%nXuV3aJk`O z8GxTm*Y@{c)P_a9KaDbgop-g5W67r+esHaTk)d{5N2@nTJ>x|w#CTEK~(xxa4LHv-a2g8pXWQMLS<;g4ly zppR(Zrf)>BNKx|CJkJv$7eQMP?w`)$ zr;FoyLxx6+V=Yhr`%s4`$xGD{m*$s6eXxIKwQTS;W#=orIM!m9$iwTZ!rEF3>o1Wn z(QpP~SsWU`Xa7wUz-?bu_wW3|o&vV&WBlA6Fv?3ZXrjzjj7gr~A&r^gd(J9<5nu zL2B}}ypmBE(UPNWIl(yC(l6=3Xn>3{dmS$1u=@j=2MF|oAvi^6fw@0ZjEC#Zb;Mb`tXyY0RHAdYs#{fK{?EUeH zCGIZ%P^gg-=-GFr7iDnkbNa3JM4wm>UZ^NTV*3K4-;9WS8rK|urhJ10xv;Cv;o7s^ z9(Xe>9V2Kdn-J@^}x$zfT+-ZaNs16`&o9D6{I+46#Sm(XK+h9<|sCHTIwZTj8(RYIw+}}83 z&>QkN=GjTY->jV#e+x{#DULOCy)6e;)N<&HOK2ETqt&gqOLIHVv#V3Pg0tY>*`Rx8 zce?@-rvLZVH`kV~rCPKz0ZsL?>=V1sJknLFuA&{ohW592=75V=%SNfolBY|S`o*Hh z>P@T0#rsQ>`wmczMep6EXW-y4<5{U_pnkE?nmxspezK~kv+8$q{d6CXC1`s->(vXz zNzcaVtugB7`@#|o)P=;S)w8tk*lrlHX zYwqLJR|sXERbkslTgho-fTmV#-i$pq;XGWAWh@)6=ihEz=f|=nhzfpAIyGb&L}*Mm8kQ{{!I6XLUldu#J>HmtX>8V1joL`Z2RK)eq##(8lMOt9QWU*$LH#lr zeAP*9ZYX2bWsO%|PM1ekT1LBx9)n*kHA8`89AuBV(=HJu;F{j*9~Za?ekv&NW2O0#lAO`CDDR7CTZ zq`^2;G7!jC8LAoD^l~Bxgs3JbwM6$?$VDK*|4T9oK)g zjtrhRjFPefoCd^oTwAF+P}!%i5;1a11$;90S-wvM(lc7CmT~=5xl$XE!<(1qVoKrA z-aaCw2ZB`Sd4Ht}k*M@GHH?mS-u5RUB{;MT zO?|p4?-pQ~{Q#~)PyL5jQHiSc!NY~$_PCSf4j=9PL=t3+wAM|p7E5m}uVkshp|uIb z(Ku`n*MP*p(D!ZgZPgIfkd>Bls;!xKuiwqV!jkY2Mz=%v?A_kkSfkINRfki@C^`yb zvC29AVzN3h>E6%WVg4&OQnm1*O*1Hp^YYB&+e%w=Js|4P1Bb`c`Em>|^Tk-k6me9A zwmM}*B-4bf%}Mj!GbLxI0UgHdm438`k3;#V@qGWX#^3-we5O&1$SS-(bu?S6d#s+1 z>m#hQ>@9dc zCmylX_yAJHbdrj$ERNo+6Prm4?b2yH8;J;an#^8EL^Zj2W-I@2n8yy}lVo@VJx%HN z91*dk15s1uOA!ZqzT|n=;i=~5T{7`i$*$$%Y(Juc)+vL71-ARKjkQNNBoa@FMF$P5 zSBC_6-A0@Ryt5~xC$+Nhha($CF`EvaaTQ0o2w^6-Bp~p zLSxMzTc(caB|q4sw@(-#un?_?#myVc{g4C>n8Tw;I`1y?(LULU19T5Xr|?;lgF%(*86EROp6} z%M0u$`PoQYCB7Xbygp%*==`5vNKoAxFqyw29-q=1)(ma&N;sWlzSQ2HDT-W~d#rL0 ziOQyi-L;;MyRLjj&{8^{ns8rzp!5VXq{H!@TMPNZ34>!stG3FpM%;RKD+8oSP2K+%GavZQ9eouWr89&G^?8oa?F zJyu`$TI$j!LvTzmO1-3m5^TlVr z?_qad1Z1`J>zvjdDa^XM2v$8f&y)~x@6lI+l0VPeH+T-emD*eP&}oa#GSO($#KZ$XZpRB~oJoJ&Wip>eNi!?m~7^HlJCcs=MNu7iYLqgQ%)^9Wq$ z0>d^W4=e_Qt@zRE?R1Y3INRk3gzt6D>X@%MwE>)sI65Tz9YMtI2A_-T6e!rDW| zzMXgk zK_7dh!(T4p((#{G*kTSCI~z-jC?9Ew9Jh6rl1jAO}i@IpiQAB#B;GAgCXizAn(3 z&j51nBD&WFaup5spt8(r+f!@&xm%FaXp?LSHti2)m97vMH8p>b+F*>lFyIlmg)A8!tqJJlPiFwGnnqZh{JQr8aXlCa5 zT*}I;Cyy!&an$u<-SWIjTE9FBQX>0JJms=!^H(&Kw)P7dBj^aaMwqTFsQ?YlKp&O+6$HS&56?Ikq z)T1}zOgI^O`Yc@lZa`fmjgRUtxB4e!MRFs(FZV~sr6eavoVxECd;*v)8P=7I9_Hc{ zh*M`1-=hO~a$yJVQnfo0DuJPE2qjqg_IKAo7p)%;LPcq=^%(mlx3FSx@mQ~#hzD(M$^J{YWnv+%}!j5IZhh52kP zMSR?;MtampsIml4|LME1bYsNLfoh-os&(&c@v>EWY~CSa>omxC$B|n;cfs>MD5nKq zW*#kNxq<+3=iBU`99$tTXUHHO0(;vfKnr2M{YAFIYTIWAdn8tMbk#;hxAZiSw$q(a zgps0#GdnhyC5sNYcz1wM(nLGM-P#=Pop>&vnDK)vF)OJ;ht;g3-~BKV`@=+03w(_V z*7V(panXfL#)aLwhxRUm)h~0|DGl*M3SPr7*^|x! z31LeWRaGPYCXP9QPsA9*wS-hklhTJOqcb4A88AoqWwR)xmbl=|NF-en#;!xtK_ga= zez7d77Wq%ZmjzdH?m?M-dCC7Vf{dfoIzRZ7z3FysKyMSLizvhx24j#aIlxVcX?59- z%^^8S3o;kb%?lmEi2@q?-Ggwx;Xq^u!{>$aOyD^?`#w-lt|CbcQ4C_piYojx5XNi) zjBeezpfwx21ZVXJj7GGU%$EbS8e>xLVYBkx&;FpxFdCsXm=h{sj9e?dwZRb$XkGUb zYcVjRZ$qejhrUv2^`X}*toMOuleBg%K=J1ORGJdZ*sgs)^%>Zk(rewo*`(?0Y(Bef z9kTxVI5+m7skPlPj%%1vU61MoF85bM@_OEa@QSRE1KG0~WaD79J2;nWkBPOxT@bIg zTC13T;5)vrxW)fsY_q~X=x@)ps=5Pf0zsF@QOn%NOy#vB^Pe$#PpNYX={krsentO5 zj0Y{Mrv_}KIEBb)Rrs*asN0)myP9pL`eFLkju!MbUFh)MH^Ek`$I(Y5nyW7KzG(0ehwsr;QkO%{?Shd(`8*&m_OFM62$JQpGfIi8lKm+h2Q-|Uu~#z(pF66(7i z4(R(!k19)DqL;}OT~zh1gA1#2w~0Vp%N5DfwTRRWW$xGM7iRaS+kt10h;!_d`b|JU z2g8>ET0nxBafx_J>_Y6$Q>VS_{iPGD=X<1d$`-vDP1bnAB=Z=y4BJO$V96@(0rLeG ziHK683%3JLS|q8SMphM3GUs}?l-ZsnslwooSZMI)Z_#cE!6qjC-CFRO3WV3WUq|L%kN<= zJZ!oF)@ao^94ZAr06#7!+mK%mA6GokNKw%XRKr z;Ip1cot)R#s-OBD2J6k=S`*gM7)L8$27pS^-9vZK9?ex&(;Gxa7wFV!c(^};V5w4& zrOIF5)eG~y zV;wnCE4Wcn{xidJ!J-K0P1&}@`bgU&qRRA@>!a?xzrZYGU))s{Jj}D5HaGx&$h~iL zo~6{hpqoi>ivrKq-s^l=VJj@2N(RVmy z)yYnjduS=@<)|y{1o*4&ix+GPQ!Yquoe-kL?IYu*q_&lfkoNpDsq$ zTi!E6r5shlOP2(S^^wqq*h`%rC#62g6Wbp~*3Wb}>AvqW2io+?xDpBeY2cFUpeem5 z1``mksStrVTWF4s^d(HeSyf>J&FKL>q9{)N z-|+l@#ZR7p(>1tR{;O-Gum87%QNz*UDwxMV*_&4V>(R8Ecq=ycbTN|5z>1jZ&*WX~ zTLVG=h|!#(M1Er?OzHJp-nunP-I&o}=5yGW2v*wVxm0V1@HJ(=si;$NGX z1VA9HYJY4*+XWEx>niis{B636raWvC@Q})aJcwF2IS4LQ)=N4HukeI?snYDFT z3Ex1_x_f3wt!A$Z=TXk&^h2h0-|YJePfP#&eKo6c*5UiQ$Cw2pOr}n{$`9y35)oP$ zxhRB-!KOE0&#*iQKN?yh4?!Z^ThmvAm(#D6!CG)-zKU4om{H1?C6#6+7FgCwVx9YP z&nlhem&4>}eyreQ;&bmJ8(Mx|y{_sBQ;#Q_^gwFu!i8`FaBbygsP$koYG51dBz$T+ zHfwVcXNdP{ojDmXTml|#-}_fPANweFu;E?dKcxYB6REMWEsTVG2j+8{MwzMN4p~nS z0*u(`q%{0IXznJ`f5n!+Q!@sG3OldKdEE$tD&AnQvT@ZYn#1pn*uOhER$Da*;gsLr z-7%`9mpi>NwZf`R?QzznqaA+aR=<2`jUwRcvDDA}RJdB}a61_$tr7YFgG!>kFp(P% zK(xj*q9y(~Wf3-$A!eo_DfChyE?)YhFLz>q4;{lcl8H}BBFQAtN2RxD59rD04<|d( zSNl>AbyRwXUq9nptb&xqL3Sdr0>>M_1{JA@m&w3CBukhBCDI%Tn%*!E6YY0n8TQtV z0zQ>Ki`|oeljMzN`Atl>Ru(@Z?z(EgXH;bo>ei2ZZ_IX%t483k#s~gnu$DJb`yb_! zR2DK&ZJOMghVwh#+*7)05swINe(|nqI*MCjqvxWq@*WAQX$v*QOOE13J{;*CP{Ys2Ok?%8Pxj)$!??uhb9EmCege8 z|46{boIIU?0mjDuv0~E#WA>|`k=LK;+ETFg_f*L>#9^DnR|?y@Sd_j>W68+th?y;y zltBDhH0U-Rmd0QGJ+y6NzB|KnEn*0Y{J<`9^ym??js^zBl;/p-R6@BGx*k&7<8 zyGxcx4O!(K)%?B=xOVtqy={aaH8QD2p&AdkfH1ZPh}6*sxcp<40U~or9sM#$9XHe!3UDuUwvq!;;S|t;b4i@t@)f>mnK=E~GyQ17owpgrdqsb)Dac$97 zzJ?dq8^g1kvioCv9g7&+qBrM*Fmeie8)SA7CQpn!v8h~dq zLnv-9pz$-_0lFpqK=aZt$2I&kQ*fpi`?4%z9Y0jdmvv3MS({T7=FBQ)wKN_>*7OaO z11-aSUvzIY878;cSC4cP70$Of({E1DrmP!CR?-xZn6Ilt41!10?^n5RxwY zmomF~^4^mc&ikPvv_VqgKOpS>BFOB>5J(WhQ!rOz4vGmT#n3rk$v7!A_w6nb!ZblL z74*itOVIisx08ci9->kih`sA(43R?aQO#Vd35S$;Gx+&t+v&O&^xVhZro+DR5-V2{ zV2}>epTKFGajQBLU4e{iv#2IgIIlENJ88GycG{fWCS?byniiY%#(T`94*}bj$L^A5 z(h14CQXJ8=^WjUk4fmcqK8JRQ82g24<`x+uJ0hl^p7EE7`?#-dr8RW z(md`HiB#qH1?IoBF8phB@1Q*d@^keMeaQQ#FOF!4`KO~6Q7cO=6$>ZaAT3ZkzX=L! zT^3<>5?IF>Tkgx}jO`|7H9lq6xl0e=mOY;TF)#`xaf-=FS#pc4`1Z;*jz|`58d(R9 zAsP7&x|oU(_%CR|%KA6kPxeY>1Dw!(Xh?zx5VUDesO)?7o+6C45C7Jv>8*oDy`P=KIL;QDfOPG>`E4Il1ohVYK^y z_=qtxIp3U4;6i+;`S>dw9i5M^xPJ>GgRzXe z9e20uemqL$K)6A4ZT^DqKdKF-Fu>@A^`RaFVb{0IBkdr^VC0>vJiib4SPGxbQF*ro>57tL)V2;Dy$S?I)iX8-_%vCBXKa2lRiuT3?H0*RZ>nx z>Tw}?ssf5Mm|+^?QxN!WONm7v-Vx?!KH@4Af zv@^EPC=lO8J&-E{rb$!6^yGPSRE;0ZbnkkyFiE&4kKP;^8GrXhSKk|Yn*Kh6$P}e? z{Y;8Fj*EI>dV7$6XxO-X7xr3@PFqBa6V!D3`Y6@QMy62hC8E-xN%9PEG2CaLu+~!Yv00+B>v#jqU7J>sE-cNLU|WA_%=C3NQq(!|=&{$N&5sWV*gA;2m+= zverU{vA=u?*}S)z@RrrC5&{<%N}I%NB1Ahi$1EGMmF~3s?mZ6OD{=+H_VgOF>#-4S zZPX)Lj}2i7jFwL?g%xgwy~Vezre=p)B$*Sii%77ssnq8fF_=cF$-%%!Qb<>(VEf~w zzkW-Lf(hctwniB3rkUrZ3PxRS)Ucu@%AS6-qF-#WMQUX|gtHR9Y=l*Qe9j6@9u6_$ zwqKYjiqgASDVyX>km@*8j(bIKf}sqL{+cIngse3QP=bJ1&`RGo=IX}3zqgiti?gSH zO|Dpc-P0+rDj)#xTx49VOc1L+v5gnNT$9sQz?MQzPs~;RPQkXvVfxd9D_;pKw<|{Z zPMP^_xNUDrp@lz7IZbDdKSi?o^nyFCz`r_tbZJhuYGi}n*>F+-KDX3p@R!T(& z&G-IjH|)HXRJsso6~%JEF_~%2C{3&-G^s2U z;o8~eT|d8a`nho)zR$eQ+JRTu{i~Gd{X*7`$5vzk^Zd(Jd=(d)qnI^j* z@BZ_}MQu04kAaYDMg)(092n$)F#TWNHhy43>@Eu=x-PHINVps>Y_#PrxWn7~6>K?U@u!Deh zx@4XW^8Ta*;tiNy_K7}Er~yYS`sZ)^47POTw%ZH`Uafa@1~-~US{&j2AzRsVZtYQ5 zeL{Gx9MkQ`TGU7wUaESWs>^F?7FTcf-Pv0-!uQa z%w5+!i*xaIy6o4~?|IrwoJZtCx8TWFzq76~y?eMMp(b-ovn3Ug==X-Id?2@Y5$;^* z(W1mNFkJ>&DA*&+q~NoM=w37$ca`=-W+O|UXwrO&#+;7MYVnpi%r+H0n(tp4118)* zp&Z}?RKmwUD*|9><@i4gp#Lxctzpnded=_+bI;Bp%tX-h>;&O7awm@a+^i_^tP0ed zW_};16e}!Okbb(7VH{ql^rJ%D`===^Su!*!Hfn!K{b^4Yabq(>2$fW_2J#Kj>ofPn z(|$)irKrp4SW71TSEwoOg2hrszscJWH$;FSm3xOCTCdRX2_|_s;U5p0KcrS`=Ke`W zF?QvN%dU9*z(Ul0qnNkKKH{#S$59roMmrwtnivJ~2?ryDl00n8_~S9->NLy9TOL~L zRX6i2c)4}S<|aOX2me%59lr04$8R+m_+EKu?zlMot@exjVyE&RbmOSyarlfgfanMY zRjn3=Oo~y~GDbPW^L(nqmekFYa zm%)V${y$6kr@sZzY+T73x$OV*-r{`5ls!2I^3cUAx<6RF9&k!avjR@P71Fe_4vu_A z0!tQxPElp54(l+bLD0*J{G(n_8b!JWA^sp_^SBfpEF zAroUkp0k9lAuU3nAwSzIC9Ca5awN^cEn5R|P_cz$B|XtwUlMmI#Ui()4#*>B7c;aQ z;h$AthVfckx@ti$Sl2jKV6fvBD65u$VRLcNtD3O)ty5O8LRDL08$yzyGt<*adSKha zj}d&*33q}1>K?>J;3%na%xnqi71*2hA{9io;t>vEmO^AAmWa*s>3^H2`5L7v@C{=F$C08crM$PnWpNk?S|A0BGC; z0!z*H9yH`TYI>B@+H{1KlVZ%vcI{V^6Cd{PjJnMIAr)Jm1O<*L<2;ZtWic7p(iRmg z5P4DCet~9oM(_XvS|7$IpBo|DwLz@f0H2$3)a~A{er-dP>-}?vBhIc zOZXMEp%Aa}gn{qjib083l;sH+?uH^uM&TlZ$h4$F{iNvFYHKX!Qs}A^3F%Fafkt&C z3=ycb`a45ajDAMsBE}my=TSC4>UI*PG#61fgVEV-zvW@I-nZo&8>|ysr0oT5v5qy$4FyTU!@f zJ%|Jh^v}=)-Mu`#xKA(aQTaFg?;RiB`nRs?6y5ylPA4545RXpj&n6T5pCWqK&ezw^ z=EGk}^nMJjUN=3r0o=ts-=7|y5B#v--FghCv-{un9f{r~?*X%We*k#iJGTaU{4Y+5 zi0*n`+#e^-L_44*^jv(UG{G7ksbbu{pwN(?j~6ygBwKoh>}^iF7GtI^$>k*vO|rZh z+P6h>jj}!M#Y;8M_^}|E2N%*XwzG`HIe1NZ`b91>>*W;c>DyOzC+qgjaDO@uq-AAy zvEo$bj3^z20<3CzX<<{t)fyl8swjd=bfv2*yyVc z1dhOIRZA1UL(#WTwJSa;$+N<4oFv&cqWvt~=$30(`ZF$b2mPwrz%e{&A7&So!m3z* zjDVVPExde5b3>yZ<6I*!tWq83Qtxzv!yCoJSt*|x4Is#>oZB(BL`h>^EmCL|q5ahM zDln~4ko_1b+*r_%EBZBHo&Ju@y=n<*Y2rT(5$m*f;O>n}@k@)Er+SK}hc6;H9dcvS zX8wElb=h(H!P_Olq`TwylSU{d|kJrG&tqj-GZNf{W- z2WUCt+L%;q@%yPTd}0K89OIU1LR|)EYt8C@AZRsgwRjA`S~qBA8@FOklE^A?DHaq{ z3@cKy*$y`MG+f0hDXK}mSNgJDt76Q$J_<Zj+>&%U{oj-4o9Z_xTlDzEWaL3~P?o>hNuyfPCkSFWLFK@aSETmai~Ros zejnJ!#>xIq&$u!a2%=0cbFiFuNb-SQn}6xa_4l2y#+j-va3XcWc92^lyEKrRf9c1= z(pd9t101#x94q(+3nT9lESY5#4UC!N?}b3QlfO>VG#)n6L%?lYko;(Q2hAI#-2bNs z3r{0k8xjQU={N-R9g^ScQ}I8|Y^;q&-&rAGUq&E*KO#LME9?H#%>K`KjYB_#Ac*{n z!7i^5$QJdW{>2`4?tjpoh2y^^tMU$bA0?|7&vXxP)RS0(AY|d3Zih>o9C(E8hpYq`GxXJ*Yfm$|cNF{((Cl7{?3M*SiYjOqlshK(RbD?g3RJjb3G@!ZdFxfy?X_Lnqo|ize+543A7Oafm6K@jn`t%<;Vj0sRqma)X{rpZw@zc|mb$l|$ zL83TPME&u^+?Wy}QnWryfEkFoNC@+2l<(*VzKU@hPV>=m8wC?Fe2k6B5c288GH_2K zV1~k8sti-}d?byOgD-)c!BJIeMu-|xv?qMnUVQIg0$O;+fzj;Dp(jj2%zP7x4$aeqFxoJLC@aM;c<alHw zdXJV)#CNW)zs?_=WZNa}kX_W?KcBtoZSNc@#y{tOyIXv=E9?(7b7}8fEsKtBY7epa zY|Py1<Osj%#9dAbW83LHPbw)Uaw=~4}moOW(7e-*Tydkem7wUO8T|`eCxO<$^ zb8I&mr4kgXHY6>_BN%T~ja zjM*fAnDgE^rYsKDX!D6z1#2kEs zUz|?Le?7wZ#3czjfDe}Zmf+umSfn;YAvRCQpo!B!8&i-REH@s9?TeIr`Bk zO!8SPM&88QiO$O(H4h#&PaZuc2MZd!Y}-jc4qX$T%a1&2FIfP?527=Jo4 z{-62?%9&Kxi`9@f%>@B>6c9z9=pg3$ME$>&lZzwCs1vgRVF3>UZjlvT@I%0v<+D7A zY309^4Pq0Jkh|8G5IXZ|lSdIQtOUvQ$x4Rp-Wd;?jS7m5E*U=njH zMgzjm5h(GHz9#MH8-z^Pui*br92-*u>Yf%p7#m9x>K;ae{|EdJFgCXT*|+55gK80g zv2px&KyN|j0v%>fLf*qnJ+lHug5cm}Wlu`@z<^}oWM$z%;-6j| zLoKMOFQlR@PasIbPzr~_*~IF~(g;e`inpeT2X0CiBX^l4M-6KJBoC7L%>`oO&5J$( zIQ=ep=LF^DRG~}-!qd6eHo#29)F6fqk5oYl`X2zAeh?8}bpPl=gZ_;pumAW>I$fFaJvcBXo_h z?~*T)!w+NukRbs)WiVOUsV||9_0>__^$ako;7dC~T|L=skoqTz5_vZea&H}xj_(2l zJ9@y5?;CLXh9>!E{1S&0=yv=~HFA?1U)+#Z99A9?SUe%1oQb>sK{nTOa8~lY7FAgu zQCx8eoXLI99puJjtZuA%eC7W6j=sGPeTX~@M7xt-{O0usO2satFXiYiCqc&;=SubcJvY=IWWwIyI)&t+;yjnG6CBd zSEklHp-jT=knc^&t>c?D_V0RiH&x8S*HE&E6U&-N7aVsrGRMPS@nB1AGH%;q*~uo{ zjEP(((L!P?#4k13!A(OFyiU@3zYpoE9Jm(1&(;m@vGT<@#XoA^&x!XS`@9`niZtJ_ z#8Lu~B=$0%k9Q_(&pO22KBwWYv;gI0V^zu#&5^x?EOON6ogU?JdMOJ@jz5cSSSOmX zqXfoR7awl|?>TI0d46lC;{3*6x7uH>Rd&S~*s#h&#c?m9uB|GFV=fkUJ}6tLqHUB< zFX%(oTOH_)G9gC_5*)CNmDKXuIJyZ#6nh#?1_P2w$qlXuQV;fpRwAMQQDEv=)IaxY zSfhx;YiTg_9g(9WaWapU^$YVS77p09gzlOJHsIA-#?-(nRo+iul)JVaHLi7q7ia;vr|HPC+(5Gy$*&K~+#GweW}vov{sd8AEnJt~4!saBtU zXzq4Fz%mIjLu}7!7D(fAk^+HNpH1EA-Ihf z3o6?wP}+nZ>=hH7`{vQ)1r~QN*%8Bp2{+r;0@NT~YXrD4CYh3@Q|Ut+k0du16Z~l` z2;Z)B8JCo#{JZ;nll3T^UmnW7dyMUBZU_$c47AA{SG&rGW_DbdMZf{Ro}SJ3%~DY( zTo*2m0_lDZ`T*EyTb4}pme0YMsKOu~UWr!q%)Rua#B}3$I6X_#CC(+eK%PU!cot{E zFfGsQwn|(V)I}++c*15wHJ50|Ez2xcw14si0v=b!0VnyW85ijGiSC|*&XQ&`E*W_S zK^$@8OqQhJ6Bkw$DR9$~2Z^Eo+qVuHa&rufOcG9UXa5LJgBX2WSdCD)a=P54*gAtM z1O`+kTm!+Q`lN1Q*$BuJ6KLzz=yt$dk6i=N2%-8iIh;D%cc&+fi~uE28Fz4GvOBFR4-MVZ~ImvCVFT`enNxh4uHV3w9!v1HSCYAd)d((o%k6N3}^V{C^J z#?hhW(nEunfh{BDYoDLFOq?R=^Du2ZY3`-j`YrAl8?IpnGfqp4lrHj(kGc|3yGr6; z(PNGcUFkx5tLW@Rq)La>QT6>)muODj34@pc`qk< z&XURN^uZwDoaVFH)z(9VJ145N(Zi}M+=_^~-|t>S0#7uicJ`}B5%ySo=rCGx>V83uM2NX<#)UohYuK}%d7T>` z#pG%|1z^g^BFfi{*eI32T$`@PQFWdYQtb`sN(!2|F-k2H15=5_zt{uac=t??`M4H; zVy*-!N4HolR%I}2+VbkcJE(_WkArS!4L{tFq9}=*lT8?BQ=n`^?+PN^jp~B%Bl?RJ zU5VM7z)|&U>sQ;6%syoBr4;!_+k#^KO$9FefV<}UP+d)T@-&Nhmsd+1m`wt={dn+c zj-c+pBf*ya4rqStGNoSB_G2Ovh%~HOJ%Wa0KryQngjAQ%T3KvHTP6~}Ou^r2Yr5aH zwZ;qxD~<=E9aB?FIuzJdY?g`p<&ewUe<)ir3{Jdme_`hyN|Dm+f4ymwEWUm|!M(q1 z0#W#f8cYe9X!x>=&A;tJ-{1FXvx(j|E|2&xKMDRxOvTjX(99CaejGMQRC2(B2PKQi z>~A^`!CQQqh}yB%<1}QgXz++QG8VsN3%k<(B1u^<<3J9=iyrp2R-;9SrU4#)N}lS-m_ig(5l!G0#7{4$@_FV)#yI!Fb%8i7D;?WAPu93TAq=T ziBJ71VFp?&-6&MHKk0-jyJBgyEa);ndq}|$G5yxXeCEzzG-@jWCfaCPhj{kH#c%6E z*1>OT`nm;qA5!(<>8@BAt!ShA_-W#08<}w@QJPa8jo}dVpM6u!sBK;wAf?Bu|IZXQ zoLM!Wk%0YCoxtSWL>|c^6Iyu!Dk$zM=SY}1h1~L0Sk8RQouiF**&zWf{+CQVjDu52 zJ{J6I>K|q#q-z}4;>WwDqJ?v%MewMqG&OW9BJelR_xH;k6g3X|6%z;tH4dcUADqqj z*X(~wGZ8nzdj8_O$LOvZ0P&P67UJ~e8M6;}?RYi&gaC~ej5Z2^KEdbMJp)4YcAK9G zM2jsI7D#p)0D3tFA8_qZ4W2Ub@R=x6s)X=?{`#L&!F+FqEdN0(aRM7u;3a5>a$(x& zD|YpNxQM=EwW7(#v4{2x!KdUoD-Cizebc|9DX7wI*aDe3RUTt$z>-khDWWdzdk>;p zc$$=QawXIFJeFoPJN+nH@Fj<)4r7RIdIuIv4enSo5#x7Oa~r5qky~(AOI#xd9_Kvr zNb_GBh!g$R02J^chM5w%2$w!0zHvb|ckejgMbg91AswIW%TjBa5z^gyF>QLhOVb}Y zx~!2Iun1;e#`J?;K#>xB{T@0X$V9X72W zCco~eXkUZOl_WLm27C`8P|3a+5ZsVC9;|D9>CpLg(0kK<@qCJj z<-hikDY-|J3Fno>4VG=zXb>MmTqhrc(`fBAoa0FVbO4o-E)E6gp(h-npAu;oD;!&i zdd_d9Z&yrQUL0&b@|6AjEhaVQQlrrF=lOD`>jKb(nzjpw4pxuh>4iFi^A>FHAUbJ} zpbV@bGIOahbrj^QO2>Yvrx2$_13jD8(5;cFp6?<($Rq9kX6~z#{ zL^E_t|Gl)zTavZB{S%q9hameLiTSPBp#{uFw!FgfL&z2DCW%;L9$7wNj%nZ*8+uhD zCVn0%8j5bVhX`beJIvk6GiZ?s{$WOoyqOEESA>=Y=E0Y`vo9f+x`<+N)h8c|aO)6o zQg*Y6dhl*z@rD~!`lml_%%XYihGz0|tgftV3~uyEu8gz$7RTA&pr7~OMtf2M!UC9h zSbWD!T~XTCROCUGZ?53EX}$FJFT{^*FO)M9aj9Fh9O}429nCf7vL?=JEUXS0!@mG_ zP`~VMr!^wvm>j#NoOF@=cLTlyl0-cIcNCvP{_!>*aD8YYvR_S)(Q zjoJByn)eTm`YALbNz)3BQ}^zO#?u)>Y8ObIx((A@hgz)xDRSNWBmg!hr@#>P+?KTyDzU? zw0VaFcJVWq(^4FjAn}BiWU`vl_j;0|5euPYzH_%3sCQIPTtlkxVasc-`tZv03(oYV zKyep^n3af|J1rkX_=OqF6Q49!l3~`f)<}XPyb`8?y8T1aHqz-04Fa%Jr~^z`Z@kPT zk6@JblGDU87~LJmdqRkK5sow|HUIe;Xur{k=DvF(j@D_qI5Td6)b72Ve=4>0@kis0 zO-hTJBY5$d9|u|%SpArj$Z5-7lKDLbva{)cGn)}?3K1}5p(3h@7al|O8tystG1ck% z=B9AF*3j$(1=P9MECY@+sF-;?Q#XoS4Yi*)P3y@j(31Z_TPfHU)ap_nDVo&ub$>Lx zCbI`iR2h>M|C1fm(2)i%ZUp3j3f8q}@_G0xRP^Xf%sy+ax0+>9qrdFcc+dBh=zF zFPAm9`?};-#{fCtRmyc`LK}neuNCIxaJ#O%Xtukm?|-kHY>j75{RLk0N1WNqvwxIf zo2UN-3o^{FugT16W&Q!zVc0ad{wD=K5DxG3K zr{GrqoC^pITr>Q!M-pr0PpVbmBBhTZv6rg|zpHxvV+ByvoPK69RaYqY{F3v4eR(04 z`3#CVT3pMx%9D87#mh*2Gly~@2BX9StqJC>s+mWajxcYz0s@ij{ zh|=L3FXrprQYuIJqOfHguUOl|E(2dA=|Msnf-(W~dAg~gSZ(^T&bhGbo8aZiV22ko zhWUV17&9PqX?x#70K?7S4((hzRg0h9Ce14iaUr$NMy&{i=H1MTx|%rr3j*Ezm7Dvr zK>ORLx@mfFe>bZ{Hb z{JsB`1wXX|UP-HNY~FLvHy7XgiL3k090iHU6c>O`g8&y>@#!k5kfGUySVw@s`VRts zp?_LP?slW498F8wtc$p(3jmR8>!<(W{Ue~Xo=dWf1Sdq_fi zeLKeN<6uJ-yVS_L!-Krj!^+_EckaXvz;zb3bY6ss;)V_@%5#$24XdP*^d8gFj(wO< z-Ug(G)14M?iwokD8tPJd7V#dhO8-@PPPlaoMEcYJfd%81mZV8s2PkjQ>aK1%J)0iV z(p8JA<4+QnQMF4}EDAgF%@CWS(g@pxdPZ~`MIBPQG8RCC>2wLX-BxyT#86vIUJk=c zmTGXWn3{?MS z-9!&fI&uO@*y+LE{R@H0cv7hx+~VK=UaAWh$c#AW(*7oWHKKihQEvWV3NTj0Y2<(2 zwcm}fQ#0-FG(svh3ssyv?O58phOME+E@|=TO(Ip0HfK3@PI6GGC~8-5sKej6G6T>Y zul~IM=U=dX366mE)E^u9D_zxOMw2fMgw<0%d`PHWx8!iZi#wLif?%36AjY2{kqosm z?9NRvlVv>Or*Gb-N%bAXZ)tO5#KRO<9n9}c>Gq1hiJS8MCEpij9&vv$;3$=TFxpwL z`r}Ud3SM*?#)|ZcLB;Aw8clHp*zf~SwhLMRnZsD0DNz8zMY%Y8C^k=1RJdsaRHxsvtbW7BTy^>#bN) zpyTF(i5#L2gEE z&ol`TrcF(Rj>M*20v$i8qY2znueD?aIUUHkxS$9jrYb3&AACbtlJ~%q(bo!!<;kh8 z#?HDcD?(#1*$7jpU7tU}{vSAr4SSh>D45#?wU}X*XuGCM{LW}wxtDw>hoPYi!5>f> zlypNH6;^5ah!5P*;fV&01f5#OPCYtv9F%AxXM(<|kMdRw#_Dfj9eKNug;Y+{5VK~l zS{(WJ(8IB^ra3EDPi4UP(sL32t%x=k3cnN7fqm5Uz$otJDldT-B9ibJTgE@Gb~J8q zZQBt}u&qUNN2wjc8iJ%*FYXi&9*}5X)-~_uS;ptPUOlNOkHm@!IVjirLk={9KV{r% z@Yoe#CFsyey1&9;o+kLe{Q%IS$FlOyju|S_yDDs*UeE#F?f`&C;*sFq|2l|zX&f70 z>s*1cvIj!P%%K!zuZo&+gtc0}nc!SS34QCZNZEJy+{R127db^`DI|XBX+`NCV#jP=1g}oAEh`_6bq=&83a)g`4l+r( zr0mn=ZklbhmJG3qYzXN`mqrw`ka010n$*6eoKALkhartYSA;3m-l2||tOPHd#e}w$ zvtE4AC<2<6H`yBgk{h*4Gb!P3@(MnI$d)hj5B02)d6Abf1Yl=7&}V5O ziTM+%P%76CwF~O8E+$1vs4dUqBmdIhMBJ5KW($K8U~DiAbbQLWwq}ejH1x^QZILCW zdlH#Pxo1Hxk0mtpA%|`bO4ysdCKrGlovxVd83SGrD1z!R*!Z1Y2S?#uGf-=v5a84) zD`C-2enfQL0(=8@nN-P3-8}{ii7E84=3RZFv1}H!#^$akCZ8Q77&ucfA&Q)p=!IOh zi#lENEzk5S$uH07Y&XqwtE=eO0jg^gOi&1IBIndTh?T+Jq~ zdPsBJT-|ySjV`pIHy;ouM71ZHqy)<;im>o%&VNrUirgId>G)MTi>IL)eji!KS7zJe zKO`V#dZP+wO)I~N)+Oj%yA2Sj>W8;|BY?~hdKjDMbLLHzf0P~^ek)ozQuL`>`Qk+_ zgyInLlbBIzV1W*C3SCVDG4)3~{=!cGKKFSC2*e!>e4dw=RR1|q8KOCPGpKOJF{<7i za<}@R(4E5O_^j5_c`0Gb*fX+(#nV6EoFgdtJ2{l{iVrDrngdv% z`)S#R!_+AZ^-_U#hdm|3*UJSqeGv5RG^2=_DG?m-WGGm72IZHa(!rE6O;I&Y7V!_3 zcb4ESgw)w_)^98u2Hh^pwqsf5r^HgH&OI~v$Nt2S-h$C!79H*imwt3(?OMvA&(toT z2Yk&d2X+=XDy8GbPmxlwi0orRH~;~m!+NRLj$s)}&U=mY>QUFI&UjFqp~pJY_7le2 z+A?q^AKZ3m)m48!|J2?Z)xAIwzwj+2rPnfKKQw|l!Bkr_UV%T6c$HsEgOQ_jUXV}% zFY?b{6&%N|&#mNBY*LOjkGN_L2yq&!#hNXVgOl<+ya!iUmoQ-a@UhW3CjeKH$VX%z zHXnx+af< zh(V2~aQ{Gck3b25)}|7a^oY}IaffBiETi|(0FCg@ysMq<;!!@$20mg=BO1jD9o?~T z2F-(l+lHPS+>fr@bE>h|YM?38hk>;cj&i%4R@_AyEeVIn*MJb-h`j|(jVfY|M3Jnk zujq?so3rck2Vv8f&uwMMK}RQbU-Et!z1l5X4AOKxH$`}wn7VMq%gGBL@h_;oXurdN zSmb%V)2^YZEln|e^sW8t53}kj?AW9i;U3Ju0r=kSTrfY5zAAs)c_4aEpFYf^?VCJx z%_CWp4E@|}NNv{f5k_aQQ~mD)T%;L$0#mk50A(EpVfp0=WYflAqTn(_fr$JJDglFG zis%N@14vS#iBWDF!Azv!3La4%^%$ssscTdpzJ>4sf+L2E6vn9kvAvlhic z&@V>u)BsjN-+{}&irHNFkn0-ZyoqU=6mZs(a8kK(mi!ksy@@ zv~~kG&k%n{N+uSDb;n}Fd7>%7VuihzS@@1qQw@xk{A-e1E^tr)cZ2p^D9{5+|6R66 z-w&m~i9ANjmr+vXerfKHt9=|LBL+3kcb5S?D=?$S>xika3Z3>9%kF5q&wg={7!YQZ&ly+Lt_(?K8O)7^0UK7 z5E+lq3pET<8iiaW-~1P7hH54M+K;D7NEPucvRAD=%v@u?SRC%lHC_%2z6!Bdji1#d zni{5>i$#fgDP2Ce-}?&W43CN4D8arpRz8UxmT;b$xze(q_Mmt^=_m{ z;4lNA&VpHMD2x>%dZYhP-pg0oK3}W9n`ji3Uhzjhe)2d6hjeIxokb*`59_4(9zVUv zS9^}0s}Mz$gUuq)aQOW+@1EY@DB^6%ueA0}U4+g-i{9dbcnZU@M#|fX;zt`Q&L&T} za7$p^@2PQZ@BX;+R4nlqfz%@)wh7(wLsS6R6YtTgtT0TxSYP`B@B949E7$h%fTw<5 z&4*AkCyBMmDb_+%;;7ghB)!Y2^V}+UoH|f}PX@*Y82|MJTguQyT0<(i0grd8rudFt zxUS$yR7W09h2HZ-`*d|reCOj=!SaF=eoK)R&nTR5sIMiEIRC<8&eIIy{E;%!OhjPY zAC?Y8m`gGRUqgTKjOJz)KdZH5abDzr%Ylw~abU zR!8UmdCz(=butWjX8JW)A7B6!SvF8fbO}-oe)g&|1ZVUx>3K9$*c-UXywK66nXlaA)R;iMRsqJ`hxgwd>CxMr@Ey%Hz{>2ta&%bI zDoM&vR!$X!GG5_zM+FG4?Mp2f@V%h8oL@+kx!U=7Za)xx?57f#3DZ8yC_mGelvEtA z*kMc+aoCSjpdnlM=i5l0UgE@3RCCLkf$g_{$Znell`OyiYohLM+tOMQ(An#Fn{k-$ zC!cv_*3Es7s+n9nDb~=t4VjtdCk+Yu3gNT99KV_l7mN-zY5*tI>1-0M!snTcO+(~3 z*QbLnEJNabk;+v36`0;~h+>g7TsgqZPUL?oLkHg4~LFXuACJTJ37?abe2H5GZKa)PeT`j-hwd6 z4crh~-d4cWPXL~j2$Y{JboEKQ7AN=4fGdyli^D(uHNN8X>L6GhchwuOIJWYxC4K}{ zf>JenHo{JtGe~<@vjo(&0Z0A(@{R?Q`eNpPWHrAmeL7m5H8b}d21s_%&-GG;M&Rxa zOYDS~?-9bCG8^SN9LA!^&yq!|G%0lEi7I`d8qIJV_5m7`W{ORXiC*1UgTJ~?o>AB4 zV&wJZepNK$6vE`8pPzLLlY|2`1R{zbl{5#L1nRf8yI95OA(tK3o(T>C4t5Qb`RjaM zn)m3Ii4t^|#0-*Aurw?VYqTRoS20<}razaRdbil_+6i}wY5!KkX2pQO#Xt{c-nltp z+ab&=aRE%8!;OD(vrcx5IT?uhearTfpv@I6aecP_utQw4cvp^z8c*&MZv_(^!Otp2 zKp_V{6DVFoO9|mAV_SVk3q0{i|Kv68nO#l&N2WqU{^V6={+W-5bvJ~CfqDQx{lsBQ z)q^4Vh-k4PLrcPzaY`tNxDql{ZWm^3r&3Mw*7+a*z~MKM_kan(Z5KLxQ0TogEN!Tm z$*&LNS)DYPAm3)goq(kI`JkU(owEBd`a^p{sV-~PI+v#8rp?GRt&Z0>m#)a;435_y66?AGjpc-rFH9d}i%&PQc_uK5>|1gUKs z!po@Dga@}PLmFzLR<;j~ zPAeHDbTvms?exlP54%&1@@qtaQ&cL$a0-3CG>ecwE@-IB_YS{ z>q^(|Z^b{}eat78k>6y^5@J5cT1<`*D&ep)(7T7g9rah;;ei)L*m~Vo9EhGFMLWk? zI~_cZ-2C@)^c509wdKuHp+e2Trvmr}Zpj`X`z$)4me?pDE_~ebi^#nmRX?J1LaQqN zq;BD~fe>GiIzaQ&un|FAP~4zix;U8l<<1A*z+p=;ABOG>YpUn1BqiagmjZ?sTc=3n zcM{I5$f}DmUs6HMMt9MKaj?`pr2_3boB30(=z6v4;jthURgqJir7Fyza2gnx8*@_k zrapoyxeOZH`LdjSqJPMav7%qRF=2*yTWYdVXT@TDzB#49GgxFSC2I;k!?>=%xjn+=M%qlb3i38K}=yr$A`~2&GOpLv&3DoWh@hZ0n$rc6e{M zE+EWmBZJs|2!BMg-I1C-oZ@%vSPwuN4Uk@a+Qo^@dZOo_#ch#Ao}gpNBp3>R&S^D)nTteJ8B69&J!U7U`@asy{PL69|K0W~2Gb^b|j9B(LI? zm%DW9>o+eY3XqU~3BMvxo8HFxdz*5XW_PGYVQ02unr%g+Af1?YF_pm}c8=J*imEhJ zrGHzKI#JeNudDCOLP&7wRn0}-)(7MuuOC9wEIGvAecxiXD?=V|RSC75owiUtUC4Uk zOd^t-5+<{Q8=Tu|87MlKhdu|i{=;DN7u|VoF?{E402uh+$n+Di$2*uy`%(V|c88-c zONF?x48=54LoXV$B}q+Ta_27#mP`D=2v$O0R~E8R_aKMZ{n_in0Pj^{_^ZZ6?=<<)n zH!)e?q=m*hL*9bgl!@}$mE2{~wiRb|<7$SQ*_L@rN$H_fKEm`S5HINY%RA=&GuWjb zg3rXzq*>K`JafaBNbk8#kmf*cKiAzk!~Bvry=G78E`LROX#@9P{PytQaHw+T@SWki zb0AZ|x_LBYnFV^U97T(r*jMHN+Y0<3D-|MG$8H7B$>J@Cu|fHzdbHA+x&9U*>s1^X z-08wdx#XEV2}jWw$2XN{#uu?dX!Bqc=_x}z2oBEFw^J3|*+^a^zX7w&(zMx}H_VKM}$*KH+%600;e=Rdk)g3V0Z6K0W-xx_?o;QAGQx~To*}+PX(7BafU%?PV zZIJCu$H1o8U0b;`v~_YvH&=y(*=4uK8HpU0z)c(~yxsQml^-Qzq>*~F;v-k>_05sy z@S?!Hae!R^aQKJvZy4*U`82TmN>J`@p_LY1NkP(F2DXZXE3_ zxH*xo?`%3*726HD`SrwrPgNbfp9r8QiJh6=+Qct$$TzKZ+NufV?bK;4ORrM@JNxz_ zbSLqQqK8B|UgU-u&ucbZDSnG-lR3vKH=G|{MT;diW3JGLsc~_lDrzOqw%5Tl3j#q? z>a6@drH^yOJj9w>mL(HVW#vY)vrPT0h$j8zfn!rDl~U=!s58PcJR1^#VXHhSInaN5Z^kxOcn}7En0gVV?~b@ z?-Th;DMKCEJ%c?3p;eU;j*0S8fYkB9*X>n(&1f%Z@wKXifh?{BFNoW@oIyJb8}=l; z|342Ie@pU-f+HfsoR3waB2gd^IF}$#HmIVhIY0ex+LWmwH0$-h8TsrN08yL;AJMPP z2BJ8EydqW(l}nDlRfaQFw3OCL2ZM(4EKy@qG2xB+@%|h(36GU-jgpgceg7NoHxJH_ z+FmF|)xM}Y-Lu_RbeJqP!L?lM76bFdX%N(UG?^Yu`^z$m82>hr3<02}xiJwR&gD9A zMG~1Nl}mE2S(csPk$QOD-7iN?%n#2!AkQTk5>41mVYbup;FCFSKE4{GGZ}W5o^E`B zkeHY6z<*^QPY}OW!aB2-WCWvcJ)UK~GznAT*u6D#8fKm_Q!3*M#P}8u3sHYRCgzp*?)oSawgC4i#mhg5t2A-Xdb-_rb*0 zGKzFKHCuRFdyU}yZQ=! z0M(`+p^d5M(v@;%)GnNiNJo<-z;WDR;vzesANcoRs?#}nR>5e^V!xF&Cqa*YwNb?q z>unn7jYw)@*8{AvoOswqSN`6+lwg;}H^rmycz7YmFlXOC2|bwS%)=xB4Pk!M?sUSp zVkgL36-JPN+G{Adgv7HPc17$v#DQi@v58H5fXteW-=Ug~7H{vzEk8?Ii|tInaZH>Z zZ8!&|iOheeAGHKQ%EWrkf~Q^Zkc3%j55wip?;MYNCj#)2E%mC3kZSB&j5i>g^L`aB z2uT0uYw^$|?!y!nr_?31S!d=2f&NVefz{ny?P+ZOu4<#)OdBKn-!!>RWMez7Vmown zir~fl)lX^=4z9(8eII4q#JXBT+V2|?P3MJI&i4FwpJs9Nj1r(Bcb06;Yr);np`U+zRtIQozREs%&BWR=d)H& zscFc~EB41#>A5iY#nWa2PR`XX=cPgI@A~V`E1b~9;!L9^#(>E>s)^}f`fp55>VRPGlPmr@H9(QAM)Tw8j#;Rmbe%ZmM9;|%PUnMH+) znCef<^b27gP;3ii&UB%($woDLmjHsWSH7v|rX z89{KhEX2qj_4RB-5spjVS0z9yW91Xx7P}%LIjnI@+`uSU84-rXziT zIRKA#WHsxDCjEBLU`&j8Aw0PS@N>y3}nI3vB4^t8cQna9`MYs z!COCY?n3ApAV;cJk{F!m%g;W$fJ(=<$>KU_e=g1ce3LzZBP_dX0EOjIA(DO^YYj3!=`0Z{Hfr4xZ$6QH{k z&I#l7n*6Nj?sP@0NOA622bl0Va0WMiZn!r}_5K@7d_l_B_OxEVHbWhec1_kaD~g$& z3xQqnmvTlRQx(i}$Ad0UDR~C{GUv!u^i22nnrKe1#w%F%5Q5I<6_EgZ%NTBOF1A{X zQKn`1YOv3t_cDg_xnpL`4Bk6p0AL^dOI>X8Hwhi1_@$*WlUb{s^Tv&q548Ix?COWMPyN@CnU5YUdqA1MA{ zt?W=rKjSp`Tf%QZ_OKr&7^VLMoJKt&Y_2Is)ata2dcV_XE>+&Yw$h3$psdQ87w!j- zKvkpBBR|Rq+cjDZpR>PiBpKPjFExJ9tu$lGO!Fw;Lag)lz<@s?*}hXURh?OB{}xgb z#Ui8-CHfQEo``5yWe+KJC4r$T3ISpYFmqYlHgm+c`wAc@QkHD-y2M*bp0CA`@oB5D z`X8{FvdSWjceQXRsDF{0Aqd1nzyCT)u-`PLHnr%GFS@Jd!|VQCi-|tCyM-%xl-Sc* z;d}4%`05#z8j--759WjY=*|ZAc)+&cN!9CJyI+W8`#i~7+&ALi1*D?))gvg#qpWN} zq#JhHg$agq{gV^$i7b=Z$+t73=n(1Mu z2hY7j%PUmWK-x|20p7>$$CEg~$>I)>FQ)NsNtU%Yc8R15xq!LD_N8y?>R|s{Is**Z zI}BDw-{?f)#R*pBRn7D|;iH%k9yTOu`*#+s+AWBYFfZ|uGj0yqr*-!yBD8dowj*k< z8QG_cET8pR+h8q+ZKiB$eN}f}qvEvSqp%ehQkMoM)w`}b;8QyrwlDwkGe=a0DJS9& z_)A`*q;SNzH&6S$>h!85sC$2eD2B<4y7*QHTq#$-YMX1uwGEdzipquCZ%X$flz*!C zVXv0)5jBV9AdD(f+PUT)D|s>zeZqf5brQ*3wkd@5M4@;B5#Es`#9XYEJVjL2h5ryn zKazEn-Dis$0BMx^r=%UoF4nc_)iMJ0Y9VtFjF)JrKav*TiIHx?51VIJSUt*r3+U`5 zj@O6nYt_NL!`~fdm19EA)2TU&j3lEvH|=i!6hm zqt>^Z7J42FF%=?vp$*fGKn8MS(04>a5enJ4<-xjt0o62lq&v9`o|})4w2M>PWI#4% zR}3rCj|K#U%IqOl3+*QM*#yGV#)^DoBhiIVGsMt{V|U`}PW7WI*cbJR7(Knurv0y( z9TTOx)i?9Kz60ryld&k#MOY4{NUKS{Gjq0}A1n6iJ(K!#+KNuc2YFoXW>!q<+Y68J zRzE`Ofq%Vt$}q#A^HJiCoKSzk<(hihZUi@UuW)rOpr;GYv$ zqL4E(T5%sE-nBP_#-&zo13)$1?R-5~A%(c^{Mi14AU*$l>BC7G_RT`3#A~W~9=d#J z8lKrid7Dvmy4AgrT-5o#sA*O_MO3CL9V25F25KDQ%`ya$&>zsa)~CR1BR7uApZ^YE zOFQO=h=ps1d?AUebrg`AUfKr{zg~=Q%D++3v{ZuE- z2i{4v$GI>jKXTBBc*Mxi%;3ELt}4_E#Wfbj*TigsR;NAhkZ8xh(jw`wFg9ddqIxKz zl*l$(nUX2VkPU-bYOgAM5|RC4?3-70yDB^z6Sh@ShpT}y-tiqqM$BSil89i_@Ch1+ zY};C;>>g(Ix0gaZmezlAiVXE_&WQXB1yE|f8azS`#nlKeXo^p6%kuk2rT*iJtwGaN zX17_TDlaYsHf7Y!p}_azFK?>EWeZm`%&qnTwJt{r_hvcOyI@l6=h0+5C6uxd5Dmf_ zT1&=t-`-;Cm(X-NMfZX9i3+h>CP(i0=+0F^mx<`}UA)qW_To9ELXK14E8g zjYMIh;nB!Fjl&fOcy`;21<^lKFgY^i5JgvtZYQ4$e`*t4!lw|3P0ZborFsmw5H_uk zxJhjuOGnwEM`}#>@sG%2M;NKy5a63qo-jJNY?+eJ)@YF%zDRwZJIc{1%$+;%SivfQ zE2DGk9C(OeCe_@)8JHP+cv1a924HGtpj2Mjg>}#e(*qNA$rQTQraJdn^rGt7oA^e= z0D>?d_@+p+0e@a_;rsiceWKNpMSnIEnq&H&nUMC=M83Q=?rh9*R=pnwxmSy!*366D zv4L~*(-U=KB>TsRAS~@D&XP<*>xgEzMS?%D1@e+!#ok^DH8Ci&4WGim0D`{pU#IcI zQTKH6ouanbEId!o_As{;7pCRC_qMj@!q<*oF6BZy}`d^VeXNvTqGBY%~PH>v0vWN#LeW}+{KRi*U0kd_?`O%WMCd=7I^^gxS2DQ0cbJIknb_`{KJUs-xqrr>kPzUd^!#@^DnKE|WA%x61pvvIkt1l;OFuC`-Ii*} zx`!>H9AFwg-Tl=ExJ}9hp&jE3!K3YywV`8-30s$e8K^;HOn3^t0XcaU@x3S1m^dUl zv>Mqt$+{55CGnizV7-nxHMWHr50PQWF^VJ?mZ)J|`#(f|V{~T05^ZeTwkNhT@x-=m z^NVdyY}>YN+jb^SUhchbz4znvQFm3Zu0Cs>?yB0Gp0;5A)SvDck=tBVne0XDp7vI8 zvw3>oyX2;QSG7|dy8rxe{0e-or$o}x?|HpR&PrbC47k zKr^%3`+S+Hn!(Gt6LWO#?7J3X^cC}=;1?>k+)pCe(o$z9BS(!RhN>z&3K6O&g~@Ru zm$Ry;w|g4moj-GsAoKQKg@#O1Fs^p=YWr5Wg>2Q?cYOjrd)wZW$S1)=f)2Ch@g*g=^?D zrO}0m`@2_J0+C&gu@F7LX$gx9olY~hjtfs%i0Dwv4{azLv#@5PTO);>POn1kO%A?| zb*6v&E#1nq*SC)o6$E=f6;-`scs)`XxwGL6S^Nm##1Ib0BFkUdQlVmeO`ArXo*YL+)|5banmVGH!kwf(yxjTx1%$*NmdhjO~qepY>Ed14VQa5`n0fCb(8O8DfM{F5SEg%vM#7 zej^mCu9WC!2WGnHh?g*Y;Govh-x#$#vHt<6m5R(J@V_6taoi5^V*U`1{iY9EL+yfozOn{ia=E1dS^2mriZhuQmywuw3>&rr#CG&1{i`$n=^HPCB`LoJm3Ss z<`(v4A5E8e8k_OdaM0+KJ>67!Pa8N=xP<3zq*Xm<-|HcpQB)Q!%^I}G2YEE6uufA? zuNe>4gW#ewKzWxOwMuybSS`<>y+%m`3CT^Qx#D_>cD73sEX3t%Ao)$>EDM`}V@_Iw z>HJ*PLWc_Mh||xc*<6SSiciyp`V|2o_rAv??sq9YZ;@n#t2I>L`oU}8)2^=ohi+;HJesJm(s%)KX{Hro@PN_gOgKh|f@jq2X2w>&A@?oPfp1m{a? z&`{E`duiy+WbcGNjHf)MO8>1BE-vC`&zu=-=wN$2wD`?5Kw>NrxbbR$Z- z-e4RxmM=A>PN2?Hh|=TdzkL9(jPINyF`3`^v*VKxaxE#&5!RvYSIYQyD-K#YBeu!% zmQt!&I-+tHV_^vK<7w{Vgi)ik8HlSZi7wIZNaK0gOZ6^8l@JiYiKwoPL&r0SB$oa@?hKi4{q1{J|}fc#TKa@e)`bB?69zt=YqCQU#vUq7D5dZ?{xe zYL401Hj{6QTxSrSegxliZ4X`6W}qgKw@3^d;;{ESa3Qh_Kk@^>r5`8Da?ul5E_`^D zGphCD9g&5`Wm?(YkAEo1&{eDf7iL~q_OwY@D2g!y3G$maOV5wL~Gcqf71*@-g&q9Rn8%y;eok)0AU}Vlo5B(-$$jr zNVbqHw1eVAFUt;J;=_dys_B>hNt>tkg2F_!G0O!@nP$Ou?p#wedPo1@*u$E(5cm2Q zWtHXwI=q4Hxt7ey4&efG8DArSI12j8|2=Q_jjHf;RjdzawD(i0lN%m*ywNzofBs!{*l}Xsk~W8#7$4K#+hY2(*(sW=S9mV1v|uK3uVzg>$eug= z$nF^rL{R$f^;f$`PnG2s0lrE9@@yaZDp@hQB^P5wkK$HlJF>PGqU6gZyr-F{i^@E+ z8TSd@4EX}Uge1U2mVk}Un{{8;tSJlkIHg~j^vF>|$U1NY#tCuV>f}5gY21oFk3w*C zsi?aRt|f+u~c|f3IdsTyqHR;KZ}tZ40#ggq0DW{`4AYNk?XW2A|UL zXiBjkBquU`C;H7ntelkMIjsGu#(+Su1d7S+tICd4kxV0JEo#=*ts2TBd`e~vVMb$X zctG|6jOV<`643SvjIq?MO%m+t`EB!-ktT=Ra1GR+I61mHyc$<32A6249 zU<1HApOn=*`K%6eQp~|A7;aNqVgF?%lurs5coWyeq#0c(uTXxDF4Pi)?|?%qAd5Yj zee;mHyq@&2%}0V_j(^m>9BV_ZT&wYYUe0(0cvKzZ7uKebS|bgaSH(I+{qD8I<;VJA z>e%Q9`TLrNmVU5jc)R5^15XSflDf&1wN8UZmfRquZ`4~fOYrDU*`9?dxE*V3;bP|A z=WUA00A+Gz*R!u*f;5q3Tj|ilm}ILq!_3{$OZNK4EMi@<2gA@BOARUrZR4D@D+s#; zBnF}Hvxi+)C$S!Jf$v~0PSXEo+Tq!E9nj(M2~u)<;v;H7I)Md-i4g5JiKV-`Iq&d~ zykJDODZ4>5yw%n18?6$Ad)<@nX4gK%{da%dtN6{`O4D0^ho;xp4|Y+@WP^p}C0|CH zYe0_TekxT!nP)baz>Y8O6DxE^fgYNT9x)56mpM^HP6G5GXhhOg=va|V<$)H?Ig-wg7Y#bUhj*A^Xo4#!Z z6~N0Qb+%q8E)Vf-+n!>+IS6uf95c&)&Zs<0+N+#Hx~xvc!jDb$Js_AxPR8pF5bIRU z9^CnJWu(Uim&uIf&FKJW8S1J{c;890mbd-Eq1$Up^3(cvlgtrFTeS@y)?<5wi4L(f z4VXp!G8R9bMB;jNRJzL7(4yuo0CsX%s5sW1rOP1Ut5J+9JUZj^7b+$vG=&)G=|x1+ zndXay{=Gcr-}zRdT0aVQW)blSSmIPgphF^_?*1Tf1P0Yd7?7ssuRn=G*1~uoRNJWO z-y%N$vL1bNbPCF>C6eehJy!`FE<5qPL?JcA{%r&S_QU!-GtZ3kDa%f}MiKq)bs4SKhH479Pt3APG@fx)_B# zz`MdzkL|={@T5|DURHF>*ppH|HDvY5-rJA>X`n}m-luF)6NRs!t^WAv^xZ zq(t^6M74e)QMB$wrelcq6`6b(@zveX0ANaI_)eqdiC*1YuFlvbW?e@r6eS9H<8#HK zl8m|QU;iw zgR!2UCKLrx2BMU49)+iSOd=17R9`xmC?FXPE8kNu!aVdrD%6YM_xy1;aRp#0E{QOU zJ!0~xpK_oW0A~KtWMUBM_GLkCO`Zs3#ff7CKHV&l5c);diSnEGiLYq4PXO60@$ zTO#PF?)Jm9kBjXTuC;x9H>r1R>8I`;m$Hp8AH?Uo*cF|J`?NoyT{P{>f*pHrpEF6i z)cf28r#zrmcjDGzHAbJxz=>-0SKJhNkk6^(jvJzW0ZbN1xAckWDB@Pz4*IBEIrUqX zZNc-QU%h>S*BWx=r8GWRB3#aBRW&}61}QQJ6`AVeXS-!%E&u|9I?WQyRG7f3llH1!<13K6(|%hjgvzb(_nTCOX>M zeD}C00qhm&1vf;8311aZr!+%+cf*fCpt^mE;E`<*x>eKCf0>b~GX%Y31ywc|fiMt3FD2O{RFE>eZ(Ylh=dE3;p#vW%JS2bb^G<6U8Z8wZ7J~sBaMEfQ22{ASg*ln*!`e=+ei|wn>6CzDCyhK4N4>~G7epqyvNBi#2$U$R?*f;%4ZyZH^w>{;`g(k5}|>x*V}u1v~1G$cCs8sjgR>R|FYb7HISV7g!fO5dGKGk%|sv# zJ+Mrt+n1ub-I!=&%(9600;KgA6#z94={xVQ82Rr{AX575(&}bPTt~k;4sfz3VN&_kc_F`w}#W(;c?S)hdZQhD6t%Y$G}pqe|Y7 ztrH)T$BeA%*bie?K~@f?NheF9Pgf3B{Z=}NphwTgan^7irfU^My$i98g^$yhW^Eq+ z6~;0IVW&_@|p&z$Q1cq$*B&sLfQu#a_ck&JYlND8B`M7`jYU zTp_{>uO5`$Y5yf!+g~te9?Mz?r<@F=Z_SbJ9`AbS4;k$y3-E6lbbZ}U1Hgy9W%>xk-+!J+x?Ql=PKf6)HvC|B9Qs2ajQ$;N1sIii@+fUv(Rq5Tp7ux0SC}Dxg4oKOZTf_5xum~s>Pe=l9)C7QJ6kQo)#Td-KdK*= zX_8>|PcYr_GX_Lyp{QAz*iJ`s4}(F-`GE|Z;NhXUe)569*hg^r8Eg&}7k+eG9t=TY z*DFiZ7^V(eJmBlvxOdOt;L8$a5&zwB-SO*J1a#wVxJK1|0hryRh3KLWEhF!1+d2Xt zkFJh`P6_=m=BllzZO-k#sB~*;30pJxQ|J+4C?`o`MB<)YGo2cnj)AaBB(9o%9&39F zZaL9-nXJ(AFMt@;SS>Wlk~6~ttVU@>m_=Yx$;wKH7XZQS8H)&V+@}n(&Wc<$E;G{C z-+CyHr6l1kvxSJv&L`rU(=nVaCIv`dRyDLbm+chToPz4#{aK|6o+w)8T%f{PodvN0Rb5a#klD{fGTNsjkynDEPG4G)S zip11RJmB59-c?7xdH_&Us74;0S`$=5ofypK#pt?dW)O?Rt3=i`_FSaPUG<)Fu+!A0 zS;~;ws8ArceUkzoAk=<2MFS?L$t!07{1$I*AnlA?$#-l24+b2 zP~dMhClvWWpSMLc8qw2BN&;IxScdXj1ir(vq<-6Qg|jJ8m6W`gzRg?t^8}|jsXx@9 zad41csI^%pGjuC4Dh`}(mDcNW1qbA|NcSLX7XV@`I4Z;C33dLka${BFKjna^?Hsbd z43Kg}Yfmb$MNj(-+^$q$0sYmJ=@m%>8Ny%0m}W|JZC9eR9~0O ztR-yQ`WZGN$f)^VwWdc}W#_%(?gP|qN*pSS6^rST5IRX0mU;vpy)7){NMiROG}DC# z@5DwuSSu2gqCUX2%V|7bJ~ebka^E-dL|4(rRh4hLs1J7rrBOG-gf>9pGwOo5*+ju7 ziP}rfO`*J-R4Zd$Gi+f5R;tx>!-rJuHRy8Hgiot_sKZ(JpK2~$WaSG3tpEwoq}#^s z4r{GGDSJTQcoqoPe<@dTdfLL11)KWu@8s+~_@*`sl=@_*Em%}*`=Pah=cidq^T_Bb zJ1;1)D@iho-TlE`0_ zo5#z1&jrUkgz;vH6q_PcA^>W-&&vLqy&xr`{~bv{As$a%n_w1?vwy~*8hoAeHrxKL zfHWjjdn%2_3ujar&BR@*z|G&+KyQF7cTSKfsG^K$+WFT*iCcmm%|W0MkYPXQ_bi+j z?0OYoR0m`0i6d^GR;MZ@UDifYR@#{&wL^dQ)&xv(#lP;YxUi1U;Rg`96>`9Aa6l0| zGWYOzP$>l}(hN+1!iBhBko)2l>RM1vax6a(W%+xeRWqQ+bYq-(aeqAU>JAOS49CyWT#zUa+7@dCF7&Hdp z-(rYVWSTC0)W*F^e*<{cw<4jlv``Vxi(1iiVK5EaZ8kvh}L7cIULvEZ<1Vp;*Qq zK~w644+w>oQM3B+h>ht;-v*xeviwSG5SF`&o!*|r;J0t2w$qwG}djDFff9lkC;H zLsMn?EpZ<(9&kdR5cobEF}&go0>@LVZ}Lu*{+7kBkAUrlrkn$ipvZP+999M4 zkB~EK@n2?9PXXaI7oxMcyPOxDU$(TiXv0N-7;i{UuU758tQzi>_n4Hf)2eO?BR4(r zM5Q|+eM;=zZmgc&s}+q7Aq}}|5k!__@+38WrC&Z7%J;*_phElQUSi%&dK8J^;%N6K?g4wc~(l~l2 zGEl2v`v4sLP3YqXnxMj0PT6fDAXdjiB(Z`L`7?aC&YB=8ZkYQ*0OX@(qeSTu;B%}a z2~%kNVUMtMZ!>nMw(YI{_?W>sNA2BVfoy{t=wTVZ1V;z4<3MCVHddr--fS@j|JGWktJBLGF})pQa#?j#|UxUJ{KWd z+1glgGXvL(YI<+g%O(v6Pb|Dcxcxk7lOUO~%t#9e_&dVEMwYD(O9tMV>2N#|=Klvn zu^#rTDp$F%Ci(`I1WVQRdp$**lSQIz{V>|pGPR`3XX0>pTPA1*#hI7=T#I(!fc}72 zOBf)P|6ysZm0WzX2v?V3GIiCV5$X<|yHy&K%fKg`^;dtU^< zrYyo5#~W-!$wO;J9F2YarZO-VYdAO-U`cX zN~_Shly@E}6!m+~&LS3xs)pDxm!1`wXEQXd4+vipo8HCQKz)WfMx)VSsidclbH0Xk zX&uw7NZSLDb#=GDfdm``^Km=xhKuvuYWK;7>mQcH;M%ZLNcdFqJRn2Oi(@oAP6~iV zB5rGCfl4xay9=$WjJiBr-okW|g@gVkP^eb~t4lmN+`ppq=-Z)|&uYzouWa079}=%H zV4COfg#SE1XbGA8w%}4m=7OMsR&#q2y1GN?9XFCe)0u^2$vV$_$gh*XsBDL(<}QD35wqp zbwx~@?)rpd2A}P;=-%XYE(`Rm1@EhUa7#hf+L5kH0?XQU+lMJqKD&Bm2b#Lat&5aJ z_wy_=Xy2_T5v}yxdKX>_lmZV6TX>F$Q4y0?DJ}g`E;=vnC4cAl<3g7(Bw+$P5JQbU zG3bK}e|?Vgt{rLfeONP1v}$CYb^9nO4W?k`Cmf{xR;2d3av~CQ6ON_GgxO2M*Z1#O zVlOgdywIhm(A3N$(@vymI@8-!jTA--88pE`93d+p)-d9Xz+P}%hoKEiMac3^+9I1| zxZ&w=L0GxvF^1~XUy>us?OFi5%~VMD4je}-)h4df-Z&Pp84G>tMwNd$fKqG!JF!f} zH}@OrPsg#+$3OYLRCh?w6@7osjTp(fLP2O9Ij03g7B92^mK0!n zT>oa8yFtj*zkG}s;J-4Rq$aHKyoQfTBn^^$2^(|?U}*$ZYITAaqbH!6XlIZ_KO-~b zSmx5lT{={&{>16W!g17-khcxStqMm2bRgr0LwEc!Jq zN(6gvBu^J~p(`yrHj@b8I{6QNGYrY%7Hb9`KFb$CBZ1}y!_j|y1OzIaa6TC1D`gzXoV{-iC)W>q?WdF@ z+&f#Js-c%B-M#gW*^V`z?ui!QJA|7P^sOV2u}Z z94CA@TGPU;CHs@P488f%4OX4_Bh?iCL6FnzB3oK_Tpk>JZaQ!YDIpu^pbPJtk7mMw zkFXD+qBH^^s<0A3K`v-xeg#T9NuD+B$thEscsL8GGHHr#l-cqf^5)(FE2OO6b^a?z z<>3SXfoc#Zn_|J^hROH2ii*pqzPQS(`t2#b%XeenPf~&M0OvF2CC~vCs>ETd#Y7+a z@^J>G9x&)z$6{7AQ`y=bNJS%E84C>qjjwq5e2)Y26!^uR z{C)}H&}(z#o51T)LS}ng$l!QxlG0DOOG zCHh>qyT_}ES9GVX1K-ttMx` zs!v|XLiPDdCWH|oMqV`ygvgeJw|*qMvvtHOsw{Nk=`q{}^r0*s1PI0N2*E9(w|+pm zl8VE)Dlu`bi~9cY=H>x{NWq@uY5Jl?R}PFA<_5*W_f_#$qZl%G~jtE)sIWrmrQw+fmgTc?iN$}*w)L8n5xQh zm-w5uC1TD&PN-K{AiF6OMk6o{8ZS?U>}YK2E|aWz1(#=!`iq?SOEGB1cISMvuF>>S z3#>xJVqPVJ%u0j@1s)}1$vmTNzqe=&mfNdUR$5vuXwE^{1pqz7v5Ew=00OwhGY>@O zz_(q=nV2?yH{^sH$^ohk3wf8n)EuB?-5_OPL~JupvR4uh+Vp&Q|N4pJ@-Xu}HLS+0 zK2rBgp-M86eh)h$!9@0+ub3^gy}ZC{`#51%Rk9HL+UPKkX1lq5O)>m+$XND*VkAx5 zDNifHtkV2vGY`9Uwh*-A$Nxfg(YhOKv|w|-0@oiH;yd%mx`)61l--tG$tgN<<3Q2Wf!@49=AB<^`2`d?}hBOwzWyhLf_id z?k1M*aHEUPIa>5wnj2(By=BgZO;c#JN3W1n^AmKbaz}DN__$RFMFzOit+lU6(np-C z33Tz#nd05I3yEwgyWYGEZWBdqC7+#TOch<5g?XnbLvrSa;?8a?9`7Z+~3A3&|CRH zl0c%p;|6r_&3KwwSa51E_NRz%4-l*9ozhlw_a+e_a?oCvmQlFht7G8ZdfVMVqe!L_ zJJ?=-z?fokE--n-X|2+H4rX?Edmsaq4xvxKQM1A&jt^G zy{CWrJB{sVxCH&EJf*Qj#_Y@T3%gj{SP{OVwk4ruPMP$2&Uw*DDL+uwKHj1;tULph zX~9VF2d1tTm^3&xv$w`|Bbh+5L`S(8L6xv){sl8Ml_TC+BWXF9|Zqdx<4Gm}JwVq*+b&6G) zYf0_I2%x1S{kFNcro6io5}nz`(6&&WGBaE(K!hNYV+(TdVRmDP6kB)ihB>)SL+x?f7GcNJwiimbw;6*w%>%j>_Zoxd;WItxJh09l-0=Y zOCl!f0!Xss7Ha!AXuyicWugd`kL!)PT_9VBR40rxDmO~q&eVHlQT76y{-}u_s$Cgf zqGEn2!U>!T3k}&#ibpa{%{LmARx6wzWelhZvUliSNddEfli%k>$ro(Fy8=hzB!yS; znbDe5%!^02P6gD3B5Mte>u((gqg{UGGf~J%0W>9DBY!L8+V81c)5mBGCi`%Do7{V2 z%s&4i&WE_&%6E=LPnA5nTNg*~Zz8maHEW@S`&$cLk8 zMlz$x*ttLtfuzx111iVB`rjlha-UyD79w&Xv3?2IIibus%81R_mcmCj7{zTSF+PxQQU~REC@nbLZYssp#ECci)%LCJQ}t2uJPtkX0M9%i~DJVq8st}VB$?Q>rFak;cCXor3oH_``2OROJ z7#&F5De+rBTqHJvF2amisyz+-B#OlZ%fz=shu(hnY&yL!QiK3(<+*6+QUkKtaVPO`3(`3-ZIW4Z zb+n@5<=lK(`nrCL@Cme>e;W`KWdVugk)1rV$%rr78(eN5fVxMED}g)ykva1_Dzw6H zj)(-@!+O)zMZQ=Y+)HxO!|YK8El89<>$(SpJo`O6Q>&yfuTu-Qw7r3e0hHz!3-;0G zoD2Ruwb&w}or8PuFe6*)+>&l?ol%Ev3Q>Rjc!f&Ob_1G@NRBp=;@7*ik=C5ruP3{4xCCfjSPQHhP^ejOGad+0lr3y z&wq~2$~kaU^pfA?vZ~3|0?eto8LTUDO3cZ3vufN zr+jg*u%xNwgvy2C z`&=`7U@JS#d-@Qy2IkBfvZ(}Etto%gO8*v0D;KS2r_m3Ohw7U zBl(~;8hdHXin#}Kb)tB);i7OM7`k=tm2_?31>uvv|8d1mM&m8?Ml!K&G!e!q>%Rbng(nDTpP zNiH#v=P~M#fvR9X3@TXR_6@;qQDh~3Sfr$U!mDVN zb=9L$WLgfR=f`c?OMA)D8CHJVU6pSV!-7>ddo{COaf{1 z5y*kKvA}PU?XYmNocLO4Ai`%4Je&m?;z-Db#8GIFNnyHY0ddf~xn@Li8lDm`7Pra$ z$g5;?9_M2Du*3hZcW&-RE(84+zV39(~Dp159MnlizUwwigFK>aT@P-F0J6Sd43P&-i?`XDA}iaQFtp)<&oR3~yPvM*|_E}~qz7dC?EYx%=3aC&y3tm{h&({&!zk8!3Z-x}!32;(-ZsSQq}LaBL1(*aA*V;-_NdzE}|#+?Mew7HVL(Ss!aMOFzs-A2j4vM-8i4q6cIEzwysIjQpjjRaRAD!j}0n z`p*+DK;$06de@LW=2f;~YoN2;uT8Oc(EQnmD{`)o8&QMHZn6;`y0JS1>HGgxtAatV zGSe}4_){iqzhxd=)3}&r-sF&cW4gYriS;dzVockTV$4nLjR{kA*-6$SnM##}ZUfss z%WXx2hs-m#9#%SR%e||vJl9qOOaEsm8Q@%uTP5nFRE=6StSEP~$BIU{=i>&~@ z_Jg`vwDGMUrs4}*H||Smn0yhnig9-3;)Hos-TtcQM18=KiusN8XhJ@}D3qh{+!y%B znn;$Oadw3LUaJ@&P%Qut`E<7ssFWbIR24zj5|HdRQFqU1NXIItO~cUgi5%=)4R98#q zr<+oPh`(jfPZ2pw67=e$cbLY*0Z`MB_+s#Iwk0J{E8!|kYFiDCky)9zTp!I%I())q zK=w;ot~|fKdNw|a6R4!nLC>S+6t2y?!HbiJtAe?UTs&$g9)2vg1xb`ZpZU{`Ct3aA zGXG^&T{Ul^xAm^Qy18sPyt!Pdw{d_9IL6qf_t9>q*B}q=UogRiQ($+{1$Y<@aNuq; zY+SBvT4MKEX_c_Dytmvsu9?QB`r38sy|i1i>@=M~XE=m*ZP-9TAB+i@ppCP88Fft6 z(JJ)RL=!8daJ7_7)fL;#RD7OgEp55_uBF5=ZVVQHzt~KlRoi>bDJsmj=j)*@S2bCv zS~~rmj`Z^@5LU`5iz1f?2QW8eBD1cgZ}9RgocMBOzblJwC7A8!Y{)ElrZ>{I`N(Cw z9T5Tk{P!zN(pVOEhAM`6yZAN{FZh*GLywxNZ0!3^dgn6s=l<&1>lk)$4j@T#KEUN zagpIw+8r-3mbpG^)_#|!UwIFipFWI`4K-^a4>em!v9V%jZJ>utwPn~y&UE}YzAzzY zAAmA9mO}V^WtU0m)9l6@y(CO7*nz)p++L&ydy`@lPeqSVlp4uiVnNCyv|*un(?Y*( zTOffH3K)k+h^c`O7$5@op>}14)Npd7dWi7Tz^ejyKZ*n}l1KOkXiO z)=@PWc1*qKIf=}{3nhf&JkB8y8?J;Tq`7SbMWUThkyVjEk16gN;v~fB;6cr{5!S+- zny9nLN+s;tyZY~dTut1|+WJITHI#d;gC6@5AjZ|C$kA8z0RFnHLmv@LJFZI!tEFd& zKSc3f*NO(4k_C)dnhNDuz+p(NM_XtnESHFLQ}9)r1)klS^8YiKfEqAR=?^!InT38Zw*zB zh|lN}%4h!BzG;vabB*=g7qe6%ueIvXKWrHq3+qaG0+cL_olx_QZ4JTq5X4|MG)EsZ zuoH9&c)C13$Wewu-kf5!jz}>pEyT|0dSSTG4Xm&y+VTX=r2NKF!g)=5e)U`N9u@yc zmwOFyxn@}YeEYXI$2d}Z{c_}cary*IhKJ!`gkSk!b)ngeo)XSrYN3$+6i0X~&U6he za47eR3;+xRj%QwBzY==(du~@g)O;u9UU0u~zpuR27L%(IXZW56Z(ri$Cw`= z-y$D?^4Fm-JsAuo-!aCEGq%;7Xw#1fy-yD(fJ!X#&u=cE$UW-SJ?ta>c1((Nyam@+ zM9(4mJksOZBuRK_?=W*G3WMb<9-zJ0@QQTqqb+mHMwa~?8FDiz{A}%#RsqJ54W+Wad5C0)t&vJ#4xU|v zDZ@3CJAzMub-!JJ7>GoOr|=mH2Yf&+z_NxZY+wdo5hXlK7W{}Lk^q>~DmZsC23m3N zfChnS3TEic6$Z#V zvcaRhNT`wrGR)?!+RbWUemw0_Q{@fi!=5v13c6wy=-*4Eufv0@JAY4l(MppmfMVBX zp2HP0qyMSc9#v|u9?ABG!P^P3mU_!p3NzT=24?T=y1xbUME1^CsgpK3bshs|P^Lpv z%Cqijn#QK4G=pYHHo1^i86*rgxgQ{0bj942m$21eQb(VxoR{!T=s}%AL#zawyqF^8 zs73x(XZgEFmEQbJ=#LQYu z%6C~7v}fZu!q6u&rN?L|{p2Q2_8;FjmI-QjtNSU1rseN?8mo9c??fh1!^CD%MF7W{ zzk~=&*ke^vN_@NB5FM#XGYSuh2`$^lzDmn{r0F30R>l$LPNVsFSzXyxSxK>O*yB2T z6?lBN#l&~#RmN3HJVgdH_r|y>&ihb??QuYu4F_F>y~A1%vSR{-Nu?v1)rFh=BDUfk!@8}#rm>!(ixZSED4t%8zlLX zO;z5Jb?Z&3h9gaqHPc{oHP<+Cx6964@~S5%LyeXl4}%?-9bW2lCUioR7@(cO>nHt7 z-!%$H>QC&}BttwXRYysaf@8pC9(m6Z+?v&N;3%NJrIL1P+>WnF)Pk$Yl~uJK-g}GW z#L!ndgLDg-n#;aaQnWxkWq20B91vG88YB9*bHv-9<Sh5At z1mh8_VnMmUEKyc6qFF+1QS+|zk5_@YE((GygbBkB2v`(i1sBE}`fEiqir&0;$F1c2 zIl{OHGj++6GEiki3(&|4QPNi}*+WZP($Kf0dKV`85kL=ST9O=6s9%3o*xEang_atC zM9mQKbA-$qx=#QW%2uVa3P5nSP@v}YQzDSyO(kn0YWdxVo>gd6HA){!3KoU4`e_`> zkyW9gRn^JK)D4p3^n+D|11nCwHQ*Tj+r|D0VWYC!@>zA7P8vES-MfUuw}I65LX0GD6yv;P!q$&pN{ql^3KjM$ha zBxED@Ch$zL$DAIwRqxLi$~J|w2j~u5^;725xhI)%q(e1+Cr%lpEU0sO-rT9@61Lc- zXh8>QOk3)wB&J)hnGb@Ra*e^Jp_4JrzN>F`o4-=E#9=bgn$v-rf=8S5R_5mYymUT* zoGhmUAa&Ki`a4+L)@^Qrn5v+pP2FBrFhwfl_S60LhiRapu0HkAfwRNQ(9)>}pqu`J zR!$otlQ+Of6b#mg2Jf#N2AO+rMpBQRs$$@1sZ6|?V3&wM*t}pL*BBd|?Plzm9_%^0D7pSF)mtbS=zeBx^jsbN80~_QP3=Zh$0RinL zz7hg&t%7C3q9^>BP=fzy2vLfHmE>;+24Nx}QO}!JbV%rm*EFKf_CI_7*G}s2llkCK zt1=x1Afl(K$|}bfi~NL0_W}(Y>5%QtA)P&XV@g_5XO!CnP}CCehh`wrRUdY z{!Z0`0@VNo$CSpKvg=fZ7oY(K0p45H$b$*g(mZsP$H64{qyDyyy`8W3WL^E&q$M4a zDI3Orn!xyJVd^ve_k$?L%0Ak4B4?u7bprg9e!4}xT|li(PCBPPdYe5hS(rnr=esH; zXOO%r({H;fW0aweTmF9(G`yM88^X^o|y5hbDW{*1n?6;w6Kl?Mk896v^ z+Y~foj8-4~0zsib(H5Bz%LZQ1)#U zH6mj~C6nyLSh9b^+xvd!egFB+?>WzNuI0Y3-*fK!T<6@+eVzL$>x$5?H&qc54s;@G z9!ua|a?Xs=5coV)qe!v3P7}ENxB_sttUMh2`t7+>Sk1`hxu}Zz>VZW5vGcc@Ed+-S zdpzRW#5P+nYpwb7AivmRCM@tAof%wqpo37fM`3KJS$KG}i}l-sH(rYusp__c)2+6M z*f4};hy1)htdmi?$!~n6iUAqJnZ(zAs$f70ERvIj%=So+_R(UeN^xk zHRBPAU|e*Djzw*c1Ex{3Vm!X^%x}g&dxK$unXij1NHBhOJ1!wk4(*66B?sa+xUlM_ zNE7_PrdRndnrV_49ub_;%LopU8J)W$-z3*SbFyZHw~J-$ zRmsQ_-U8ajsze6`Ir~pYeg2U`ep*!GID`w11M61fW_N+1k@GikbmNd#IUHx8HHWK8OX}cZZMqD1Qx;GyI}49?RF` zX!@1gHFCMloW<|EWi{45*RB?JI^{(b6`NwgjuMnbGsQWkk*=Z()w6FZjS3_T7Oy&< zHdV+`Y>}fUR6Id-HIr(d3^tRhuB7Ta+S6{Fmi%~}(U<^awn@}~< ztXIbJ%?Gc_6MU(-L9xNREb;*a>Wn3Qv0DcGo<+DYPBdN!O{p2Qjl|IAgL6saRB}j2 zm@&eaN}B+;k%P)vId!UzOZ~TMdG#4(^iaEu++`F99O?k20!Js}trrIwgJA(nc!AKQ zmzbKlH=U?1gUMNKfJ!9n_(e%eV$lX@=T5j&x=ZPl7z9^>0Un1CeK%}S?_fXm;*P6^ zVXfay&NnF#fzli0Jn3r!byjEJJ|>0hSaFk-v0L;F_`$*1HQU9R z{(wyL^Y_a#-|<>nU8>2!OS~BBfv(aA2{Mo9q5*i;1RYrEiBY0{T^{A~N~^1vmnX~! z;j439gY&@Rw}Hg0T&6I-ixTnIbm1rq^>6 z6BuUVTm(0ri@b0x{mMB1XP8o-BHTnN7TIU(RK3(*LbnM@NZQ1@$;(krZ&QvF64MY- z7m<5Ke`H~U_!)NLA@w~uWd7RZy9HKdX)<=z5bN_gdbA9VeH(#o83rWt<8odkr>tDs z%&V9bCiH6o^moinnhk2wuJBaSSB?BY&3sS(haIQeQjRxHIo^U7)ZzxK!UstmzN1A# zqrPY7W>G$y88Jq6GT6GeIYYNs7oNfmpuxySp*hgvdgKKG@>KJoVFjDe%dJ zbk$~NGodSJ#)Vw7L|};U_s-lDzk{c@FEn^*O_JGPb0Zz`Esfn_Wt)!_@8GK`p=gvc z6)0LNev+E6G}pmKBeJR;L6sB&THBT7PSh$ztwBFfywC{c2Rrdc(b`#QoN>(9eADrx zok2hW=$M0=YBKok_RdCx6*zl4-?PqvS&aLUIVZR-_4P~6NQ7+P=q@X*?MQF^04~#P z=!6ycS$uxmks;WhR)EkiE-5`{IMFrTFG`L(iB?xmxnpMoqe)=5M>P_k?md}A23*+F zBdUKJXUv^3hkU2x)NL<#6b_gu&8OJE4}KA<(+QeQ3bC!Ewb=V;Is0Q_c}XX250qM? z_IM_0_%8IPElF0=2-x$(L3gpXd9;&Q_5CTyc7Pqs&rkfs!q3Mje#t6dVF$}e+OW6h zz&ukAXn6xf7cG~?gVbz(chB~oN>aB?q1j=@Z z=5t=CB8o=mg-$c7aN&H16+iRz1YyV6iukH!13U~?P+|=U4`EhP5+-q5709I)RQ|_AGW*3@~Hck zm`f1rzAXr3bPMG4bjN4%Tif(H@ATTU>Gi?*nnU;!GkW2K>&sLs(4LF0Jw>_%;a|Nq z>od1ZP;8>k0TeBh?6O=5B3+LmQH)HRz^09QWdoqach$Jcrvqp7$k$a9YMmf`Qoxrc z6Ov42{_6Otb~^>xrF^1Owc^c>)1$0KS0Mek?TNoJ%FY4q|e6J~|>{|!$=YxMgf4AoY zZuM6S%0k=zkMYg4_J87X+kwvZY|JkF!zjB4eiuu;*0ECFu+PvB5%|JY9#{=j(y2&Y z=9!aci`Y?M^6XRgn}GRFd?ChVzLSds=Ig8E4(kOydgMmCeQ_hX;jf{ zlF)3_)ntkkG^iIeLD=WCevcR@*8U%ZRp&oeD>>(;G{+SXcazgYQtFWrkCD+_WcXcV z)c5689~1oyqt{|P0QI%4#zL>l>1iPj Tn-oQL4UG^YFxVl?VUhm=r%lFU diff --git a/Einführung in die Neurowissenschaften - Fragenkatalog.tex b/Einführung in die Neurowissenschaften - Fragenkatalog.tex index e0e3548..395c0d8 100644 --- a/Einführung in die Neurowissenschaften - Fragenkatalog.tex +++ b/Einführung in die Neurowissenschaften - Fragenkatalog.tex @@ -646,212 +646,224 @@ \question Nennen Sie die drei Hirnhäute! \begin{solution} - + Dura mater, Arachnoidea, Pia mater \end{solution} \question Welche Hirnhaut grenzt unmittelbar an den Cortex? \begin{solution} - + Pia mater \end{solution} \question Welche Hirnhaut grenzt unmittelbar an den Schädel? \begin{solution} - + Dura mater \end{solution} \question Über wie viele Arterien erfolgt die Blutzufuhr zum Gehirn? \begin{solution} - + 6 \end{solution} \question Durch welche Struktur kann der Ausfall einer der zuführenden Arterien ausgeglichen werden? \begin{solution} - + Durch einen Ring in der Hirnbasis \end{solution} \question Wie viele Hirnventrikel gibt es? \begin{solution} - +5 \end{solution} \question Wo und durch welche Struktur wird das Nervenwasser gebildet? \begin{solution} - + In den Ventrikeln( durch Kapillargeflechte der Pia mater) gebildet \end{solution} \question Wo wird das Nervenwasser wieder resorbiert? \begin{solution} - + Arachoidalzotten im Sinus sagittalis superior \end{solution} \question Wo befinden sich weiße und graue Masse im Rückenmark? \begin{solution} - + weiße Masse außen, graue Masse innen \end{solution} \question Wo endet das Rückenmark beim Erwachsenen? \begin{solution} - + Obere Lendenwirbelsäule \end{solution} \question Was sind die beiden wichtigsten Grundfunktionen des Rückenmarks? \begin{solution} - + \begin{itemize*} + \item Verbindung zwischen Gehirn und dem größten Teil des restlichen Körpers + \item Implementierung wichtiger somatomotorischer und viszeraler Reflexe + \end{itemize*} \end{solution} \question Wie viele Spinalnervenpaare gibt es? \begin{solution} - +31 Paare \end{solution} \question Was ist ein Dermatom? \begin{solution} - +\begin{itemize*} + \item Assoziation zwischen Körperoberfläche und Spiralnerv/ Rückenmarkssegmente + \item Klinisch bedeutsam für diagnose von Schäden +\end{itemize*} \end{solution} \question Nennen Sie die drei versorgenden Arterien des Rückenmarks! \begin{solution} - + \begin{itemize*} + \item A. spinales posterolateralis (paar) + \item A spinales anterior (unpaar) + \end{itemize*} \end{solution} \question Welcher Anteil des Rückenmarks wird über die Arteria spinalis anterior versorgt? \begin{solution} - + Vorderen zwei drittel des Rückenmarks \end{solution} \question Welcher Anteil des Rückenmarks wird über die beiden Arterii spinalis posteriolateralis versorgt? \begin{solution} - + Hinteres drittel des Rückenmarks \end{solution} \question Nennen Sie die drei Häute des Rückenmarks! \begin{solution} - + Dura Mater, Arachnoidea, Pia Mater \end{solution} \question Zwischen welchen Rückenmarkshäuten befindet sich Nervenwasser? \begin{solution} - + Pia Mater und Arachnoidea \end{solution} \question Zwischen welchen Rückenmarkshäuten befinden sich venöse Blutgefäße? \begin{solution} - + Epiduralraum (zwischen Knochenhaut und Dura) \end{solution} \question Was wird durch Schädigung oder Durchtrennung der ventralen Wurzel verursacht? \begin{solution} - + schlaffe Lähmung \end{solution} \question Was passiert bei schlaffer Lähmung mit den betroffenen Muskeln? \begin{solution} - + Atropie (Rückbildung der Wurzel) der Muskeln \end{solution} \question Was ist der Krankheitsmechanismus bei Amyotrophischer Lateralsklerose? \begin{solution} - + Absterben der 1. und 2. Motoneuronen im Vorderhorn des Rückenmarks, Tod normalerweise + innerhalb von 5 Jahren \end{solution} \question Nennen Sie drei wichtige Ursachen für Querschnittslähmung! \begin{solution} - + Linearfraktur, Kompressionsfraktur, Trümmerfraktur \end{solution} \question Was sind die Auswirkungen einer Durchtrennung des Rückenmarks bei C4? \begin{solution} - + Tetraplegie (Lähmung ab dem Hals an) \end{solution} \question Was sind die Auswirkungen einer Durchtrennung des Rückenmarks bei L1? \begin{solution} - + Paraplegia, paralysis below the waist \end{solution} \question Nennen Sie drei Ursachen für Bandscheibenvorfälle? \begin{solution} - + Genetische Prädisposition, einseitige Belastung, Schwäche der paravertebralen Muskulatur, Altersbedingte Degeneration \end{solution} \question In welchem Wirbelsäulenabschnitt treten die meisten Bandscheibenvorfälle auf? \begin{solution} - + Lenden-WS \end{solution} \question Nenne Sie zwei wirksame prophylaktische Maßnahmen gegen Bandscheibenvorfälle! \begin{solution} - + Aufbau der paravertebralen Muskulatur, Rückengerechtes Heben/Sitzen, Aufgrund genetischer Ursachen kann trotz Vorbeugung ein BS auftreten \end{solution} \question Welches sind die beiden Grundformen von Schädel-Hirn-Traumata? \begin{solution} - + Gedeckt oder offen \end{solution} \question Deutet eine Bewusstlosigkeit von 45 Minuten auf eine Gehirnerschütterung, eine Gehirnprellung oder eine Gehirnquetschung hin? \begin{solution} - + Gehirnprellung \end{solution} \question Nennen Sie 5 typische Symptome für Schädel-Hirn-Traumata! \begin{solution} - + Bewusstlosigkeit, Übelkeit, Schwindel, neurologische Ausfälle, Amnesien + - Kopfschmerzen \end{solution} \question Nennen Sie 3 Therapiemaßnahmen bei Schädel-Hirn-Traumata! \begin{solution} - + Rihe, Beobachtung (Krnakenhaus), Druckentlastung, Symptombehandlung, Rehabilitation \end{solution} \question Nennen Sie die beiden Grundformen cerebrovaskulärer Störungen! \begin{solution} - + Cerebrale Hämorrhagie, Celebrale Ischämie \end{solution} \question Nennen Sie 3 mögliche Ursachen für Hämorrhagien! \begin{solution} - + Arteriosklerose, Amyloidangiopathie, Gefäßveränderungen, Aneurysmen, Traume \end{solution} \question Nennen Sie 3 wichtige Risikofaktoren für Hämorrhagien! \begin{solution} - + Bluthochdruck, Einnahme von Gerinnungshemmern, Nikotin, Alkohol \end{solution} \question Nennen Sie 2 mögliche unmittelbare Ursachen cerebraler Ischämien! \begin{solution} - + Einengung oder Verschluss von Aterien (Thrombose), Embolie, Arteriosklerose \end{solution} \question Was ist der wichtigste Faktor bei der Schlaganfalltherapie? \begin{solution} - + Zeitlich schnellstmögliche Aufnahme in Stroke Unit \end{solution} \question Nennen Sie 3 wichtige Therapiemaßnahmen bei Ischämien! \begin{solution} - + Thrombolyse, Mechanische Thrombose Entferung. Rehabilitation, Behandlung von Ödemen, Stabilisierung der Atmung \end{solution} \question Nennen Sie 4 wichtige Hirntumorklassen (nach der Gewebsart)! \begin{solution} - + Meningeome, Gliome, Blastome, Metastasen, andere Primäre Hirntumore (Lympphome) \end{solution} \question Welche Klasse von Hirntumoren (nach der Gewebsart) ist am häufigsten? \begin{solution} - + Gliome \end{solution} \question Nennen Sie 5 typische Symptome für Hirntumore! \begin{solution} - + Neu autretende Kopfschmerzen nachts/morgens, Übelkeit, Erbrechen, Sehstörungen, Krampfanfälle, Neurologische Anzeichen (Lähmungserscheinungen, Sprach- und Koordinationsstörungen, + Ungeschicklichkeit), Persönlichkeitsveränderung \end{solution} \question Welches sind die beiden typischen neuropathologischen Befunde bei Alzheimer? \begin{solution} - + Ausgedehnte neuronale Degeneration, Neurofibrilläre Verklumpung \end{solution} \question In welchen Hirnarealen sind neuropathologische Veränderungen bei Alzheimer besonders anzutreffen? @@ -861,42 +873,42 @@ \question Welche Art von Lernen/Gedächtnis ist nicht von der Alzheimerschen Krankheit betroffen? \begin{solution} - + Sensor-motorisches Lernen \end{solution} \question Welcher Neurotransmitter spielt eine besondere Rolle bei der Parkinsonschen Krankheit? \begin{solution} - + Dopamin \end{solution} \question Welche Hirnstruktur spielt eine besondere Rolle bei der Parkinsonschen Krankheit? \begin{solution} - + Substania nigra \end{solution} \question Nennen Sie 5 typische Symptome der Parkinsonschen Krankheit! \begin{solution} - + Ruhetremor, Rigor, Maskenartiges Gesicht, Bradykinese, spezifischer Gang \end{solution} \question Nennen Sie die 2 wichtigsten Behandlungsstrategien bei Parkinson! \begin{solution} - + Medikation von L-DOPA oder Dopaminagonist, Tiefenhirnstimulation in Basalganglien \end{solution} \question Wie hoch ist das Erkrankungsrisiko einer Person, deren Mutter an Chorea Huntington erkrankt ist? \begin{solution} - + 50\%, da autosomal dominant vererbt \end{solution} \question Welche Nervenzellen werden bei der Amyotrophen Lateralsklerose geschädigt? \begin{solution} - + Motoneuronen im Cortex, im Rückenmark oder in Hirnnervenkernen \end{solution} \question Bei welcher Krankheit wird das Myelin der Axone angegriffen? \begin{solution} - + Multiple Sklerose MS \end{solution} \question Nennen Sie die 4 Grundprinzipien des sensomotorischen Systems! @@ -906,117 +918,121 @@ \question Nennen Sie die 5 sensomotorischen Systeme! \begin{solution} - + Eigenreflexapperat, Fremdreflexapperat, Vestibulozerebellares System, Extrapyramidales System, Pyramidales System \end{solution} \question Was sind die beiden Aufgaben des Eigenreflexapparates? \begin{solution} - + Anpassung von Muskellängen, Anpassung von Muskelspannung an die Schwerkraft \end{solution} \question Wie viele synaptische Verknüpfungen befinden sich zwischen Sensor und Effektor des Eigenreflexapparates (ohne motorische Endplatten)? \begin{solution} - + Monosynaptisch (eine synaptische Verbindung) \end{solution} \question Wo befinden sich die Zellkörper der somatoafferenten Neuronen? \begin{solution} - + In den Spinalganglion, keien Berührung zu anderen Axonen mit dem Zellkörper \end{solution} \question In welchem Teil des Rückenmarks befinden sich die Motorneuronen? \begin{solution} - + Graue Masse \end{solution} \question Über welche Nervenwurzel verlassen die motorischen Fasern das Rückenmark? \begin{solution} - + Radix anterior \end{solution} \question Wird ein Muskel i.d.R. von genau einem Rückenmarkssegment versorgt? \begin{solution} - + Jeder Muskel wird von nervenfasen mehrerer Rückenmarkssegmente versorgt \end{solution} \question Was ist eine motorische Einheit? \begin{solution} - + Gesamtheit der von Neuronen innervierten Muskelfaser \end{solution} \question Wie viele motorische Endplatten kontaktieren eine Muskelfaser? \begin{solution} - + Jede Muskelzelle nur von einer Endplatte \end{solution} \question Wovon hängt die Größe einer motorischen Einheit ab? \begin{solution} - + Von der komplexität der Motorik \end{solution} \question Durch welche Sensoren werden die Muskellänge und die Muskelspannung gemessen? \begin{solution} - + Muskelspindeln \end{solution} \question Was ist die Rolle der Gamma-Neuronen im Eigenreflexapparat? \begin{solution} - + Veränderung der Länge der Spindelfasern \end{solution} \question Welcher Muskel wird beim Patellarsehnenreflex inhibiert? \begin{solution} - + Beinbeuger (Bizeps) \end{solution} \question Was ist die Funktion des Fremdreflexapparates? \begin{solution} - + Automatische Reaktion auf Reize außerhalb der Muskulatur \end{solution} \question Welche grundsätzlichen Typen von Haut- und Körperrezeptoren gibt es? \begin{solution} - + Eingekapselte, organartige differenzierbare Strukturen für Tastempfindlichkeit oder freie Nervenendigungen für Schmerz- und Temperaturreize \end{solution} \question Welche Typen von Berührungs/Drucksensoren gibt es? \begin{solution} - + \begin{itemize*} + \item Langsam adaptierend: Druckwahrnehmung + \item Schnell adaptierend: Berührungswahrnehmung + \item Sehr schnell adaptierend: Vibrationswahrnehmung + \end{itemize*} \end{solution} \question Welche afferenten Nervenfasern haben die größte Übertragungsgeschwindigkeit? \begin{solution} - + Aalpha-Fasern (70-120 m/s) \end{solution} \question Welche sensorische Information wird durch C-Fasern übermittelt? \begin{solution} - + Temperatur und Schmerz \end{solution} \question Welcher Typ afferenter Nervenfasern ist marklos? \begin{solution} - + C-Fasern \end{solution} \question Wohin ziehen die Hinterstrangbahnen im Rückenmark? \begin{solution} - + Zur Medulla oblongata \end{solution} \question Wo kreuzen die Hinterstrangbahnen auf die kontralaterale Seite? \begin{solution} - + Im Hirnstamm \end{solution} \question Nennen sie zwei wichtiges sensomotorische Assoziationscortexareale! \begin{solution} - + Posterior-parietal Assoziationscortex, Dorsal präfrontal assotiationscortex \end{solution} \question Woher erhält der parietale Assoziationscortex seinen Input? \begin{solution} - + Sensorischen Arealen (visuellem cortex, auditorischem Cortex, somatosensorischem Cortex, ...) \end{solution} \question Über wie viele Neuronen wird im pyramidalen System die Information an die Muskeln übertragen? @@ -1026,122 +1042,125 @@ \question Nennen Sie die 7 Stationen der Sehbahn! \begin{solution} - + Retina, Sehnerv (2.Hirnnerv), Chiasma opticum, Sehnerventrakt, Äußerer Kniehöcker, Radiatio optica, Primäre Sehrinde, Sekundäre Sehrinde \end{solution} \question In welcher Hemisphäre wird die Information von der Netzhaut des rechten Auges verarbeitet? \begin{solution} - + Linke Großhirnhemisphäre \end{solution} \question Aus welchen drei Häuten besteht der hintere Teil des Augapfels? \begin{solution} - + Hornhaut, Aderhaut, Netzhaut \end{solution} \question Wo befindet sich die Hornhaut des Auges? \begin{solution} - + Von tränenwasser benetzt, vordere Teil der äußeren Augenhaut, frontaler Abschluss des Augapfels \end{solution} \question Worauf wirkt der Ziliarmuskel? \begin{solution} - + Zonularfasern (Bindegewebsfasern) \end{solution} \question Was ist der Vor- und der Nachteil einer weiten Pupille? \begin{solution} - + Nachteil: weniger scharfes Bild;\quad Vorteil: hohe Empfindlichkeit \end{solution} \question Was ist der Vor- und der Nachteil einer engen Pupille? \begin{solution} - + Nachteil: empfindlichkeit gering;\quad Vorteil: schärferes Bild \end{solution} \question Welche Teile des autonomen Nervensystems bewirken die Erweiterung bzw. die Verengung der Pupille? \begin{solution} - + Sympathisches NS und parasympathisches NS \end{solution} \question Wie wirkt Stress auf die Pupille? \begin{solution} - + Die Pupille wird geweitet \end{solution} \question Wie wirkt Müdigkeit auf die Pupille? \begin{solution} - + Kontraktion der Pupille \end{solution} \question Wie wirkt eine Entspannung des Ziliarmuskels auf die Linsenwölbung? \begin{solution} - + Fernakkommodation, gespannte Zonularfasern, flache Linsenkrümmung \end{solution} \question Welche Linsenwölbung bewirkt eine Fernakkomodation? \begin{solution} - + Flache Linsesnkrümmung \end{solution} \question Welche Arten von Fehlsichtigkeit werden durch Sammel- bzw. Zerstreuungslinsen behoben? \begin{solution} - + \begin{itemize*} + \item Sammellinsen - Weitsichtigkeit + \item Zerstreuungslinsen - Kurzsichtigkeit + \end{itemize*} \end{solution} \question Nennen Sie die 5 Zelltypen der Retina! \begin{solution} - + Stäbchen, Zapfen, Horizontalzellen, Biolarzellen, retinale Ganglienzellen, amakrine Zellen \end{solution} \question Welchen Neurotransmitter schütten Fotorezeptoren aus? \begin{solution} - + Glutamat \end{solution} \question Was ist der Neurotransmitter der Ganglien- und Bipolarzellen? \begin{solution} - + Glutamat \end{solution} \question Was ist der Neurotransmitter der amacrinen und Horizontalzellen? \begin{solution} - + GABA \end{solution} \question Welche Zelltypen kontaktieren die Synapsen der Fotorezeptoren? \begin{solution} - + Horizontal und bipolarzellen \end{solution} \question Wie viele synaptische Kontakte befinden sich zwischen Sehnerv und Lichtsinneszellen? \begin{solution} - + 130 Mio \end{solution} \question Welche Zellart der Netzhaut ist dem einfallenden Licht am nächsten? \begin{solution} - + Axone retinaler Ganglienzellen \end{solution} \question Welche beiden Arten von Fotorezeptoren gibt es in der Retina? \begin{solution} - + Stäbchenzellen, Zapfenzellen \end{solution} \question Welche Art von Fotorezeptoren ist für die Farbwahrnehmung zuständig? \begin{solution} - + Zapfen \end{solution} \question Welche der beiden Arten von Fotorezeptoren ist zahlreicher? \begin{solution} - + Stäbchen \end{solution} \question Welche Auswirkungen hat Konvergenz in der Retina auf die Qualität der visuellen Information? \begin{solution} - + Geringere Auflösung, höhere Lichtempfindlichkeit \end{solution} \question Welche Auswirkungen hat laterale Inhibition in der Retina auf die Qualität der visuellen Information?